Fundamentos de a elementar vol08 limitesderivadeasnocoesdeintegral

Page 1


GELSON IEZZI CARLOS M UR AK AM I NI LS ON JOSÉ M AC HA DO

~

edição

FU ND AM EN TO S DE

M A T E M Ã T IC A ELEMENTAR LIMITES

DE RIV AD AS

NOÇÕES DE INT EG RA L

60 exercícios resolvidos com resposta 266 exercícios propostos com resp osta 100 testes de vestibular com resposta

ATUAJ. EDITORA

8


Capa Roberto F ranklin Rondino SyJvio Ulhoa Cintra Filho Rua Inhambu, 1235 - S. Paulo Composição e desenhos AM Produções Gráficas Ltda. Rua Castro Alves, 135 - S. Paulo

APRESENTACÃO I

Artes Atual Editora Ltda. Fotolitos H. O. P. Fotol itos Ltda. Rua Delmira Ferreira, 325 - S. Paulo CIP-Bruil. CatalogaçÃo-ua-Publicação câmara Brasileira do Livro, SP l'uMlIIIIantoa de lI&tellática .l•••ntar / a.l8Oft Iezzi 0 ' 0 [.t al.]. -- são Paulo: Atual. 1977-

198,.

Co-auto",,: Carl08 Murlllc.... i. Oa\'aldo Doles, aamuel BazAD o JOfll~ Ricolau POlIpeU, Nilson Machado I 11. autoria dOB 't'011111•• ".ri. entre 08 autorelll. Cont.11:do: ,.,1. Conjuntoll. funções. 5. ed. __ 1'.2. Logarttlloa. 6: ali.. _. v.,. 'lrigoI1Olllatrla. 5. ali. • •- .... 4. Sequeneia••••trizes, deta1'lainantes, siet..... 4••d. -- v.,5. COlDbinatSriaJ probabilidt«ee. 4. ed. -_ v.6. Complexos, polinomio15, equaçoes. 4. ed. __ v a 7. Geometria analítica. __ v.8. Limites, derivadas, noções de integral. ,. ed. -- va9. Geolletri/il, plana. 4. ed. __ v.10. GeOllletria eapacial. 2. ed.

1.: Hate.1Uca (20 grau) I. Do1ce. Osvaldo I 1938II. Iezlti, GII1l5On, 1939- UI. Bazzu, aMuel, 1946IV. Hachado, Nileon JoÁ, 1947- V. Hurak_i, Car_ loe, 194-3- VI. Pompeu, Jod: Nico1au, 194517. CDD-510.0? 18. -5100'7 1ndicII para cat~ogo sistell&tico: 1. Hatllllllhica : Estudo li ensino 510.07 (17.) 5100'7 (18.)

Todos os direitos reservados à ATUAL EDITORA LTOA. rua josé antonio coelho, 785 Telefone: 575 - 1544 04011 - São Paulo - SP LUYLVVS 24681097531

"Fundamentos de Matemática Elementar" é uma coleção em dez volumes elaborada com a pretensão de dar ao estudante uma visão global da Matemática, ao n(vel da escola de 'P. grau. Desenvolvendo os programas em geral adotados para o curso colegial, os "Fundamentos" visam aos alunos em preparativos para exames vestibulares, aos universitários que necessitam rever a Matemática Elementar e também, como é óbvio, àqueles alunos de colegial mais interessados na "rainha das ciências". No desenvolvimento dos inúmeros cap(tulos dos livros de "Fundamentos" procuramos seguir uma ordem lógica na apresentação de conceitos e propriedades. Salvo algumas exceções bem conhecidas da Matemática Elementar, as proposições e teoremas estão sempre acompanhados das respectivas demonstrações. Na estruturação das séries de exerdcios, buscamos sempre uma ordenação crescente de dificuldade. Partimos de problemas simples e tentamos chegar a questões que envolvem outros assuntos já vistos, obrigando o estudante a uma revisão. A seqüência do texto sugere uma dosagem para teoria e exerc(cios. Os exerc(cios resolvidos, apresentados em meio aos propostos, pretendem sempre dar explicação sobre alguma novidade que aparece. No final do volume o aluno pode encontrar a resposta para cada problema proposto e assim, ter seu reforço positivo ou partir à procura do erro cometido. A última parte de c..ada volume é constitut'da por testes de vestibulares até 1.977 selecion~dos e res<,lvidos o que pode ser usado para uma revisão da matéria estudada. Queremos c Jnslgnar aqui nossos agradecimentos sinceros ao Prof. Or. Fernando Furquim de Alme da cujo apóio foi imprescindt'vel para que pudéssemos homenagear nesta coleção alguns dos ',randes matemáticos, relatando fatos notáveis de suas vidas e sua ob "as. Finalmene, como há sempre uma enorme distância entre o anseio dos autores e o valor de sua obra, g(,star(amos de receber dos colegas professores uma apre· ciação sobre este \'abalho, notadamente os comentários crl'ticos, os quais agradecemos. Os autores


ÍNDICE

CAPfTULO I -

FUNCÕES

I. A noção de função ... . . . . . . . . . . . . . . . . . . . . . . . . . . . . . . 11. Principais funções elementares

l·H 4-H

111. Composição de funções '. . . IV. Funções ínverslveis . . . . . . . . . . . . . . . . . . . . . . . . . . . . . . . ..

10·H l4-H

V. Operações com funções . . . . . . . . . . . . . . . . . . . . . . . . . . . . . .

19-H

CAPfTULO 11 -

LIMITES

I. Noção de limite de uma função

2l-H

11. Definição de limite .. . . . . . . . . . . . . . . . . . . . . . . . . . . . . . ..

25-H

111. Unicidade do limite. . . . . . . . . . . . . . . . . . . . . . . . . . . . . . . ..

26-H

IV. Propriedade do limite de uma função. . . . . . . . . . . . . . . . . . . ..

3l-H

39-H

V. Limite de uma função polinomial V I. Limites laterais

48-H

CAPfTULO 111 - O INFINITO I. Limites infinitos 11. Propriedades dos limites infinitos 111. Limites no infinito IV. Propriedades dos limites no infinito

54·H . . . . . . . ..

63·H 70·H 80·H


CAPfTULO IV - COMPLEMENTOS SOBRE LIMITES I. Teoremas adicionais sobre limites 11. Limites trigonométricos

.

86-H

.

90-H

111. Limites da função exponencial

.

94-H

IV. Limites da função logarítmica

.

99-H

.

103-H

V. Limite exponencial fundamental

CAPfTULO V - CONTINUIDADE I. Noção de continuidade 11. Propriedades das funções contínuas 111. Limite da

{y f(x)

.

lll-H

.

117-H

.

119-H

CAPfTULO VI - DERIVADAS I. Derivada no ponto

Xo

..•••.•••••••••••••••••••••••••

11. Interpretação geométrica 111. Interpretação cinemática V. Derivadas das funções elementares

CAPfTULO VII

129-H 130·H

V I. Derivada e continuidade

133-H

REGRAS DE DERIVAÇÃO

I. Derivada da soma 11. Derivada do produto

135-H ,

136-H

111. Derivada do quociente . . . . . . . . . . . . . . . . . . . . . . . . . . . . . ..

139-H

IV. Derivada de uma função composta (Regra de Cadeia) . . . . . . . . ..

143-H

V. Derivada da função inversa . . . . . . . . . . . . . . . . . . . . . . . . . . ..

145-H

VI. Derivadas sucessivas. . . . . . . . . . . . . . . . . . . . . . . . . . . . . . . ..

151-H

CAPfTULO VIII - ESTUDO DA VARIAÇÃO DAS FUNÇÕES I. Máximos e mínimos 11. Derivada - Crescimento - Decréscimo . . . . . . . . . . . . . . . . . . . .. 111. Determinação dos extremantes . . . . . . . . . . . . . ..

182-H

V. Ponto de inflexão V I. Variação das funções . . . . . . . . . . . . . . . . . . . . . . . . . . . . . . ..

185-H 188-H

CAPfTULO IX - NOÇÕES DE CALCULO INTEGRAL I. Introdução - Área. . . . . . . . . . . . . . . . . . . . . . . . . . . . . . . . .. 11. A integral definida

192-H 196-H

111. O cálculo da integral . . . . . . . . . . . . . . . . . . . . . . . . . . . . . . ..

200-H

IV. Algumas técnicas de integração. . . . . . . . . • . . . . . . . . . . . . . ..

211-H

V. Uma aplicação geométrica: Cálculo de volumes

214-H

RESPOSTAS DE EXERCICIOS

217-H

TESTES

228-H

RESPOSTAS DOS TESTES

244-H

122-H 124-H 127-H

IV. Função derivada . . . . . . . . . . . . . . . . . . . . . . . . . . . . . . . . . ..

IV. Concavidade

153-H 158-H 169-H


Karl T. W. Weierstrass (1815路1897)


CAPÍTULO I

FUNÇÕES Boêmio revela-se em Matemática Karl Theodor Wilhelm Weierstrass nasceu na Alemanha, de família católica liberal.

Neste capítulo resumiremos aspectos essenciais do estudo das funções, feito ao longo dos volumes 1, 2 e 3 desta coleção. Introduziremos mais algumas noções que serão necessárias ao desenvolvimento deste livro.

Weierstrass não gostava de música mas saiu-se muito bem nos estudos. I ncentivado por seu pai, foi para a Universidade de Bonn estudar Direito. Aí se tornou perito em beber e em esgrima em lugar de Direito e Matemática, saindo sem graduar-se.

I. A NOÇÃO DE FUNÇÃO

Em Münster preparou-se para o ensino secundário, onde foi protegido por Gudermann, que iniciou Weierstrass em teoria das funções, seguindo os passos de Abel. Obteve seu diploma de professor aos 26 anos, ensinando em várias escolas secundárias e mais tarde na Universidade de Berlim, onde em suas conferências dava ênfase à "teoria estática da variável", sem nenhum recurso de pontos ou retas móveis, nenhum abandono de quantidades infinitamente pequenas, só ocupando-se dos números reais, da operação de adição e sua inversa e da relação "menor que". O simbolismo de Weierstrass e seu aluno Heine expulsou do Cálculo a noção de variabilidade, sendo desnecessário usar infinitesimais fixos. Estava assim tentando substituir conceitos intuitivos por precisão lógica e demonstrações rigorosas. Weierstrass tentou separar o Cálculo da Geometria baseando-se apenas no conceito de números. Para isso foi necessário definir número irracional independentemente de limite. Chegou à conclusão da existência de um limite de uma seqüência convergente tomando a própria seqüência como o número ou limite e definiu número irracional como seqüência ordenada de um agregado de racionais, contribuindo não só para a definição de número real mas também para um melhor conceito de limites, que é em essência o que temos hoje. Weierstrass fez suas primeiras descobertas aos quarenta anos de idade e foi reconhecido como o maior analista do mundo, notável exceção da idéia comumente aceita de que um matemático deve revelar-se cedo.

1. Definição

Dados dois conjuntos A e B, não vazios, chama-se relação de A em B um conjunto formado por pares ordenados (x, v) em que x E A e V E B. Exemplo

Se A = {a, b, c, d} RI = {(a, Dl}

e

B

=

{O, 1, 2},

R2 = {(a, 1),

(b, O),

(b, 1),

(c, 2),

R 3 = {(a, O),

(b, 1),

(c, 1),

(d,2)}

são três exemplos de relações de

então: (d,2)}

A em B.

2. Definição Uma relação f de A em B recebe o nome de função definida em A com imagens em B ou aplicação de A em B se, e somente se, para todo x E A existe um só V E B tal que (x, V) E f. No exemplo anterior, só a relação R3 é uma função, pois em RIos elementos b, c, d não participam de nenhum par e em R2 o elemento b participa de dois pares.

l-H


.J

3. Lei de correspondência

4. Domínio e imagem

Geralmente, existe uma sentença aberta y; f(x) que expressa a lei mediante a qual, dado um x E A, determina-se o y E B de modo que (x, y) E f. Assim, por exemplo, dados os conjuntos A; {O, 1, 2, 3} e B; {O, 1, 2, 3, 4, 5, 6, 7, 8, 9} e a sentença aberta y;' x2 , é possível considerar a função:

Chama-se domínio da função f: A ----4 B o conjunto A. Notação: D(f). Chama-se imagem da função f: A ----4 B o conjunto constituído pelos elementos y E B para os quais existe algum x E A tal que (x, y) E f. Notação: Im(f). Chama-se contradomínio da função f: A ----> B o conjunto B. Notação: CD(f). Por exemplo, se A; {O, 1, 2, 3}, B; {O, 1,2,3,4,5,6,7,8, 9}e f: A ----> B é definida pela sentença y; x 2 , temos:

f ; {(O, O),

(1, 1),

(2,4),

(3, 9)} 2

de

A em B, cujos pares (x, y) verificam a lei y; x . Para indicarmos uma função f de A em B que obedece à lei de correspondência y = f(x), vamos usar a seguinte notação: f:A--->B

A

x~f(x)

f ; {(O, O), (1, 1), (2, 4), (3, 9)}

Freqüentemente encontramos funções em que a lei de correspondência para obter y a partir de x muda, dependendo do valor de x. Dizemos que essas funções são definidas por várias sentenças. Exemplos

1«;»

f:IR ---> IR

tal que

f(x) ; { 1, se x":; O -1,sex>0

é uma função definida por duas sentenças: y ; 1 ou y ; -1

2«;»

(quando

x..:; O)

(quando

x> O)

se f(x); O, se { x, se

x

<O

O..:; x x;;" 1

D(f) ; {O, 1, 2, 3} Im(f) ; {O, 1, 4, 9} CD(f) ; {O, 1, 2, 3, 4, 5,6,7,8, 9} É evidente que, para todo f, Im(f) C B. Lembremos ainda que, feita a representação cartesiana (gráfico) da função f, temos: I) Domínio D(f) é o conjunto das abscissas dos pontos do gráfico, isto é, o conjunto das abscissas dos pontos tais que as retas verticais por eles conduzidas interceptam o gráfico. 11) Imagem Im(f) é o conjunto das ordenadas dos pontos do gráfico, isto é, o conjunto das ordenadas dos pontos tais que as retas horizontais por eles conduzidas interceptam o gráfico. Exemplos

f:IR ---> IR tal que -x,

B

y

y

<1

é uma função definida por três sentenças: y ; -x quando x E] -00, O[ ou y ; O quando x E [O, 1[ ou y ; x quando x E [1, +oo[

As funções definidas por várias sentenças têm uma importância especial neste livro.

2-H

l(

D(f) ; [-2, 1] Im(f) ; [O, 4]

l(

D(f) ; IR: Im(f) ; IR+*

3-H


• Uma função está bem definida quando são conhecidos O(f). CO(f) e a lei de correspondência y = f(x). É comum. entretanto. darmos apenas a sentença aberta y = f(x) para nos referirmos a uma função f. Neste caso. fica subentendido que O(f) é o conjunto formado pelos números reais cujas imagens são reais. isto é:

Chama-se grau de uma função polinomial f. não nula. o número natural p p. tal que ap "* O e ai = O para todo i

>

Exemplos 1?) 2?) 3?) 4?)

x E O(f) <= y = f(x) E IR

f{x) g(x) h(x) i(x)

= 1 + 2x + 5x 2 + 7x 3 tem grau 3 = 2 + 3x 2 tem grau 2 = 1 + 4x tem grau 1 = 3 tem grau O

Uma função polinomial do tipo f(x) = k. isto é. uma função em que ao = k e ai = a2 = ... = O é chamada função constante.

5. Funções iguais Duas funções f: A ~ B e g: C -->- O são iguais se. e somente se. A = C. B = O e f{x) = g(x) para todo x E A.

y

Exemplos 1?) Se A={-1.0.1} e B={0.1.2.4}. asfunçõesf:A-->-B e g: A -->- B dadas por f{x) = x 2 e g(x) = x4 são iguais pois:

o gráfico de uma função

constante

é uma reta paralela ao eixo dos x. pelo ponto (O. k). A imagem é o conjunto

lO. k)

Im = {k}. f(-1) = (_1)2 = (_1)4 = g(-1) fIO) = 0 2 = 04 = g(O) f(1) = 12 = 14 = g(1) 29)

x

Se A=IR* e B=IR. as funções f:A-->-B e g:A-->-B dadas e 9 () x = X2 - 2x sao Iguais POIS. para to d o x E IR * • temos: x x x2 - 2x f(x) = x - 2 = - . (x - 2) = = g(x)

por f( x ) = x - 2

N.

x

• •

Uma função polinomial que apresenta ao = b. ai = a "* O e a2 = a3 = ... = O

é chamada função afim. portanto. afim é uma função polinomial do tipo f(x) = = ax + b. com a"* O.

o gráfico

x

(-

~. O).

de uma função afim é uma reta passando pelos pontos (O. b) e

Quando a

> O. a função afim é crescente e. se a < o. ela

é decrescente.

Sua imagem é IR.

11. PRINCIPAIS FUNÇOES ELEMENTARES

y

8>

O

6. Funções polinomiais Dada a seqüência finita de números reais (ao. ai. a2 ..... anjo chama-se função polinomial associada a esta seqüência a função f: IR -->- IR dada por: f(x) = ao + alx + a2x2 + ... + anx n Os reais ao. ai. a2 ..... an são chamados coeficientes e as parcelas ao. alx. a2x2 ..... anx n são denominadas termos da função polinomial. Uma função polinomial que tem todos os coeficientes nulos é chamada função nula.

4-H

x

5-H


*

Uma função polinomial que tem ao = c, ai = b, a2 = a O e a3 = a4 = ... = O é chamada função quadrática, portanto, quadrática é uma função polinomial do tipo f(xl = ax2 + bx + c, com a O. O gráfico de uma função quadrática é uma parábola que tem eixo de simetria

*

na reta x

2~

= -

e vértice no ponto V (-

2~

,-

:a

l. Se a> O, a parábola tem

concavidade voltada para cima e, se a < O, para baixo. Conforme d = b2 - 4ac seja positivo, nulo ou negativo, a intersecção da parábola com o eixo dos x é formada por 2, 1 ou nenhum ponto, respectivamente. Assim, são os seguintes seis tipos de gráficos que podem ser obtidos para funções quadráticas.

v

a> O

V

e

t:.>0

t:.<0

Dado um número real x, seja P sua imagem no ciclo trigonométrico. As coordenadas de P em relação ao sistema uOv, OP2 e OP I , são chamadas cos x (cosseno de x) e sen x (seno de x), respectivamente.

A

u

B'

Chama-se função seno a função f: IR --> IR que associa a cada real x o real OP I = sen x, isto é, f(x) = sen x. São notáveis as seguintes propriedades da função seno: 1~) sua imagem é Im = [-1, 1], isto é, -1';;; sen x .;;; para todo x E IR; 2~) é periódica e seu período é 211'; 3~) seu gráfico é a senóide.

a>O

e

v

7. Funções circulares

I

v = sen x

I

I

I I

I IV

x

x

x

I V

v

v

V

x

I

Chama-se função cosseno a função f: IR --> IR que associa a cada real x o real OP2 = cos x, isto é, f(x) = cos x. São notáveis as seguintes propriedades da função cosseno: 1~) sua imagem é o intervalo [-1, 1], isto é, -1';;; cos x .;;; 1 para todo x E IR; 2~) é periódica e seu período é 211'; 3~) seu gráfico é a cossenóide. y=cosx

I I

I

a< O e

t:.>0

6-H

a<O e

t:. = O

x

7-H


11

Para x E IR e x =1="2 + k11 (k E Z) sabemos que cos x =1= O e, então, existe o quociente

sen x cos x

denominado tg x_

Dado um número real a, com O < a =1= 1, chama-se função exponencial de X f: IR ----> IR definida pela lei f(x) = a _ Destacamos as seguintes propriedades das funções exponenciais: X 1~) sua imagem é IR:, isto é, a > O para todo x E IR; 2~) se O < a < 1, a função é decrescente e, se a> 1 a função é crescente; 3~) seu gráfico tem um dos seguintes aspectos:

base a a função

Chama-se função tangente a função f:{x E IR Ix =I=~+ krr} ----> IR . a ca d a x o rea I tg x associa

8. Funções exponenciais

que

sen x .IStO e, - f(x) = tgx. cos x Destacam-se as seguintes propriedades da função tangente: 1~) sua imagem é IR, isto é, para todo y E IR existe um x E IR tal que tg x = y; 2~) é periódica e seu período é 11; 3~) seu gráfico é a tangentóide. = --,

y

= tg x

x

x

EXERCICIOS

H.1

Construir os gráficos das seguintes funções definidas em IR: ai f (xl

=

{1.2.

=

{-1.

1

-311

2'

O

se x";;;; se x O

>

x

b)

f (x) 2

se x";;;; 1 x. se x 1

>

cl f (x) = {x, se x =1= O 3 1. se x = O

r

x. se x <-1

d) f 4 (x) =

O, se -1 ,,;;;; x ,,;;;; 1

x, se x> 1

x+1.sex<o el fs(x) = { (x _ 1)2, se x;;. O x 2 + 2x + 1, se x,,;;;; O + 1. se x O

t) f 6 (xl = { x2

8-H

>

9-H


H.2

Construir os gráficos das seguintes funções elementares:

a) f(x) = Ixl, b) g{x) = c) hlx)

d) i(x)

1:1

=..!..x' =~2 x

isto é,

se

f(x)

x *0

~

{x, se x;;;' O -x, se x O

<

e glO)

~

Isto quer dizer que a função F leva cada x E A no elemento F(x) obtido da seguinte forma: sobre x E A aplica-se f, obtendo o elemento f(x) E B, e sobre f(x) aplica-se g, obtendo-se o elemento g(f(x)) E C, também chamado F(x).

O.

9 o f X -J-r-

x

(lê-se:

"g círculo f").

O.

*

O.

10. Exemplos

e) j(x) = x 3 .

H.3

A função F, composta de 9 e f, também pode ser indicada com o símbolo

Construir o gráfico da

1 C?) funçã~

f: IR

IR

cos x, { 2 x,

dada pela lei:

para

para

x

x ~ O

>O

Consideremos os conjuntos

A

~

{-1, O, 1, 2},

B

~

{O, 1, 2, 3, 4}

C ~ {1, 3, 5, 7, 9}. Consideremos também as funções f:A - > B tal que f(x) ~ x2 e g:B-> C tal que g(x) ~ 2x + 1.

~

É imediato que: ~

f(-1)

1, fiO)

O,

1(1)

1 e

f(2)

~

4.

Também é evidente que: g(O) ~ 1,

111.

COMPOSiÇÃO DE FUNÇÕES

9. Definição

Dadas as funções f: A - > B e g: B - > C, chama-se função composta de F: A -----.,. C definida pela lei F (x) = g(f(x).

9 com f a função

g( 1) ~ 3,

g(2) ~ 5,

g(3) ~ 7,

e g(4) ~ 9

Neste caso, a função composta F é a função de A em C que tem o seguinte comportamento: F(-1) F (O) F(l) F(2)

~g(fH))~ ~ ~ ~

g(f(O)) g(f(l)) g(f(2))

o esquema

~ ~ ~

g(1) g(O) g(1) g(4)

~ ~ ~ ~

3 1 3 9

ilustra o que ocorreu:

B

10-H

11- H


A função F tem também uma lei de correspondência que pode ser encontrada se procurarmos o valor de F(x): F(x) = g(f(x)) = 2 • f(x) + 1 = 2x 2 + 1 De forma geral, para obtermos a lei de correspondência da função composta F = 9 o f devemos trocar x por f(x) na lei de g. 29) Sejam as funções de IR em IR :f(x) = sen x e g(x) = x 2 • A composta de 9 com f é a função F: IR ~ IR tal que:

3x2 2Ç») A função F(x) = cos e + I como seria decomposta? Olhando o esquema para calcular F(x), temos: x2 + 1 e 3x2 + I ---+. cos e 3x2 + I x--f~-~ 3 9 h então F é a composta ho(gof), sendo f(x) = 3x 2

+ 1, g(x) = eX e h(x) = cos x.

F(x) = (gof)(x) = g(f(x)) = (f(X))2 = sen 2 x 3Ç») Sejam as funções de IR em IR :f(x) = 2x e g(x) = eX. A composta de 9 com f é a função F :IR ~ IR tal que: F(x) = (gof)(x) = g(f(x)) = el(x) = e 2x

11. Observações

1':1) A composta gof só é definida quando o contradomínio de f é igual ao domínio de g.

2~) Quando A = C, isto é, f:A _ _ B e g: B __ A é possível definir duas compostas gof = FI e fog = F2 .

EXERClélOS

Assim, por exemplo, se f :IR+ ----.IR é dada por f(x) =.../X e 9 :IR ---> IR+ é dada por g(x) = x 2 + 1, temos:

H.4

Se f:A ---> B é dada pela lei f(x) = X - 1, g:B - - C é dada por g(x) = = 2x + 1, A ~ {1, 2, 3}, B ~ {a, 1, 2, 3, 4} e C={a,1,2,3,4,5,6, 7,B,9}, determinar os pares ordenados que constituem gOf.

H.5

Se f e 9 são funções de IR em IR dadas pelas leis f(x) = x3 e g(x) as leis que definem as compostas: gOl, fOg, fof e gOg.

H.6

Sejam as funções reais f(x) _- x + 2, g(x) ~ x2 e h(x) = 2 x. Determinar hOgOI e fOgOh.

Ftlx)

(gof)(x)

g(f(x)) = (f(X))2 + 1 = (.../X)2 + 1 = x + 1

F 2 (x) = (fog)(x) = f(g(x)) = sendo

FI :IR+

~

IR+

e

v"'"9"W

=

R

+1

F 2 :IR __ IR.

De maneira geral, quando ambas existem, gof e fog são funções distintas e isto nos obriga a dobrar a atenção quando compomos.

H.7

12. Para a compreensão de alguns assuntos deste livro é fundamental que saibamos decompor (sempre que isto for possível) uma função em duas ou mais funções elementares.

(sen X)2 9

12-H

I x2 + 1 I

F(x) = sen (x 2 + 4) F(x) ~ tg x3 F(x) = tg2x F(x) = 2cOS x f) F(x) = sen 3 x b) c) d) e)

1Ç»)

X --,-----+. sen x

x + 1, obter

Determinar funções elementares I e 9 de modo que gOl = F, quando F é uma função real dada por uma das leis abaixo: a) F (x) =

Exemplos A função F (x) = sen 2 x deve ser vista como F (x) = (sen X)2, portanto, F é a composta gof, sendo g(x) = x 2 e f(x) = sen x, uma vez que o esquema para calcular F(x) a partir de x é o seguinte:

~

H.a

Determinar as funções elementares f, 9 e h de modo que hOgOf = F, sendo F uma função real dada por F(xl = cos 2x+3 .

13-H


IV.

FUNÇÕES INVERSIVEIS

13.

Dada uma função

f:A ---.. B,

14. consideremos a relação inversa de

f-I = {(V, x) E B X A

I (x,

y) E

f:

f}

Quase sempre f-I não é uma função ou porque existe y E B para o qual não há x E A com (y, x) E f-I ou porque para o mesmo y E B existem XI, X2 E A com XI X2, (y, XI) E f-I e (y, X2) E f~l. Vejamos dois exemplos:

Definição

Uma função f:A ---.. B é invers/vel se, e somente se, a relação inversa de f também é uma função, isto é, para cada y E B existe um único X E A tal que y = f(x). Indica-se a função inversa de f COIr a notação f-I.

"*

15.

Observações 1~)

f

é

função

f -I

não é fu nção

B

Sendo

f-I

a função inversa de f, temos as seguintes propriedades:

a) DWI) = B = Im(f) b) Im(f- I ) = A = D(f) c) (y, x) E f-I <= (x, y) E f

A

f-I

d) o gráfico de y = x.

é simétrico do gráfico de f em relação à ret.'

2a )

Dada a função inversível f:A ---.. B, definida pela lei póra obtermos a lei que define f-I procedemos assim:

f-I

não é função

f(x),

a) transformamos algebricamente a expressão y = f(x) até expressarmos X em função de y:x = f-I (y). b) na lei x = f-I (y) trocamos os nomes das variáveis (x por y e vice-versa), obtendo a lei y ~ f-I (x).

(5 não tem correspondente)

é função

y

Assim, por exemplo, se f :IR obter a inversa de f, temos:

---+

IR é dada por f(x)

B

f(x)

~

Y = 3x + 2

=

x

=

3x + 2 e queremos

~ 3

Permutando as variáveis, temos: x - 2

y~-­

3

portanto,

f-I

é uma função de IR em IR dada por

x - 2 f-I (x) =-3-

(3 tem dois correspondentes)

16. É imediato que f-I de um único x E A.

14-H

é uma função quando todo y E B é o correspondente

Inversas notáveis

Há algumas funções, inversas de funções elementares, cuja importância é grande para o estudo que faremos neste volume:

15-H


a. a função y

c. a fu nção arco-seno:

= v-;

A função f :IR+ ~ IR+ dada pela lei de correspondência y = x 2 é invers í-

....r;..

vel. Sua inversa é f-I: IR+ ----+ IR+ dada por y = Seus gráficos, simétricos em relação à bissetriz do 1Çl quadrante, são os seguintes:

A função

seno (y

contradomínio

[-1, 1],

[

71

71]

- 2' 2"

=

y = arc sen x

sen xl.

quando restrita ao domínio

71

é inversível e sua inversa é a função de .

dada pela lei

71

[ - - -] e ao 2' 2

[-1, 1] em

y = arc sen x. Y

Y

f-I (x)

=

are sen x

A partir da senóide, usando a simetria em relação à bissetriz y = x, constru ímos o gráfico ao lado. x

2

x

x

b. a função logarítmica:

y = log.x

(O

<a

*'

d. a função arco-cosseno: 1)

A função cosseno

*'

A função f: IR ~ IR: dada pela lei y = a , O < a 1, chamada exponencial, é inversível. Sua inversa é f-I: IR: ----+ IR dada por y = log.x, chamada logarítmica. Dependendo do valor de a, os gráficos da logarítmica e da exponencial tomam um dos aspectos seguintes: X

[-1,

11,

dada pela lei

y

contradom ínio [o,

71]

y = arc cos x

(y = cos xl.

quando restrita ao domínio

é inversível e sua inversa é a função de =

f(x)

=.

/

/'

a> 1

16-H

/.

0<.<1

O

-1

(x) = 109

a

I

x

I -1

f

I

I

/'/

-,

I

I x

em

I

-----~--l

YO'X

/"

/

11

y = are cos x 71

/'

[-1,

Analogamente a função anterior, temos o gráfico abaixo.

Y

x

e ao

711

arc cos x.

y = cos x Y

[O,

I I

- - - -- - -

x

-1

O

x

17-H


e. a função arco-tangente:

Y

= arc tg x

A função tangente (y = tg x), quando restrita ao domínio contra-domínio IR, é inversível e sua inversa é a função de IR em pela lei

y = arc tg x.

rr

rr

]-2"' 2"[ e ao rr

rr

)-2' 2 [

dada

Eis o gráfico:

V.

OPERAÇÕES COM FUNÇÕES

17.

Adição

Dadas duas funções f:A ~ 8 e g:A ---> 8, chama-se soma f + g a função h:A ---> 8 definida pela lei h(x) = (f + g)(x) = f(x) + g(x). Por exemplo, sejam as funções de IR em IR: f(x) = eX e g(x) = e-x. Sua soma é a função h(x) = eX + e-x.

!y=arctgx rr

18.

"2

--+---.,..Io""-----f-._x X

Dadas duas funções f:A -----> 8 e g: A ~ 8, chama-se diferença f - g a função h:A -----> 8 definida pela lei h(x) = (f - g)(x) = f(x) - g(x). Como exemplo, sejam as funções de IR em IR: f(x) = sen x e g(x) = log x. Sua diferença é a função h(x) = sen x - log x.

19.

Multiplicação

Dadas as funções f:A ---> 8 e g:A ----> 8, chama-se produto fg a função 8 definida pela lei h(x) = (fg)(x) = f(x) • g(x). Assim, se f(x) = x 2 e g(x) = cos x são funções de IR em IR, seu produto é a função h(x) = x 2 • cos x. h:A

EXERCICIOS H.9

Subtração

Examinar cada uma das funções abaixo a estabelecer quais são inverslveis. Para estas,

20.

~

Quociente

defi ni r a inversa. a) f: {a, b,

c}

---+ {a', b',

d

tal que f = {(a, a'), (b, b'), (c, c')}.

cl h :IR ----+ IR tal que h(x) = 1 - Sx. d) i: IR --.. IR tal que j(xl = x 3 - 2.

' f f(x) h:A -----> B definida pela lei h(x) = (- )(x) =-(-) para x E A =

9

e) j: IR _----+ IR+ tal que j(xl = x 2. •• 1 f) p:IR ---+ IR tal que p(x) =

Assim, se

x'

H.10

Determinar a inversa da função x, f(x) =

f a função { I } x E A g(x) * O .

Dadas as funções f:A ---> 8 e g:A --->8, chama-se quociente

b) g:{1,2,3}----+{4,S,6,7} tal que g(1I=4, g(2)=6 e g(3)=4.

f(x) = x 2

9x

e g(x) = x - 1 são funções de IR em IR, seu quo2

ciente é a função

f:IR ---+IR

h(x) =

~1 definida em IR - {1}. x -

assim definida:

quando x <;;; 1

x+l

- 2 - ' quando 1 <x <;;;3

L

x 2 - 7, quando x >3

H.11

Sejam as funções g(x) =

V;-:-;-.

f: IR ----+ IR

tal que

Determinar a função

f(x) = 2x - 3

H.12

Determinar a inversa da função

f: IR: ---+ IR

H.13

Determinar a inversa da função

f:[-rr, rr)----+ [-1,1)

18-H

e

g: IR -----> IR

tal que

g-IOf-l . dada por

f(x) = log .,;;. .

dada pela lei

f(x) = sen1-.

19-H


Autoditada cria a Análise CAPÍTULO II Gottfried Weilhelm Leibniz nasceu em Leipizig; aOs quinze anos entrou na Universidade, aos dezessete já era bacharel e aoS vinte doutorou-se em Nuremberg. Adquiriu grande conhecimento geral em Teologia, Direito, Filosofia e

Matem~tica

LIMITE

sendo considerado um dos

últimos sábios. Viajou muito representando o governo como diplomata e, numa de suas visitas a Londres, em 1643, tornou-se membro do Royal Society.

Leibniz, por ser autodidata, freqüentemente redescobria teorias e as desenvolvias - em sé" romo é o caso de sua primeira rea I ·Izaçao nes

In

f'Inltas: .

1 "411 = "11 - 3'

1 + 5"

- 7"1 +

expansão da teoria de Gregori. Ao estudar um problema proposto por Huygens, acabou por fazer uma descoberta, -o triângulo harmônico, análogo ao triângulo de Pascal que fascinava Leibniz. Passou então a

estudar as obras de Pascal sobre cilóides e séries infinitas, generalizando um método impor· tante para soma e diferença de funções, tanto racionais como irracionais, algébricas ou transcendentes (palavra que ele criou). Percebendo a grande importância das notações como auxiliar de pensamento, é résponsável por muitas delas como dx e dy para diferenciais em x e y, fydx para integral e foi o primeiro a empregar as expressões "cálculo diferencial", "cálculo integral" e "função". Usou o ponto para multiplicação e escreveu proporção na forma a : b = c : d o que nos sugeriu: para indicar divisão. Ainda criou a notação --.. . . para Ué semelhante a" e : : : : : para "é congruente a Leibniz e Nevvton é que persistiram no uso do sinal =, criado por Recorde,

I.

NOÇÃO DE LIMITE DE UMA FUNÇÃO

21.

Seja a função f{x)

=

(2x

*

~x1~ ~~

- 1) definida para todo x real e x

*1.

Se x 1, podemos dividir o numerador e o denominador por x - 1 obtendo f(x) = 2x + 1. Estudemos os valores da função f quando x assume valores próximos de 1, mas diferentes de 1. Atribuindo a x valores próximos de 1, porém menores que 1, temos:

fl

até hoje usado. Em 1684, sob o tl'tulo de "Um novo método para máximos e mínimos, e também para

tangentes, que não é obstruído por quantidades irracionais", expõe, pela primeira vez, seu cálculo diferencial dando às fórmulas de derivação:

dx

n

n

1

dxy

=

xdy + ydx, d _x y

=

ydx -,xdy e y

x f(x)

° 1

0,5

0,75

0,9

0,99

0,999

2

2,5

2,8

2,98

2,998

-= n x - dx, juntamente com aplicações geométricas.

Sua obra mais famosa é "Acta Eruditorum" (Anotações dos eruditos) onde observou uma diferenciação e integração são operações inversas enunciando o teorema fundamental do cálculo e mostrando que as funções transcendentes são fundamentais em Análise.

Se atribuirmos a x valores próximos de 1, porém maiores que 1, temos:

x

2

1,5

1,25

1,1

1,01

1,001

f(x)

5

4

3,5

3,2

3,02

3,002

Sua teoria de diferenciação, pelas notações

que usou, foi mais aceita do que a Teoria dos Fluxos· de Newton, embora oS dois tivessem desenvolvido a Análise na mesma época. Em 1963, numa carta a L'Hospital, chegou

Gottfried W. Leibniz (1646 - 1716)

a dar antecipação da teoria dos determinantes. Como filósofo pretendia reduzir as discussões lógicas a formas sistemáticas. Otimista ao extremo, sempre acreditou numa futura universali. zação da linguagem, o que foi muito produtivo para a Matemática.

Observemos em ambas as tabelas que, quando x se aproxima cada vez mais de 1, f(x) aproxima-se cada vez mais de 3, isto é, quanto mais ptóximo de 1 estiver x, tanto mais próximo de 3 estará f(x). 21-H


Notemos na primeira tabela que: x = 0,9

== f(x) = 2,8

isto é,

x - 1 = -0,1

==

f(x) - 3 = -0,2

x = 0,99

==

f(x) = 2,98

isto é,

x - 1 = -0,01

==

f(x) - 3 = -0,02

x = 0,999

==

f(x) = 2,998 isto é,

x - 1 = -0,001

==

f(x) - 3 = -0,002

É importante perceber que 8 depende do vemos que: 1~) Ix-ll=0,1==lf(x)-31=02 então se for dado lO = 0,2, tomamos 8 = Ó,l

==

x = 1,01

== f(x) = 3,02

f(x) = 3,2

e afirmamos que

29) Ix-li = 0,01 == If(x) - 31 = 002 então se for dado lO = 0,02, tomamos 8 = á,Ol O < Ix - 11 < 0,01

isto é,

x- l = 0,1

==

isto é,

x - 1 = 0,01

== fI)' - 3 = 0,02

x - 1 = 0,001

==

f(x) - 3 = 0,2

==

e temos:

If(x) - 31 < 0,02

==

3?) Ix-ll=O,OOl If(x) -31=0002 então se for dado lO = 0,002, tomamos 8 = O,OÓl e temos: O < Ix-li < 0,001

x = 1,001 == f(x) = 3,002 isto é,

considerado. Nas duas tabelas

O < Ix - 11 < 0,1 == If(x) - 31 < 0,2

e, a segunda tabela nos mostra que:

x = 1,1

== If(x) -31 < 0,002

(x) - 3 = 0,002 Notemos que, dado lO,

tomamos 8 = ~ . Generalizando, afirmamos que,

portanto, pelas duas tabelas vemos que:

I x - 1\

podemos tomar 8 = ~ tal que

qualquer que seja o valor positivo lO,

0,1 == If(x) - 31 = 0,2 Ix - 11 = 0,01 == If(x) - 31 = 0,02 Ix - 11 = 0,001 == If(x) - 31 = 0,002 =

lO 0<lx-11<8 = 2

Observemos que podemos tornar f(x) tão próximo de 3 quanto desejarmos, bastando para isto tomarmos x suficientemente próximo de 1. Um outro modo de dizermos isto é dizer: podemos tornar o módulo da diferença entre f(x) e 3 tão pequeno quanto desejarmos desde que tomemos o m6dulo da diferença entre x e 1 suficientemente pequeno.

==

If{x) - 31<lO

De fato, 0< Ix-ll<8 =Í

== IX-11<

~ ==2Ix~11<lO

== 12x - 21 < lO == I~ - 31 <

=

lO

=

If(x) _ 31 < lO

flx)

22.

A linguagem utilizada até aqui não é uma linguagem matemática, pois ao dizermos "I f(x) - 31 tão pequeno quanto desejarmos" e "I x - 1\ suficientemente pequeno", não sabemos quantificar o quão pequenas devem ser essas diferenças. A Matemática usa s(mbolos para indicar essas diferenças pequenas. Os s(mbolos usualmente são lO (epsilon) e 8 (delta). Assim, dado um número positivo lO, se desejamos If(x) - 31 menor que lO, devemos tomar Ix-li suficientemente pequeno, isto é, devemos encontrar um número positivo 8, suficientemente pequeno, de tal modo que O < Ix - 11 < 8

==

If(x) - 31 < lO

*

*

A condição O< Ix-li é neste caso equivalente a O Ix-lI, isto é, x 1, porque estamos interessados nos valores de f(x). quando x está próximo de 1, mas não para x = 1.

v Notando que

J

O < Ix - 11 < 8 ~ 1 - ó < x < 1 + 8

e

ex

*1

If(x) - 31 < lO ~ 3 - lO < f(x) < 3 + lO vejamos qual é o significado do lO e 8 no gráfico ao lado. . Para todo x entre 1 - 8 e 1 + 8 e x 1, temos os valores de f(x) entre 3 - lO e 3 + lO.

*

1-8

1+

8

x

22-H 23-H


23. que

o valor considerado o pode assumir.

para

2

Assim se considerarmos

o

I

x+a

==

0< Ix-11<01

24. Seja I um intervalo aberto ao qual pertence o número real a. Seja f uma função definida para x E I - {a}. Dizemos que o limite de f(x). quando x tende a a, é L e escrevemos lim f(x) = L, se para todo € > O, existir o > O

teremos tam b'em

o ="3€

DEFINIÇÃO DE LIMITE

11.

não é único, é simplesmente o maior valor

3

I f(x) - 31

tal que se O < I x - a I < Em símbolos, temos:

<€

o

então

If(x) - LI <

€.

De fato:

€ ==>lx-11<3

0< Ix-11<01

3

== 1l0.J - 31

2€ ===> 12x-21<3 ===> If(x) - 31 <

2€ mas 3

<

==>2Ix-11<

2€

<

2€ 3

== É importante observarmos nesta definição que nada é mencionado sobre o valor da função quando x = a, isto é, não é necessário que a função esteja definida em a. Assim no exemplo anterior, vimos que

==

3

f(xl

I im __(_2,--x_+--;--,-,1)'----"-(x07----:1é...) (x - 1)

2€

3

x+1

==

If(x) - 31 <

mas

o

o

< x < 1 + OI

x = 1.

3 + €

x+a

*' f(a).

f(x) = {2 X + 1 se x*'1 5 se x = 1

3

temos:

e x*,1 e

lim f(xl

Por exemplo, na função

satisfará 1-0<x<1+0

3

y

o

1 - OI

não está definida para

(x - 1)

Pode ocorrer que a função esteja definida em a e

Considerando oI < o, percebemos que o intervalo de extremos 1 - I e 1 + I está contido no intervalo de extremos 1 e 1 + e, portanto, todo x que satisfaz

o

f(x) = (2x + 1) (x - 1)

lim (2x + 1) x+1

lim f(x)

lim (2x + 1)

X+l

x+l

3

*' f( 1)

x*'1

e, conseqüentemente, teremos 3 - € < f(x) < 3 + €

l-li l-O 1i 1+0 1+1i 1

x

o que pode ser confirmado no gráfico ao lado. Desde que, para qualquer valor positivo priado para

o tal que O < Ix - 11 < li

dizemos que o limite de

f(x).

É importante ter sempre em mente no cálculo de lim f(x) que interessa o x+a €,

podemos encontrar um valor apro-

== if(x)

- 31 <

para x tendendo a 1, é 3. lim f(x) = 3

Em símbolos:

comportamento d, f(x) quando x se aproxima de a e não o que ocorre com f quando :< = a. O próxi.no teorema afirma que uma função não pode aproximar-se de dois números diferel'tes qua Ido x se aproxima de a. É o teorema da unicidade do limite de uma fun~ão; ele nos garante que se o limite de uma função existe, então ele é único.

x+!

25-H 24-H


111. UNICIDADE DO LIMITE

EXERCICIOS

H.14 Seja a função f definida por 25.

f(x) = 5x - 2

para todo x real.

lim f(x) = 8 x->2

Se

Teorema encontre um 8 para E = 0,01 Se

e

lim f(x) x->a

lim f(x) x->a

então

tal que

O < Ix - 21<8

LI

=

Iflx) - 81 <0,01

Solução If(x) - 81 < 0,01 <= 115x - 21 - 81 < 0,01 <= 15x - 101 < 0,01 <= 5 • Ix - 21 < 0,01 <= Ix - 21 < 0,002 Se tomarmos 8

Demonstração

'*

=def

lim f(x) = LI

(V E

=

lim f(x) x->a

L2

d~

<===*

38 1

> O IO <

> O,

3 82

I x - a I < ÓI

> O IO<

=

O < Ix - 21 <8 1

I x - a I < Ó2

=

0,

> o,

VE

If(x) - LII

If(x)

- LI I +

'*

-1 . Sabendo que x + 1 definida para todo x real e x calcular 8 de modo que O < Ix + 11 < Ó ==> If(xl + 21 < 0,01.

que a fração

I LI

E

I LI - L 2 I

2

Ix

- ai <

Ó ==> I LI - L2 1 < 2E

vem

- L2 I . r } 2 ,3ó=mIn18 1 ,8 2 10<lx-al<8

~

_.

. .

2

:3 deve

4, quao proxlmo de

estar x para

x+ 3

If(x) - L 2 I

9x 2 - 4 ~

esteja próxima de

4,

com aproximação inferior a O,0001?

H.19 Usando a definição demonstre que fim 13x + 2) x->I

=

5.

Solução

=

Devemos mostrar que, para qualquer

0<lx-11<8

E

> O,

existe Ó

>O

tal que:

=113x+21-51<E

Notemos que

I LI - L 2 I < I LI - L 2 I

que é uma contradição e, portanto, a nossa suposição é falsa. Logo

26-H

flxl ~ -2,

x2 - 1 = ---

. 9x 2 - 4 H.18 Supondo conhecido que 11m2 3X~-2

3 8 = min {8 1 , 8 2 } I O <

Se tomarmos

=

f(x)

+

e, portanto:

E=

H.17 Seja a função

x+-2

X+-l

mas I LI - L2 I .;;

para

=

lim

3ó = min {ÓI, 8 2 } 10< Ix - ai < 8 =

> O,

H.16 Dada a função f tal que f(xl = 5 - 2x, x E IR, determine um número 8 para E = 0,001 de modo que O < Ix + 21 <8 If(x) - 91 < E, sabendo que fim f(xl = 9.

.

e 8 .;; 8 2 e, considerandoeDe

+ If(x) - L2 I < 2 E

V E

If(xl - 81 <0,01

=

IL I -L 2 1 = I(L I -f(x))+(f(x)-L 2 ) I.;; ILI-f(x)1 + If(x)-L 2 1 = = I f(x) - LI I + I f(x) - L2 I 8 = min {ÓI' Ó2}, temos 8 .;; 8 1

=

H.15 Seja f uma função tal que f(x) = 3x + 2, x E IR. Se lim flxl = 5, encontre um 8 para E = 0,01 x->I tal que O < Ix - 11 < 8 If(xl - 51 < 0,01

Escrevendo LI - L2 como LI - f(x) + f(x) - L2 e aplicando a desigualdade triangular (Ia + bl .;; Ia I + Ib I, V a, b E.: IR). temos:

Pondo temos:

If(xl - 81 < 0,01

é verdadeira, porque todo número x que salisfaça a desigualdade O < Ix - 21 < 8 1 satisfará também a desigualdade O < Ix - 21 < 8.

@

If(x) - L2 1 < €)

=

Notemos que qualquer número positivô menor que 0,002 pode ser usado no lugar de 0,002 como sendo o 8 pedido, isto é, se O < 8 1 < 0,002, a afirmação

CD

If(x) - LI I < E)

(V E

=

> O,

0,002, teremos:

O < Ix - 21 < 0,002

Demonstraremos este teorema por redução ao absurdo. Supondo LI L2 , temos:

'x~a

=

LI

1(3x+2)-51<E<=13x-31<E<=3Ix-ll<E<=lx-11<

E

3

27-H


Assim, se escolhermos /l =

f,

De fato

teremos:

O < Ix - 11 <

==>~ ==~

V€>0,3ó=t>0 I O<lx-ll </l==1(3x+2)-51<€

-1<x-l< 1 - €' < x 2 < 1 + €' =

=

De fato se

I

o<lx-ll < / l = 3 " = x-li < 3 ==3Ix-ll

=

13x - 31 <

=

1(3x + 2) - 51 <

<

==

- €' < x2 - 1 < €' == Ix2 - 11 < €' O;;; €

=-

a) lim x 2 = 4 x+2 b) lim (x 2 + 1) = 10

x+-3

aI lim (4x - 1) = 7 x+2 b) lim (4 - 2x) = -2 x+3 cl lim (3x - 2) = -5

c) lim (1 - x2 ) = - 3 x+2 H.23 Prove pela definição de limite que

x+-l

lim _9_= 3 x+2 x + 1

Demonstre usando a definição que lim x 2 = 1. Solução

X+l

Solução

Devemos provar

Devemos provar

V € >

==-

H.22 Prove pela definição de limite que:

H.20 Demonstre usando a definição que:

H.21

==-

1 - ~ ~ < ~ -1 ~ -1 =~<x<~

/l "* Ix - 11 < -1 <x-l <1-

O,

I

3/l > O

O<

Ix - 1\ <

/l

Ix 2 - 11 <

=

V€>O,

3/l>0

I 0<lx-21 <

9 / l = 1 ___ 31 <€ x+ 1

Notemos que 9 _ _ 3 1 < € = _ € < _ 9_ _ 1_ 9 <3+€ 3 <€=3_€< x + 1 x + 1 x+ 1

Notemos que

Ix2 - 1 I <

€ == - € < x 2 - 1 < € =

Suponhamos que o valor de isto é O < Ix - 11 < /l O;;; 1 =

/l

1 - € < x2 < 1 + €

Considerando €' > O tal que €' = € se O < € < 3 ou O < €' < 3 se € ;'3, mos:

que queremos encontrar seja menor ou igual aI,

==>~<Ixl<~ ==~<x<~ ==>~-I<x-l<~-1

~

{Ix - 11 <e

-

Ix -11 <

~

=

==>

=

9 3-€'

==>

3€' Ix-21 < - 3-€' e { 3€' . Ix-21 < - 3 + €'

==> - 1

1-~

Notando que

9 -3>x-2> 3+€' -3

Notando que O < 0<1-

~ < ~ -1 <1, temos: ~ > O onde €'=€ se 0<€<1

para todo € > O, existe /l = 1 ou O < €' < 1 se €;;;. 1, tal que O<

28-H

3-€0;;;3-€'<~1 x +

<3+€'0;;;3+€ =0<3_€,<_9_ <3+€,,,* x + 1 1 x+l 1 9 9 = = > 3 - € , > - 9 - > 3 + € , ==~>x+l>3+€'

Ix - 11 < 1 ==> -1 < x-I < 1 ="* O < x < 2

e sendo €' > O tal que se O < € < 1 então €' = € ou se € ;;;, 1 então 0<€'<1, temos 1 - € O;;; 1 - €' < x 2 < 1 + €' O;;; 1 + € == O < 1 - €' < x2 < 1 + €'

Ix - 11 <

/l

== Ix 2 - 1 I <

te-

3€' =""3+€' >

3€' 3 + €' <

3€'

3"=€"

3€' =="3=€' >

temos para todo € >

-3€' x-2> 3+€

O,

existe

==

/l =

O onde €' = € se O < € < 3 ou O < €' < 3 se € ;;;, 3, tal que

0<IX-21</l==I~1 x +

-31<€

29-H


De fato:

IV. PROPRIEDADES DO LIMITE DE UMA FUNÇÃO

O<lx-21<o =lx-21< ==>

-3 E'

3E' 3

+

==>

E'

3E'

3E'

3+€' < x - 2 < 3+"€' < 3-=€'

-3E' 3E' =3+E' <x-2< 3-€' = =_9_

3+E'

<

x+1

<

9

3-E'

=3_E,<_9_ < 3+€' x + 1

I ~1 x +

=

31

-

< €'

X+I

~6_=

2

x + 2

~4_ x+1 3 - x

b) 11m

c) Iim _ _2_

= 2

x+3 2x - 5

=

9 9 3+€' -3<x-2< 3_€,-3 =

1 x+1 1 3+€' < ~9- <"3=€'

=_E,<_9_ -3<€' x + 1

26.

= ==>

No parágrafo anterior vimos que, para provarmos lim f(x) = L, devemos exibir x+a um o > O para um dado E > O. Considerando que freqüentemente uma função é constru ída a partir de funções mais simples; por exemplo uma' função polinomial f é uma soma finita de funções do tipo fi(x) = aixi onde ai E IR e i E IN, isto é:

==>

f(x)

= ao

+ alx + a2x2 + ... + anx n

n

=L

2

n

ai xi

i=o

.;;;; E.

H.24 Prove pela definição de limite Que: a) lim

=

=

L

fj(x)

i=o

Se as funções fi têm limites para x tendendo a a, então uma combinação conveniente nos fornece o limite de f quando x tende a a. A fim de que não tenhamos que voltar repetidamente à definição de limite para provarmos lim f(x) = L, vamos apresentar as propriedades algébricas do limite x+a de uma função. No que segue estamos supondo que a é elemento de um intervalo aberto I, e que em I - {a} estão definidas as funções f, g, ... "envolvidas" na propriedade. '-

H.25 Prove pela definição de limite Que:

IIm~=4

X+2

27.

1~ Propriedade

então

"Se f é a função definida por f(x) lim c = c". x+a

x - 1

H.26 Prove pela definição de limite Que: 11m x 3 = 1 X+I

=

c onde c E IR, para todo x real,

Demonstração Devemos provar:

'ri E> O, 3 o > O I O < Ix - ai < o

=

If(x) - cl < E

~ sempre verdadeiro, pois

If(x) - cl

28.

Ic - cl

=

O< E

2~ Propriedade

Se c E IR e Iim f(x) x.a

3O-H

=

=

L então lim [c • f(x)] = c • lim f(x) = c • L. x.a x+a

31-H


Demonstração

Demonstração

Devemos provar Devemos considerar dois casos:

'ri € > O, 3 & > O I O < I x - aI < & .u v

1C? caso c = O Se c = O então c • f(x) = O • f(x) = O Pela 1~ propriedade temos

3 &2

x+a

>O

Considerando

21? caso c

=1=

T emos:

3&1>00<lx-al<&1

lim [c • f(x) 1 = Iim O = O = c • L

X-1'-a

€ € > O, consl'd eremos 2"'

=== I (f + g)(x) - (L + M) I < €

O < Ix - ai

< &2

===-

If(x) -

LI <

== Ig(x) - M I <

li = min {& I, &2},

€ "2 €

2

e portanto &..;; &I

e &..;; &2, vem €

O

& =min{li l ,&2} I o<lx-al<& ==lf(x)-LI+lg(x)-MI<2+2"=€

Devemos provar

'rI€>O, 3&>01 O<lx-al<&

===lc.f(x)-c.LI<€

Mas pela desigualdade triangular, temos:

Temos por hipótese

If(x) - LI + Ig(x) - MI..;; If(x) + g(x) - (L + M)\ = I(f + g)(x) - (L+M)I

lim f(x) = L x+a

então

isto é,

3 li = min {&I, &2} I O < Ix - ai < &

'rI€ > O,

3 &1 > O I O < Ix - ai < &1 €

Então 'ri € > O, considerando Icl

3&

>O \

O < Ix - ai

<

&

(L+M)I < €

==> If(x) - LI < €

30. Esta propriedade pode ser estendida para uma soma de um número finito de funções, isto é,

temos:

Se lim f;(x) = Li, i E N e 1";; i ..;; n,

=== If(x) -

LI <

x+a

Icl

então

isto é 3&>010<lx-al<&

== lU + g)(x) -

== Icl • If(x) - LI <

lim (

~ • Icl = €

X+a

t

fi)(x) =

i=l

t

Li·

i=l

ou seja

3&

>OI

O < Ix - ai

<&

Demonstração:

== Ic.f(x)-c.LI <€

Faremos a demonstração por indução finita 1~ PARTE

29.

Para

3~ Propriedade

n = 1 é verdadeira, pois I

Se

lim f(x) = L e lim g(x) = M x+a

32-H

x+a

então

lim (f + g)(x) x+a

L + M.

lim

(L

x+a

i=l

I

fi(X)) = lim fi (x) = LI = x+a

Li=l

Li

33-H


2~

Lema 2

PARTE Supondo que a propriedade seja verdadeira para lim

(LP

x+a

i=l

fi)(x) ~

LP

p+1

(L

x+a

i=l

lim f(x)

isto é,

L

=

p + 1,

isto é

VE > O,

Considerando que

i'='l

X-:l>-a

1=1

lim [(

L

x+a

i=l

fi) + f p + l

lim

(L

x+a

i=l

]

P+I

fi) (x) + lim f p + I (x)

=

x+a

If(x) 1 - I LI 3 o[ > O

P+I

L

Li + L p + I =

j=l

L

Se

lim f(x) x"a

e

lim g(x) x"a

=

M

então

lim (f - g)(x) x"a

L - M

lim [f(x) - g(x)] = lim [f(x) + (-1) • g(x)] X~

x+a

=

lim f(x) - lim g(x) x+a

=

=

L,

e tomando

Ix - a I < o [

I O < Ix - ai < o I

lim g(x) = O, x"a

=

====> If(x)

Sendo

L

se, e somente se,

lim (f(x) - L)

=

O

== 33.

x+a

isto é,

(V E > O,

o> O 10< Ix - ai < o

=

If(x) - L I < E)

=

=('VE>O, 30>Oi O<lx-al<o==lf(x)-L-OI <E)= lim (I(xl - LI x+a

34-H

=lg(x)I<E

temos

0<lx-al<02

=lg(x)l<

I 0< ix - ai < 02

o = min {OI, 02},

=

=

O

E 1+ iLI

0

(1 + ILI) • Ig(x)1 < E

e portanto

o.;;; OI

e o .;;; 02,

I O < Ix - ai < o

If(x) • g(x)1 = lf(x)1 • Ig(x)1 < (1 + ILI)

temos

=

Ig(x)1 < E

0

5~ Propriedade

Se

Prova

=

I < 1 + IL I

(j)

x+a

=

I f(x) - LI < E

If(x) - L I < 1

O<lx-al<o

Lema 1

lim f(x) = L x+a

=

Ix - a I < o

.;;; If(x) - L I e portanto:

1 + I LI

3 02 > O

L - M

x+a

Antes de passarmos para a próxima propriedade vamos considerar dois lemas.

lim fix)

I < E

isto é,

V E > O, 3 o = min {OI, 02} > O

32.

====> If(x) • g(x)

isto é,

X~

lim f(x) + lim [(-1) • g(x)] x+a

O<

30 2>01

x+a

O.

vem

Considerando que

Demonstração lim (f - g)(x)

Iim f(x) x"a

VE>O, 30>01 L

=

=

Li

i=l

4~ Propriedade

31.

E

3 01

(x) mas

P

=

= 1, > O I

e fazendo =

O então lim (f. g)(x) x+a

V E > O, 3 o > O I O <

P

fi)(x)

=

3 o > O I O < Ix - ai < o

Li

P+I

(L

lim g(x) x+a

Devemos provar que n

De fato lim

e

Prova

Lj

p+1

fi (x))=

L

x+a

i=l·

provemos que é verdadeira para lim

n = p,

lim f(x) = L x+a

e 11m g(x) = M x+a

então

lim (f· g)(x)

LM

x+a

Demonstração Notemos que (f· g)(x)

=

f(x) • g(x) = f(x) • g(x) - L • g(x) + L • g(x) - LM + LM

35-H


isto é,

Prova

(f· g)(x) ~ [f(x) - L] . g(x) + L . [g(x) - M] + LM

De

1) lim f(xl ~ L <=> lim (f(xl - LI ~ O, x->a ~

L

vem:

"tE> O, 3 8

>O I

Tomando

2

x->a

2) 11m g(x)

M <=> lim (g(x) - M)

x-+a

~

O,

E

ILI

=

x+a

===> lim [(f(x) - L) • g(x)] ~ O

x+a

Ix - ai

<8

===> If(x) - LI<E

temos

38 > O I O < Ix - ai < 8

=> lim (f· g)(x) ~ lim Uf(x) - L] • g(x) + L • [g(x) - M] + LM} x+a

ILI

"2 < f(x)

ILI

<2

- L

x+a

i) se L> O então

x+a ~

ii) se L

<O

então

34.

Esta propriedade pode ser estendida para um produto de um número finito de funções, isto é, se n

n

então

x+a

L +

2ILI

===>

ILI

T .

L

3L

O < 2"" < f(x) < -2-

L • O + LM ~ LM

x+a

lim fi(x) ~ Li

ILI

T < f(x) <

=> L -

+ lim {L • [g(x) - MJ} + lim LM ~

O + L . lim [g(x) - M] + LM

===> If(x) - LI <

Então são poss(veis dois casos:

x+a

~ lim {[f(x) - L] . g(x)} x+a

lim

fi fj)(x)

fi

x-:l>-a

i= I

i=l

L 0# O,

então, para

3L

Considerando

I

< f(x) <"2 < O

O<

temos N =

L

2

Li (*)

~

I

~I

I > O,

< If(x) I <

I 3 L I. 2

temos:

1,;;;; i ,;;;; n O < Ix - ai < 8

A demonstração por indução finita fica como exercicio.

===> if(x)1 > N

6~ Propriedade

35.

Se

lim f(x)

=

L

então

Iim (f)n (x)

x+a

fi

<

x+a

temos

i E IN,

O

x+a

3) lim (f(x) - L) ~ O e lim g(x) ~ M

~

~

lim f(x) x+a

Considerando que

=

=

L n,

n E N*, Lema 4

x+a

(Trata-se do caso particular da propriedade vista no parágrafo 34, fazendo f 2 ~ . . . = f n = f).

36.

Considerando que

Lema 3

lim g(x)

=

M 0# O,

pelo lema 3, temos:

x+a

lim f(x)

L 0# O então existem

=

8

e

N

positivos tais que

x+a

38 1 ,N>01 0<lx-al<8 1

O < Ix - ai < 8

(*) O s{mbolo fi f. (lê-se: produt6ria dos fatores f," com i EN, 1 ,;;;; i i =1

1

===> If(x)i > N.

n I

n

11 f. = fi • f2 • f3 ••• f . i =1 I n

36-H

x->a

Prova

Antes da próxima propriedade, vejamos mais dois lemas

Se

Iim g(x) = M 0# O então

Se

===> - - < Ig(x)1

<n

significa:

De

===>lg(x)I>N

=

1

N

lim g(x)

~

M,

vem

x+a

"tE>O, 38>01 0<lx-al<8

===>Ig(x)-MI<E

37-H


Considerando

E'

Se

I M I • N, temos:

lim f(x) ~ L

e lim g(x)

x+a

LI'

M

então:

x+a

lim c = c x->a

o~

Sendo

min {OI, 02J,

'tE> O, 3 0= min {Oj, 02} ==>

1~1_

~I

g(x)

M

L2 •

vem:

Ig(x) - M I

TgTx)nMT

lim Ic.f(x))= c ·Iim f(x) = c· L x+a

O<lx-al<o

==>

1

I g(x) - MI . ig(x)1 •

1

lim

TMT <

x+a

l(f

+ g) (x)] = lim f(x) + lim g(x) ~ L + M

x+a

E·IMI·N N. IMI

x+a

x~

lim I(f - g) (x)] = lim f(x) - lim g(x) = L - M x+a

X~

=E

Ls.

L6 •

x+a

lim [(f· g) (x)] = lim f(x) • lim g(x) = L· M x+a

x+a

x+a

lim [(f)n (x)] ~ ['im f(x)]n = Ln x->a

x->a

lim f(x)

37.

L7 •

7~ Propriedade

Se

lim f(x) = L e lim g(x)

M =1= O então

x->a

x->a

L f lim (-) (x) - M 9 x->a

La·

f lim [( - ) (x)] x->a 9 lim x->a

iffW

L

x->a

(M =1= O)

lim g(x) - M x->a

~VL (se

lim f(x)

n

x->a

n E N*

n é ímpar e

e L L .;; O)

>O

ou se

Demonstração

V.

Pelo lema 4 temos: lim g(x)

~

M =1= O

=

x->a

1 . 1 11m --- - M x->a g(x)

LIMITE DE UMA FUNÇÃO POLINOMIAL

Uma das conseqüências das propriedades L é a regra para obter o limite de uma função polinomial.

então f lim (-) (x) x->a

9

lim [f(x) x->a

_1 ) = L· -1 g(x) M

L M

39.

Teorema

o limite de uma função polinomial f(x) ~ ao + ai x + a2 x

2

+ ... + anx

n

n

=

L

aj E IR, para x tenden-

1=0

38.

do para a, é igual ao valor numérico de f(x) para x ~ a, Antes de provarmos esta proposição, provemos que lim x

8~ Propriedade

Se

lim f(x) = L x+a

<O

então

lim ~ f(x) =

x+a

VL

com

L

>O

e

n E N' ou

e n é ímpar, A demonstração deste teorema será feita oportunamente, mas iremos aplicá-lo quando for necessário. Por uma questão de simplicidade indicaremos as propriedades de limites, como sendo as propriedades L e vamos fazer rápido sumário dessas propriedades. L

38-H

a,

x->a

É trivialmente verdadeira pois, dado O < Ix - ai < € ==> Ix - ai < E. n

Provemos agora que

lim I

L

x+a

i=o

E> O,

(ajx n - i )] ~

basta tomarmos

n

L

o~ E

e temos

(ai an - i ).

i=o

39-H


De fato, por apl icações sucessivas das propriedades, temos

~: [~ =

t

[ai (Iim x)n-i] = x~a

i=o

=

(aiXn-il]

t

x 3 + 2x 2 - 3x + 2 x 2 + 4x + 3

d)

~ [~: (aiXn-i~ = ~ [ai(~: xn-i~

x 3 + 2x 2 - 3x· + 2 x2 + 4x + 3

(La)

(x 3 + 2x 2 - 3x + 2) (x 2 + 4x - 3)

i (ai an - ) H.28 Calcule os seguintes limites, especificando em cada passagem a propriedade ou o teorema

j:.o

utilizado.

a) lim 14x 2 - 7x + 5) X+l b) lim Ix 3 - 2x 2 - 4x + 3)

t)

2X -5r lim (3X22 +3x+4 _x x+2 (x 3 -3x2 _2X_5)2 2x 2 - 9x + 2 x+4

g) lim

x+-l

3x + 2

cl lim x2 _ 6x + 5 x+2

h) Iim

x+-}

3x 2 - 5x + 4 2x + 1 X+-l

d) 11m EXERCICIOS

H.27 Calcule os seguintes limites, especificando em cada passagem a propriedade ou o teorema utilizado.

a) lim (3x 2 - 5x + 2) x+2 x 2 + 2x - 3 b) lim 4x - 3 x+-l

r

X+2

3

d) lim x+-2

lim x+-2

x 2 + 2x - 3 x+-3 5 - 3x

e) lim

H.29 Calcular lim

2 (2X - x + 1 3x - 2

c) lim x+1

i)

j)

)2X 2 +3x-4 5x - 4

f

3x 3 - 5x 2 - x + 2 4x + 3

-/2x 2 + 3x + 2 6 - 4x x+2

lim

x 2 -4 x - x

~2

Solução Temos lim Ix 2 - 4) ~ O e 11m (x 2 -2x) ~ O e nada podemos concluir ainda sobre o li-

x 3 + 2x 2 - 3x + 2 x 2 + 4x + 3

x+2 mite procurado.

x+2

Solução

Os polinômios (x 2 - 4) e (x 2 - 2x) anulam-se para x ~ 2, portanto, pelo teorema de D'Alembert, são divisíveis por x - 2, isto é:

a) pelo teorema da função polinomial IT), vem: lim (3x 2 - 5x + 2) ~ 3 • 2 2 - 5 • 2 + 2 = 4 X+2

x2 - 4 (x + 2)(x - 2) x 2 - 2x ~ xIx - 2)

b)

~2x - 3

11m

4x - 3

x+-l

Iim (x 2 + 2x - 3) X+-1 -cc---,-~-Iim (4x - 3) X+-l

c) Iim x+1 (

2x2 _ x + 1) 2 (L6) = 3x - 2

lim (2x 2 - x

x+1 lim (3x - 2) ( x+1

4O-H

(L7)

(.

11m x+1

2X 2 -X+1\2 (L7) 3x - 2 J

x+2 x

Considerando que no cálculo do limite de uma função, quando x tende a a, interessa o comportamento da função quando x se aproxima de a e não o que ocorre com a função quando x = a, concluímos: x2 - 4 Iim -2--~ lim x + 2 ~ 2. x+2 x - 2x x+2 x

H.30 Calcular os limites:

x2 - 1 a) Iim - - X+l x - 1

4 - x2

b) lim - - x+-2 2 + x

41-H


. 4x 2 - 9 cl 11m . - - - 3 2x - 3

x3 - 1 g) 11m ~ x+1

x+2"

x 2 - 4x + 3 di lim 2 x+3 x - x - 6 e) Ilm

l

f)

Temos

8 + x3 2 x+-2 4 - x

i)

lim (2x 3 + x 2 - 4x + 11 ~ O e x+l

h) lim

2x 2 + 5x - 3 2x 2 - 5x + 2

x+2"

Solução

Os polinômios

lim (x 3 - 3x 2 + 5x - 31 ~ O. X+l

(2x 3 + x 2 - 4x + 11

Ix 3 - 3x 2 + 5x - 31

e

portanto, pelo teorema de D'Alembert, são divisfveis por

anulam-se para x - 1,

(x - 1),

isto é,

x - 1

é

um fator comum em 12x 3 + x 2 - 4x + 1) e (x 3 - 3x 2 + 5x - 31. Efetuando as divisões de (2x 3 + x 2 - 4x + 11 e (x 3 - 3x 2 + 5x - 3) por Ix - 11,

x 4 - 16

11m x+2 ~

obtemos:

6x 2 + 11 x + 3 lim 3 2x 2 -=5~ x+-T

(x - 11 • (2x 2 + 3x - 11 (x - 1) • Ix 2 - 2x + 3)

2x 3 + x 2 - 4x + 1 x 3 - 3x 2"+ 5x - 3

2x 2 + 3x - 1 x 2 -2x+3

Então

H.31

2x 3 + x 2 - 4x + 1 lim x3 _ 3x2 + 5x - 3 x+1

Seja a função f definida por

f(xl =

I

Calcular

x2 - 3x + 2 X - 1

se

x

3

se

X

=1

H.35

lim flxl. X+I

2x 2 + 3x - 1 lim x 2 _ 2x + 3 ~ 2 x+1

Calcular os limites: x 3 + 3x 2 - x - 3 x3 - x2 + 2 X+-l

cl lim x+l

x 3 - 6x - 9 x 3 - 8x - 3

d) lim x+2

a) lim

Solução

b) lim x+3

Como no cálculo do limite de uma função, quando x tende a a, interessa o comportamento da função quando x se aproxima de a e não o que ocorre com a função quando

x 3 - 3x 2 + 6x - 4 x 3 - 4x 2 + 8x - 5 x 4 - 10x + 4 x 3 _ 2x 2

x = a, temos: x 2 - 3x + 2 (x - l)1x - 2) lim flx) ~ lim - ...- - - ~ 1im 11m (x - 2) x+I X+I X - 1 x+1 ~ x+1

.---rx--=-lT -

-1

H.36

Calcular

I im x+l

3x 3 - 4x 2 - x + 2 2x 3 - 3x 2 + 1

Solução H.32

Seja a função f

definida por Temos

flx)

~

rX2 - 3x - 2 x - 2 3

Calcular

se

x

i=

se

x

~

2

3x 3 - 4x 2 - x + 2 (x - 1) 13x2 - x - 21 3x 2 - x - 2 ~ (x - 11 12x2 - x - 1) ~ 2x 2 - x - 1 2x 3 - 3x 2 + 1

então . 3x 3 - 4x 2 - x + 2 lim 2x3 _ 3x2 + 1 x+1

+ 9x + 9 x + 3

{ 3 se

Mostre que

x+l

Efetuando as divisões de 3x 3 - 4x 2 .. x + 2 e 2x 3 - 3x 2 + 1 por x - 1, temos:

H.33 Seja a função

flx) ~

lim 12x 3 - 3x 2 + 1) ~ O.

x+l

2

lim flxl. x+2

2x2

lim (3x 3 - 4x 2 - x + 21 ~ O e

x

-3

3x 2 - x - 2 lim 2x2 -x-l x+1

mas lim 13x 2 - x - 2) ~ O e lim 12x 2 - x - 1) ~ O x-H x+l

lim t(x) ~ 3. x+-3

então H.34

Calcular

42-H

3

2

lim 2x + x - 4x + 1 x+! -;;3 - 3x 2 + 5x -'3

.

. 3x 2 - x - 2 lim ....- - x+1 2x 2 - x - 1

. Ix - 1) 13x + 2) I Im x+l Ix - 11 (2x + 11

lim 3x+2 =~ x+l 2x + 1 3

43-H


H.37 Calcular os limites:

H.41

x 3 - 3x + 2 a) lim 4 x+1 x - 4x + 3

x 4 + 2x 3 - 5x 2 - 12x - 4 4 + 7x 3 + 2x 2 - 12x - 8 2x x+-2

x2 - 4

v'X+2-~

..j x 2

e) lim x+1

- ..j

x 2 + 3x - 3 - 3x + 3 2 x - 3x + 2

xm _ 1

x 2 - a2

lim--'----~ X'+'8 x-a

d) lim -n--l

x-H x n

x2

n

+x

x~

fi

x - 1

lim

x+a

lim~

x+2

x - a

xm - a m

n

c)lim~

~-2

H.42 Calcular

e)lim~

b) lim -3--3

X+l

x 2 -9

d) lim x+2

H.38 Calcular os limites:

x+_aa

2-~

.;;:;:J- 2 c) 11m 2 x+1 x - 3x + 2

d) lim

a2 -

x2 - 1

b) lim x+3

x 4 _ x 3 - x 2 + 5x + 4 c) lim x 3 + 4x 2 + 5x + 2 x+-l

a)

3-~

a) lim x+1

x 4 + 4x 3 + x 2 - 12x - 12 2x 3 + 7x 2 + 4x - 4

b) lim x+-2

Calcular os limites:

4x + 1 - 3

Solução

xn _ an

Como 11m

(y'3;::2 - 2)

x+2

H.39 Calcular

x- 3

~

Solução Como lim x+3

L7

(~- 2

) = O e lim (x - 31 = O. não podemos aplicar a propriedade

"conjugado" do numerador, temos: (x - 3)

(x - 3)

N 1+x

+ 2)

a, então

y'1+"; -2 lim x+3

x-3

lim x+3

bl lim x+o cl lim X+1

44-H

...,r;- 1

+2

x

x-I

~

- 2

(~+21

3(~ + 3)

4(~+2)

3(V4x""+1 +3) 9 =Iim x+2 V 4x + 1 - 3 x+2 4(~ + 2) 8 lim.~

- 2

Y2x'+1 - 3

a) 11m • r--;:::

.;;: x+4 yx-2-V2

..j

1-~

y';+J

= 4(x - 2)(~+2)

(~+3)

H.43 Calcular os limites:

4

1 - 2x - x 2 - 1 d) lim - - - - - x x+o

x-I

(~-3)'(~+3)

1

-v'1+X

H.40 Calcular os limites: ai lim X+1

(~-2)' (~+2)

-2

~-3

e então

(~-2)(~ +2) (x - 3) (VT+i< + 2

x - 3

x+2

3(x - 2)(-,/4x+1 + 31

x+-3

(limite do quociente). Multiplicando o numerador e o denominador da fração pelo

~-2

= O e lim (~ - 3) = O, multiplicamos o numerador e o

denominador pelo "conjugado" do numerador e também pelo "conjugado" do denominador.

y'1+";-2 I.Im --"---'---~-=x+3

.

e) lim X+O

b) I i m

4-~

• r;;-;;---cc X+6 2 -yl0-x

cl lim x+o di lim x+2

Vh+4 -y';+4 .~

vx+l-l

..j x 2 + x - 2 - ..j x2 -

x+2

v'X+2-2

~-~ x

...[2;. - ...r;+1

f) lim - - - - - : - - x+1 x-I

H.44 Calcular os limites:

..j 2x 2

- 3x + 2 - 2 ' x+2 y 3x' - 5x - 1 - 1

a) lim ./

b) lim x+ -1

..j 3x 2 + 4x + 2 - 1 V x 2 + 3x + 6 - 2 45-H


x - 2

Solução

lim 3r=====~-­ X->2 3x - 5 - 1

H.45 Calcular

'\I

Notemos que

Solução lim (x - 2) = O e lim

Notemos

X+2

X+2

Lembrando da identidade

[(~)2

(~ 3x - 5 -

- 5 - 1

1)

+ </3x - 5

3(x . 2)

3

lim x->64

..v-; + 1 x +1

..r;:-

V

~:':64

{fX =

y,

= y2

..

= lim

y->2

y2 + 2y + 4 Y + 2

c) lim x+l

= lim y y->2

=3

~-1 ~- 1 ~-1

d) lim .3r:---x->oVl+x-l

Calcular.,J1S-.~' es: ""

H.51

1

a) lim _

x+a

H.47 Calcular os limites:

b) 11m X+-2

(.lf";)2

x - x2

b)lim ) ~ x->-l V 2x + 3 - 1

a) lim x->o 1+~

~ =

1

</8 - 2x + x 2 - 2 c) lim - - - - - - ; : ; - - x+O

l-tE

e

~ = 6~ = ~ = 2

b) lim .3rx+l V x - 1

H.46 Calcular os limites:

+

(..v-; - 4) = O

H,50 Calcular os limites:

X+-l

x

= y3

y3 - 8 lim - - y->2 y2 - 4

VX - 4

a) lim

x

lim x->64

...r;8 .3r

e então

x+o

e lim x->64

temos:

[(~ 3x - 5)2 + ~ 3x - 5 + 1] (~)2+~+1

v--;: = (~)3

e notando que

+ 1].

(X-2)[(~)2+~+11

~-1 a) 11m _ _-::_

=O

H.39 e H.45: Vamos, entretanto, apresentar um novo processo. Fazendo temos

(x-2)[(~5)2 +~+11

x-2

(v--;:- 8)

Poderfamos empregar no cálculo deste limite os processos mencionados nos exercfcios - 1) = O.

a3 - b 3 = (a - blla 2 + ab t b 2 L vamos multiplicar o nu-

merador e o denominador por

~ 3x

(~

lim x->64

xV--;:- a

a 'J"'iX - V_r~

V;-.1

</3x 2 - 7x + 1 + 1

c) lim x->2 </2x2 - 5x + 3 - 1

~-2 1+~

b) 11m --....,-

x->l

c) lim x->l

x - 1

~-~

d) I im _...,.,...---,__

x+a

x - a

~-

rv;-

e) lim - - - - - x->a nr- nrV x - va

~-1 ~-1

H.48 Calcular os limites:

~-3

a) lim 3 x->l ~ + 1

c) 11m

V 3><3

- 5x + 6 - 2

x->l </ x 2 - 3x + 1 + 1

~-2

b) lim _ r--;x->2vx-1-1

...[;-8 H.49 Calcular

46-H

lim .3rx->64 V x - 4

47-H


VI.

40.

LIMITES LATERAIS

41.

Definição

f(x).

Seja f uma função definida em um intervalo aberto la, b[. O limite de quando x se aproxima de a pela direita, será L e escrevemos

Lembremos que ao considerarmos lim f(x), estávamos interessados no com-

Iim

x+a

portamento da função nos valores próximos de a, isto é, nos valores de x pertencentes a um intervalo aberto contendo a mas diferente,s de a e portanto, nos valores desse intervalo que são maiores ou menores que a. Entretanto, o comportamento em algumas funções, Quando x está próximo de a, mas assume valores menores que a, é diferente do comportamento da mesma função, quando x está próximo de a, mas assume valores maiores que a.

f(x)

=

L

X-..a+

. ' v<>0, talquese O<x-a<ô então lf(x)-L1<E. se, para todo E > O, eXistir Em símbolos, temos:

Assim, por exemplo, na função f(x) =

4-X se x<l 2 se x = 1 { x - 2 se x > 1

atribuindo a x valores próximos de 1, porém menores que 1, (à esquerda de 1), temos:

x

O

0,5

0,75

0,9

0,99

0,999

f(x)

4

3,5

3,25

3,1

3,01

3,001

42.

Definição

Seja f uma função definida em um intervalo aberto ]b, ar. O limite de f(x), quando x se aproxima de a pela esquerda, será L e escrevemos lim

f(x)

=

L

x-..a-

se, para todo E> O, existir ô > O, tal que se -li < x - a < O então !f(x) - L I < E. Em símbolos, temos:

e atribuindo a x valores próximos de 1, porém maiores que 1, (à direita' e 1). temos:

x

2

1,5

1,25

1,1

1,01

1,001

f(x) .

O

-0,5

-0,75

-0,9

-0,99

-0,999

43. As propriedades de limites (propriedades L) e o teorema do limite da funç~o polinomial são válidos se substituirmos " x -+ a " por ~~ x -+ a+" ou por "x ..... a ".

Observamos que, se está próximo de 1, à esquerda de 1, então os valores da função estão próximos de 3, e se x está próximo de 1, à direita, então os valores da função estão próximos de -1. Em casos como este, onde supomos x assumindo valores próximos de 1, mas somente a esquerda ou somente a direita de 1, consideramos os limites laterais pela esquerda ou pela direita de 1, que definiremos a seguir.

48-H

49-H


temos:

EXERCICIOS

lim f(x)

lim

X'+l+

X+l.+

(3 - x)

Nos exercícios H.52 a H.57, tl dada uma função f. Calcule os limites indicados, se existirem; se 0(5) limite(s) não existiriem) especificar a razão.

=2

e lim f(x) = Iim )(+1-

(x 2 - 4) = -3

{ 3x - 2 se x> 1 2 se x = 1 4x + 1 se x < 1 b) 11m t(x) a) lim t(x)

H.52 f(x)

x+l-

Como os limítes laterais são diferentes, dizemos que lim t(x) não existe. A x"l justificação da não existência de um limite devido ao fato de os limites laterais serem diferentes é dada no teorema que segue.

=

x+l-

X+l+

{ 3 - 2x se

H.53 f(x)

a) lim

Seja I um intervalo aberto contendo a e seja f uma função definida para x E I - {a}. Temos lim f(x) = L se, e somente se, existirem lim f(x) e x+a

- 5 < x - a < O ou

( '9'e>O, 35 >0 10< Ix -ai <5 =-It(x) -LI<e)

o,

OU

0<x-a<5 =* It(x)-L/<e)-

3 5 > O I - 5 < x e- a < O

'9' e> O, 3 5 > O I O < x - a < 5

lim

f(x)

L

-

e { lim

f(x)

L

x+a+

== lf(x) ==-

LI

<e]

If(x) - LI < e

H.56 t(x)

c) lim t(x) X"2

=

se x >3 b) lim f(x) )(+3-

f(x)

a) lim x+3+

_

21x2 - 3x - 1 H.57 f(x)

=

{ _x 2 + 6x - 7

a) lim

f(x)

se x

H.58 Dada a função

lim

f(x).

c) lim f(x) x"3

<2

se x = 2 se x> 2 b) lim f(x)

x"r

X+2+

x+O-

50-H

f(x)

lim

{ x2 - 3x + 2 se x";;3 <=o-

8 - 2x

('9'e>0,35>01-5<x-a<0

c) lim t(x) X"3

x+2-

x+2+

lim f(x). L x"a

x.....

b)

f(x)

a) lim

'9' e>

b)

2 se x < 2 { 1- x se x ~ 2 O x - 1 se x> 2

=

O< x - a < 5

temos

lim f(x) x+3 -

x+-I

5x se x<3

4 -

H.56 t(x)

O < Ix - ai < 5 -

c) lim f(x)

=

a) lim f(x) x.. 3'"

Notando que

lim f(x) )(+-1-

{ 2x - 5 se x ;;'3

H.54 f(x)

x+a+

e forem ambos iguais a L.

_I

b)

f(x)

Demonstração

-

se x <-1

x+-l+

Teorema

lim f(x) x+a-

x;;' -1

=

4 - x

44.

c) lim t(x) )(+1

c)

Iim f(x) X"2

Ixl para todo x E ' caIcu Ie I'Im definida por f(x) = -x IR.

x+o+

f() x

e

Existe lim f(x)7 x+O

51-H


H.65 Dada. a. função máximo inteiro (*), denotada por f(x) = [xl para todo x E iR, calcule se eXistir.

Solução Lembrando que

I-:

Ixl =

temos lim

a) Iim

f(x)

~

x?'-o-

se

x<o

b) Iim [xl X"l-

g) lim (x + [xl) X+l+

c) lim [xl X"l

h) lim

d) lim

j)

Ixl

lim ~= lim x+o+ x x+O+

x

1

~

. Ixl 10m x+o- x

lim

-x = lim

x+o- x

=

a) lim

~) f(x

f(x)

b) 11m

2+

definida em IR -

r:

f(x)

b) lim

c) lim f(x) x"l { } 2.

f(xl

x+2+

x 3 - 4x2 + x +6

= 12x2 _ 9x + 101

3x+a se x>-2

a) lim f(x) X+2+

definida em IR -

para que exista

Iim f(x) x+-2

f(x) =

3x

2

- 5x - 2 x - 2 3 - ax - x 2 a E IR

se x < 2 se x ;;'2

para que exista

Iim f(x) x"2

h}, c) Iim f(x)

b) lim f(x) x+2-

bl lim f(x) x+2-

a E IR

c) Iim f(x) X"2

f(x)

definida em IR -

{

determine

x+2-

x 3 - 6x2 + 11 x - 6 Ix _ 2\

e) lim

52-H

definida em IR -

X+2+

H.63 f(x) =

{

H.68 Dada a função f definida por

b) 11m f(x) x"l-

f(x)

a) Iim

4x + 3 se x"';-2 f(x) =

definida em IR - {,).

13x2 - 5x - 21 x _ 2

f(x)

{~3}.

H.67 Dada a função f definida por

determine

x+l+

H.64 f(x)

3x-2sex>-1 3 se x = -1 { 5-ax se x<-1

f(x)

~I~:',~~,~' aI Iim

c} 11m f(x) x+-l

f(x)

x+-l-

= 13x - 21 2 _ 3x

a) lim

H.6

H.66 Dada a fu nção f defi nida por

Determine a E IR para que exista lim f(x) X"-l

definida em IR - {-,).

x+_l T

Iim (x + [xl) X"l

x+o-

f(x) =

H~) ~11 x +

./~

(x - [xl)

e) lim (x - [xl) x+!-

(-1) = -1

tirem.

//

(x + [xJ)

X+l-

X+l+

Considerando que 11m f(x) =1= lim f(x) concluímos que não existe lim f(x). x+o+ x+ox.. o Nos exercícios H.59 a H.54 é dada uma função f. Calcule os limites indicados se exis-

.

lim (x - [xl) X"l

X+l+

e lim

f)

x ;;'0

f(x) = lim x-+O+

x~O+

[xl

se

x+2

{

5} 2'"2 .

(*)

c) lim f(x) x"2

A função máximo inteiro 11 e função f: IR .. Z tel que f(x) que n"'; x < n 1.

+

=[ x 1= n

tel

53-H


48.

Definição x

Seja I um intervalo aberto que con· tém a. Seja f uma função definida em I _ {a}. Dizemos que quando x se aproxima de a, f(x} decresce ilimitadamente e escrevemos lim f(x} = -

, 1 I Im - - =

00

x+a

x->I- X -

se, para qualquer número M < O, existir li > O tal que se O < I x - a I < li então f(x}

50.

Em símbolos, temos .

=

("IM < O, 3li

-00 _

1 --=+00

x+l + X -

1

Definição

>OI

o<lx - ai < li

==>

f(x) < M)

Seja I um intervalo aberto que contém a e seja f uma função definida em I - {a}. Dizemos que, quando x se aproxima de a por valores maiores que a, f(x) cresce ilimitadamente e escrevemos

x+a

f(x)

lim

Insistimos novamente em observar que o símbolo "- 00" não representa nenhum número real, mas indica o que ocdtre com a função quando x se aproxima de a.

49.

,

e I Im

_00

1

------t-----.J4------

y

< M.

lim f(x}

Observemos que se x assume valores próximos de 1, à esquerda de 1, os valores da função decrescem ilimitadamente e se x assume valores próximos de 1 à direita de 1, então os valores da funçã~ crescem ilimitadamente. Estamos consideran~o os limites laterais que são "infini· tos e escrevemos:

Consideremos agora a função h definida por h(x) =

1 x _ 1 para todo x real

=

M

+ 00

x"*a+

se, qualquer que seja o número M > O existir ó > O tal que se O < x - a < então f(x) > M. Em símbolos:

i

lim x+a+ f(x) =

+00 _

x

("IM> 0,3 li > O I O < x - a < li

'* f(x) > M)

e x*" 1. Atribuindo a x valores próximos de 1, porém menores que 1, temos:

Coloquemos com símbolos as definições de lim f(x) = -

00,

x+a+

O

0,5

0,75

0,9

0,99

0,999

x f(x)

-1

-2

-4

-10

-100

-1000

e atribuindo a

x

valores próximos de

1, porém maiores que

e lim

f(x)

= -

00:

x+a-

~~a+ f(x) - -

1, temos:

lim

00 -

f(x) - + 00

_

("IM

< 0,3

li > O I O < x - a

("IM> O, 3 li > O I - li

x+a-

x f(x)

2 1

1,5

1,25

1,1

1,01

1,001

4

10

100

1000

lim

2

lim f(x) = + 00 x+a-

f(x) = -

00 _

("IM

< O,

3 li > O I - li

<x

<x

< li =

- a

- a

<O

<O =

f(x)

~

< M)

f(x) > M)

f(x)

< M)

x+a-

57-H 56-H


Para concluirmos que os valores de uma função cresciam infinitamente ou decresciam infinitamente, quando x se aproximava de a, pela esquerda ou pela direita de a, construímos uma tabela de valores da função quando x estava próximo de a. Vejamos como chegar à mesma conclusão sem construirmos essa tabela.

Assim, existe li l > O tal que

o<

Ix - ai

< li l

==>f(x) >

c

2"

(1 )

>0

isto é, f(x) > O quando x está próximo de a. . f(x) Mas, por h ·Ipotese, g(x) > O quan d o x esta. pró' xlmo d e a, entao 9 x N

Sejam

lim f(x) = c

e 9 funções tais que

x+a

I) lim f(x) = X+.

+ 00

g(x)

11) lim f(x) = x+. g(x)

00

'* O e

lim g(x) = O então:

se f(x) < O quando x está próximo de a; g(x)

Demonstração Faremos a demonstração de I e deixaremos a prova de li, que é feita de . f(x) modo análogo, a cargo do leitor. Para demonstrar que 11m - - = + 00 deveX+. g(x) mos mostrar 'v'M > 0,3 li > O I O < Ix - ai <li ==> f(x) > M

>O

quando x está próximo de a. Pela definição de lim g(x) = O, temos: x+.

X+a

se f(x) > O quando x está próximo de a; g(x)

()

'v'€>O,3li 2 >01 0<lx-al<li 2 =lg(x)I<€ mas,

Ig(x)i = g(x)

já que g(x) > O quando x está próximo de a, então:

'v' € > 0,3 li 2 > O

I O < Ix - ai < li 2

=

g(x) < €

(2)

Com base nas afirmações (1) e (2). podemos concluir que para qualquer € > O existe li = min {li l , li 2 } tal que

O < Ix - a I

f(x) g(x)

< li =

>

c 2€

c Assim, dado M > O, seja € = 2 M e li = min {li I, li 2 } > O onde li I e li 2 são números positivos que satisfazem (1) e (2) respectivamente, então: dado M > O, 3 li = min {li I, 1i 2 } > O tal que O < Ix _ a I < li

Vamos considerar dois casos:

==

f(x) g(x)

>

c

2€

c

2C'= M 2M

TI? caso: Supondo c > O Por hipótese temos

Iim f(x)

c > O.

=

o que prova que

isto é:

X+.

'v' € > O. 3 li > O I O < Ix - ai < b T omemos

c € = '2' entao eXiste N .

oI

==

> O ta I que

=-

< f(x)

- c

ou ainda

O < Ix - ai

58-H

2!? caso: Supondo c < O c

"2

Se lim f(x) = c < O então lim [-f(x) l x+.

• •. d ta proxlmo e a,

ou seja O<lx-ai<ol

< OI =

c 2

< f(x) <

3c 2

= + 00.

x+. g(x)

If(x) - cl < €

==> If(x) - cl <

c 2

l 'lm f(x)

c

< 2'

x+.

=

-c> O e se f((X)) > O quando x es9 X

-f(x) O d .•. d entao -(-I > quan o x esta proxlmo e a. N

-g x

Considerando as funções h e j tais que h(x) = -f(x) para todo x do domínio de f e j(x) = -g(x) para todo x do domínio de g, temos pelo primeiro caso já demonstrado lim x+.

h(x) j(x)

=+00

59-H


mas

então

h(x)

-f(x) •

f(x)

j(x)

-g{x)

g(x)

f(x)

rim x+a

sinal de f(x) = 1 - x

+

sinal de g(x) = (x - 2)2

+

-

O

-

+00

g(x)

Observação: este teorema continua válido se "x~a+"

"x -+ a"

for substituido por

ou "x~a-".

+

+

O

i

sinal de

+

f(x) 1 - x g(x) = (x - 2)2

Notemos que

f(x) g(x) =

I

-

O

-

I I I

: 1 - x

(X":2j2

<O

quando x está próximo de 2, então

. 1 - x 11m ~_ 2) =-00

EXERCICIOS

X+2

H.59 Calcule:

X

H.70 Calcule:

3x + 2 a) lim 2 x+I (x - 11

3x -4 a) lim (x - 2)2 x+2

d) lim x+o

1 - x bl lim (x - 2)2 x+2

2x + 3 . b) Ilm~

e) lim

Solução

1 - 3x c) lim (x - 1)2 x+1

ai Como lim (3x + 2)

x+-3

=

5 e lim (x - 1)2

x-H

f(x)

=

O, estudemos o sinal de g(x)

=

3x + 2 (x _ 1)2

5x + 2

x+-l Ix

+ 1I

2x 2 + 5x - 3 f) lim Ix + 21 X'1'-2

H.71 Calcule: 1

-2/3 sinal de flxl = 3x + 2 sinal de g(x) = (x - 1)2

:

+

O

+

!

+

O

2x + 1

a) lim

x

Solução: (2x + 1) = lim

Como lim

+

x-H -

(2x + 1 I = 3 e lim

x..,.t+

f(xl 2x + 1 91Xi =...,.--:-;-

>O

quan d o x esta• prÓ· xlmo de 1 • enta-o:

3x + 2

lim~

quando x está próximo de 1.

=+00

sinal de f(xl = 2x + 1

x

-

sinal de g(xl = x - 1

-

O

+

-

X+I

x+2

quando x está próximo

x+l+

-1/2

f(x) = (x 3x _+1)2 2 g(x)

b) Como lim (1 - x) = -1

(x - 1) = lim (x - 1) = O, estudemos

x+l-

+

+

O

X+l+ X - 1

+

. o Sinal de

f(x) 3x + 2 g(x) = (x - 1)2

2x + 1

b) lim

X+l- x - 1

sinal de

N otemos que

x

3x 2 - 5x + 2 x2

x-+-2

quando x está próximo de 1.

&O-H

x

2

e lim (x - 2)2 X+2

de 2.

. f(x) = O. estudemos o Sinal de g(x)

=

1- x (x _ 2)2

sinal de

f(x) 2x + 1 9(x) =...,.--:-;-

+

O

-

+ O

+

II I I

+

: 61-H


f(x) 2x + 1 Notemos que g(x) ~"""X"=1

<O

quando

está próximo de I, à esquerda, en-

)C

tão

2x + 1 x-I

>O

quando

)C

lim

X~l""

PROPRIEDADES DOS LIMITES INFINITOS

Veremos a seguir dez teoremas cujos enunciados serão apresentados com o símbolo "x -+ a", mas que serão válidos se trocarmos esse símbolo por "x -+ a-H ou "x ..... a+".

2x + 1 = _ 00 X+l- x - 1

lim

e ~ g(x)

11.

está próximo de 'I, à direita, então

~~+oo

52.

Teorema

x - 1

2x + 1

.

Se

2x + 1

Observemos que não tem significado falarmos em I Im - - pois lim - - = - ooe

x-I

X"I

x"l- x-I

lim f(x) = + 00 e

lim g(x} = + 00, então lim (f + g) (x)

x+a

x+a

+ 00

.

x+a

Demonstração Para provarmos que x + 4 a) lim -x"-2- x + 2 b)

f)

rIm

x + 4 -X+-2+ x + 2

c) lim x+3-

3x + 2 lims + 5 - 2x x+"2

mas

2x + 3 ( )3 x+]- x - 1

1 - 2x

h) lim

x -3

X+l+

rIm 3x + 2 -x..!- 5 - 2x

lim

j)

lim

2

"I

~

°I

> O, 3 li I >

O < I x - a I < li I

e

lim g(x) = + 00,

isto é, se tomarmos

x"a

x+o x

~

~

2x 2 -" 3x - 5 (2 - x)3 x+2+

> O,

temos:

=-- f(x) > O,

>

~

temos:

=g(x)

então considerando li = min {li l

H.73 Mostre pela definição que lim-;'

~

isto é, se tomarmos

> M

1

2x 2 - 3x - 5 (2 - x)3 x+2-

n

Iim f(x) = + 00, x"a

2x + 3 ( )3

x -

=-- (f + g)(x)

V M > O, 3 li > O I O < Ix - a I < li

g) lim

1 - 2x d) Iim x+3+ ~ e)

lim (f + g)(x) = + 00 devemos provar x"a

H.72 Determine:

+ 00

,

M > 2""

li 2 }. temos:

VM>O,31i>OI O<lx-al<1i

==>f(x)+g(x»

~ +~

=M

H.74 Mostre pela definição que: a) lim

1

--,- =-00

X+O- X

.

b) 11m

x+o+

3

1

= +00

53.

Teorema

X

Se

lim f(x) = -

00

e

x+a

lim g(x) = -

00.

x+a

então Iim (t + g)(x) = -

00.

A

x+a

demonstração deste teorema é feita de modo análogo ao teorema anterior; deixaremos a cargo do leitor.

54.

Observação Se

Iim f(x) = + 00, Iim g(x) = + 00, Iim h(x) = x+a

x+a

00

x+a

e lim i(x) = -

00

não

x+a

podemos estabelecer uma lei geral para os seguintes limites:

62-H

63-H


lim (f - g)(x); lim (h - i)(x} e Iim (f x+a x+a x+a

+ h)(x}.

Por exemplo, consideremos as funções

1 f(x) =4

para todo x real e x

. 41 IIm x+o X

= + 00

*- O. e

x

1 e g(x) =2 definidas

x

= + 00,

lim f(x) x+a

Se

O < Ix - ai

< 1)2

então

Observemos que

= + 00

f(x)

>

O e 1)2

>O

tais que se

> ~. ex

= min {I)I' 1)2}, temos que para todo M> O, existe I) >0

Considerando I)

I'Im 2 1 x+o X

M ex

então existem

< Ix - aI < I)

tal que se O

então (f • g)(x)

= f(x)

• g(x)

>~ . ex = M, ex

e calculemos lim (f - g)(x) x+o

= lim

(f(x) _ g(x))

x+o

= lim (1 x+o

1\

\7 - )(2)

56.

Se lim f(x) = -

Se considerarmos as funções

X+I+ X - 1

=+

00

e

e lim g(x) = b x+a

00

x+a

1 3 f(x) = - - e g(x) = - 3 x - 1 x - 1

, 1 IIm - -

Teorema

3

lim 3 X+I+ X -

definidas em IR - {1 } ter (amos

=+

I) se

11) se

> O, b < O, b

então então

00

*- O,

então:

Iim (f. g)(x) = - 00 x+a lim (f. g)(x) = + 00 x+a

A demonstração deste teorema ficará como exerdcio.

Mas lim (f - g)(x) = Iim (f(x} - g(x)) = lim (_1-1 X+I+ X+I+ X+I+ X =

_

- 3_3- )

X - 1

=

X2 + x - 2 I' (x - 1)(x + 2) x +2 , I Im = Im = lim - 1 X+I+ (x - 1 )(x 2 + x + 1) X+I+ (x - 1)(x 2 + x + 1) x+l+ x 2 + x + 1

57.

Observação Se

+ 00 (ou -

lim f(x) =

00)

e lim g(x) = O,

x+a

nula, não podemos formular uma lei geral para

Iim (f. g)(x), x+a

Teorema Se

lim f(x) = x+a

I) se

11) se

> O, b < O, b

R' e as funções gl (x)

+ 00 e Iim g(x) = b X+a

*-

O

64-H

4

=

=~ definidas em x

Observemos que

então

lim (f • g)(x) = + 00 x+a lim (f • g)(x) = - 00 x+a

Iim gl(x)

x~

= lim x4 = O e lim g2(X) = lim x 2 = O. x~

X~

x~

Mas

Demonstração

O<

= x

~

e f 2 (x) x e g2 (x) = x 2 definidas em IR.

Por exemplo, consideremos as funções fi (x)

55.

onde 9 não é a função

x+a

Faremos apenas a demonstração de I. Se lim g(x) = b > O, então existem x+a Ix - ai < 1)1 então g(x) ex,

>

lim (fI" gl)(x)

ex> O

e

1)1 > O tais que se

x~

=

lim (_1_. x 4 x2

x~

) =

lim x 2 = O

x~

lim (f2 .g2)(X) = Iim (_1_ • x 2 ) = lim _1_= x~ x~ x4 x~ x 2

e

+00.

65-H


58.

Teorema

1 = +00, lim g(x ) = 11m . 1 = +00 e I'Im h() lim f(x) = lim ~ 4" x = x+o

Se lim f(x) = + 00 e lim g(x) = + 00, então Iim (f. g)(x) = + 00. x+a

Da

~8

x+o X

x+o

x+o X

x+o

-1

= lim - , =-00 x+O X

Demonstração Se lim f(x) = + 00, então existem x+a

O < I x - aI < 6 1

,

então

f(x)

VM > O

> v"M. e se I

-IM

e 61

>O

tais que se

Mas

lim g(x) = + 00 ,então existem x+a

> O e 62 > O tais que se O < Ix - ai < 6 2 ,então g(x) >

9

x+O

x+O

< Ix

- ai

< 6'

então f(x) • g(x)

> v"M . v"M

= M,

= lim

lim (.!!...)(x) h x+O

59.

lim f(x) x+a

x+a

x+a

à do teorema anterior,

62.

Teorema

= lim x 2 = O x+o

~ ~ -

~4

)

-=l

= lim --12

= - 00

x+O X

x2

-1 h x2 ) lim (-)(x) = Iim - - = lim (-1) = -1 f x+o ~ x+o x+o

= + 00 e lim g(x) = - 00, então lim (f. g)(x) = - 00 ,

A demonstração deste teorema é feita de modo análogo portanto, ficará como exercício.

60.

x+o

Teorema Se

1

?"

Considerando 6 = min {6 1 , 6 2 } temos para todo M >0, existe 6 >0 tal que se O

~ ~2

= lim - -) lim (-)(x) f

v"M .

Teorema Se

Se Iim f(x) = - 00 e lim g(x) = -00, então Iim (f. g)(x) = + 00.

x2

lim f(x) x+a

x+a x+a x+a Demonstrar este teorema a título de exercício.

~

+ 00,

então

lim f(1 ) = O. x+a

X

Demonstração Se lim f(x) = + oo,então existem M> O e 6> O tal que se O< Ix - ai <6,

61.

x+a

Observação Se lim f(x)

então

= + 00

x+a

(ou - 00) e Iim g(x) = + 00 (ou - 00) então não podemos x+a

estabelecer uma lei geral para

lim x+a

Mas

I f(~) I < ~

9

x

56-H

> M.

f(x) > M > O <=> If(x) I> M <=>

(-..!.. )(x).

1 1 1 Por exemplo, consideremos as funções f(x) ="2" ' g(x) =7.4 e h(x)= - x 2 definidas em IR·. Observemos que

f(x)

Tomando

= M' temos para todo

X

0< I x - a I < 6, então

1_

1 _ f(x)

01 < €

> O,

existe 6

e, portanto,

>O

tal que se

lim f(1 ) = O. x+a

X

67-H


63.

Teorema

Se

lim f(x) = - 00,

então

x+a

66. Antes de prosseguirmos, façamos um resumo dos teoremas apresentados, lemo brando que as proposições permanecerão válidas se substituirmos o símbolo " X ~ a" por "x ~ a+" ou " X -+ a-".

lim f(1 ) = O, X+a

X

A demonstração ficará a cargo do leitor Dados

64.

lim f(x) =+00 x+a

lim g(x) =+00 x+a

lim (f + g)(x) = + 00 x+a

lim f(x) x+a

=-00

Iim g(x) =-00 x+a

lim (f + g) (x) = - 00 x+a

Iim f(x) x+a

::;+00

lim g(x) = b*O x+a

Iim (f • g)(x) = x+a

lim f(x) x+a

:-00

Iim g(x) = b*O x+a

lim (f • g)(x) x+a

Iim f(x) x+a

= + 00

lim g(x) =+00 x+a

lim (f • g)(x) = + 00 x+a

lim f(x) x+a

= +00

lim g(x) x+a

lim (f • g)(x) = -00 x+a

lim f(x) =-00 x+a

lim g(x) x+a

Teorema

Se

lim f(x) = O. então x+a

lim x+a

I

1

1= + 00,

f(x)

Demonstração

Se Iim f(x) = O, então existem I: > O e 6> O tais que se O< Ix - ai <6, x+a então If(x) 1< 1:, Mas

Tomando O<

Ix

1 M = €'

- a I < 6, então

temos para todo M > O, existe 6 > O

I I

f(1X ) > M

e, portanto,

I I=

~z;, f(~)

tal que

Se existir 6 tal que para todo x que satisfaça O < I x - a I < 6 tenhamos lim -f(1) x+a X

=

lim x+a

1__1

1 = f(x)

=-00

lim (f· g)(x) x+a

r

=

se - 00 se oo

{::

se se

b>O b<O b>O b<O

= +00

+ 00,

Observação

f(x) > O, então

=-00

se

lim f(x) x+a 65.

Conclusão

+ 00,

= +00

lim

th

. 11m

1 O TIl = X

x+a

X

= O

lim f(x) = _ 00 x+a

x+a

lim f(x) = O x+a

I"Im - 1 x+a' f(x)

I I = +00

Se existir 6 tal que para todo x que satisfaça O < I x - a1< 6 tenhamos f(x) < O, então

IIm ' - 1 = "Im x+a f(x) x+a

68-H

I-f(x)1- I= - 00 69-H


68.

Não poderemos estabelecer uma lei para os seguintes casos: lim f(x) =+00 x+.a

lim g(x) ;+00 x"a

Iim (f - g)(x) x"a

lim f(x) =-00 X.. a

lim g(x) ; - 00 x"a

X..

lim f(x) x"a

lim g(x) ; - 00 x"a

lim (f + g)(x) X.. a

lim g(x) ; O X" a

lim (f • g)(x) x"a

Iim g(x) ; - 00 (ou + 00) X.. a

lim (x) x"a 9

lim f(x) x"a

;

;

+00

+ 00 (ou - 00)

lim f(x) ; + 00 (ou - 00) x"a

;

Definição

? y

Iim (f - g)(x) a

;

?

;

?

Seja f uma função definida em um intervalo aberto la, + 00[. Dizemos que, quando x cresce ilimitadamente, f(x) se aproxima de L e escrevemos lim

-!.

;

;

L+€

f(x); L

L L -€

x++oo

?

se, para qualquer número N > O tal que se . x If(x) - LI <e.

?

> O, > N

existir então

N

x

Em símbolos, temos:

111.

LIMITES NO INFINITO f(x} ; L _

Seja a função f definida por f(x); x + 2 para todo x real e x =F O. Atrix buindo a x os valores 1, 5, 10, 100, 1000, 10000 e assim por diante, de tal forma que x cresça ilimitadamente, temos:

67.

(Ve;

> O, 3 N >

Consideremos novamente a função f(x); x + 2 . Atribuindo a x os valores x -1, - 5, -10, -100, -1000, -10000 e assim por diante, de tal forma que x decresça ilimitadamente, temos:

69.

x

1

5

10

100

1000

10000

f(x)

3

1,4

1,2

1,02

1,002

1,0002 y

Observamos que, à medida que x cresce através de valores positivos, os valores da função f se aproximam cada vez mais de 1, isto é, podemos tornar f(x) tão próximo de 1 quanto desejarmos, se atribuirmos para x - - - - - - + valores cada vez maiores. x

Escrevemos, então: lim x++oo

70-H

x+2 x

x

-1

-5

-10

-100

-1000

-10000

f(x)

-1

0,6

0,8

0,98

0,998

0,9998

Observamos que, à medida que x decresce através de valores negativos, os valores da função se aproximam cada vez mais de 1, isto é, podemos tornar f(x) tão próximo de 1 quanto desejarmos, se atribuirmos a x valores cada vez menores. Escrevemos, então: lim x+-oo

x + 2

x

71-H


70.

Definiç ão

y

Seja f uma função definida em um intervalo aberto ]- 00, a[. Dizemos que, quando x decresc e ilimitad amente, f(x) aproxima-se de L e escrevemos lim

f(x)

~

L

x~-oo

se, para qualque r número N < O tal que se x If(x) - LI < f.

f

Observamos que, a medida que x decresce através de valores negativo s, os valores da função crescem e ilimitad amente. Em outras palavras , dizemos que podemos tornar f(x) tão grande quanto desejarmos, isto é, maior que qualque r número positivo, tomand o para x valores negativos cujos módulo s sejam suficientemente grandes e escrevemos

L+ f

-------r-------\

L

--- --1 --- -

L

> O, existir < N então

lim x+

-f

73.

I

I N

x

Em símbolo s, temos: lim x+-oo

f(xl

=L

<=>

l'v'E > O, 3 N

<OI

x

<N

==> If(xl - LI

f(x)

~

Definições

y

Seja f uma função definida em um intervalo aberto la, + 00[. Dizemos que, quando x cresce ilimitad amente, f(x) cresce também ilimitad amente e escrevemos

< fI

lim x++

+ 00

-00

f(x)

~

M

+ 00

00

se, para qualque r número M > O, existir N > O tal que se x> N então f(x) > M.

71. Seja a função f(x) = x 2 , definida para todo x real. Atribui ndo a x os valores 1, 5, 10, 100, 1000 e assim sucessivamente, de tal forma que x cresça ilimitad amente, temos:

N

Em símbolo s, temos: lim

f(x)

x++oo

= + 00

<=> ("tM > O, 3 N > O I x > N

=

x

f(x) > M)

x

x

f(x)

10

5

25

100

Coloqu emos com símbolo s as definiçõ es de f(x) - 00, lim f(x) ~ + 00 e lim

1000

lim

100

10000

1000000

x++oo

Observamos que, a medida que x cresce através de valores positivo s, os valores da função também crescem e ilimitad amente. Em outras palavras , dizemos que podemo s tornar f(x) tão grande quanto desejarmos, isto é, maior que qualque r número positivo , tomand o para x valores suficien temente grandes e escrevemos: f(x) = + co

lim

lim

x+-oo

lim x+-oo

< O, :a. N >

f(x)

-

f(xl

+ 00 <=> ('I1'M > O, 3 N

x++oo

00

<=>

('I1'M

f(x)

=-

00

X+-oo

O I x > N

=

f(xl

< M)

<OI

x

<N

==> f(xl >

<OI

x

<N

=== f(xl < M)

M)

x++oo

72. Se agora atribuir mos a x os valores -1, -5, -10, -100, -1000 e assim sucessivamente, de tal forma que x decresça ilimitad amente, temos: x

f(x)

-1

-5

-10

-100

-1000

1

25

100

10000

1000000

lim x+-

f(x) = 00

00

<=> ('I1'M

< O,

3 N

Para conclui rmos algo com relação ao compor tamento dos valores da função quando x crescia ou decrescia ilimitad amente, constru ímos uma tabela de valores de x e f(x). Vejamos como chegar à mesma conclus ão, sem constru irmos essa tabela.

72-H

73-H


74.

Teorema

76.

Se c E IR

então

lim

c

lim

~

c

c

Teorema Se n é um número inteiro positivo, então:

X~-oo

X~+OO

I) lim

Demonstração

x~+oo

A demonstração é bastante simples, já que

> O,

V €

3 N

>O

I x

>N

== O ~ Ic - cl

x"*+

75.

c

Demonstração

c

~

~ O ~ O

x~- 00 X

é trivialmente verdadeira e portanto Iim

n

11) lim

<€

_1_ x"

Fica como exercício

00

Teorema

77.

Se n é um número inteiro e positivo então I) Iim x n =+00 x++

Se f(x) ~ ao + alx + a2x2 + ... + anx n , a n então: lim f(x) Iim (anx n ) e lim f(x)

00

xn ~

11) lim

{+oo

n é par n é ímpar

se se

-00

x+-oo

Teorema

x++oo

x++ 00

x+-

"* O,

é uma função polinomial,

00

Demonstração Faremos a demonstração de II por indução sobre n.

tI? caso:

n é lmpar

Demonstração

A proposição é verdadeira para n ~ 1 pois ("IM ==Iim X~-oo.

~x<M)

x+-

Por aplicações sucessivas das propriedades e teoremas, temos: lim f(x) ~ Iim (ao + ai x + a2x2 + + anx n )

< O, 3 M < O I x < M ~

x++

00

X++ 00

00

Supondo que a proposição seja verdadeira para n dadeira para n ~ p + 2, isto é, se lim

x+-

xP ~ -

00

~

x P +2 ~ -

então lim

00

... +

p mostremos que é ver00.

X+- 00

+ _a_l_ + _a_2_

De fato, por aplicações sucessivas dos teoremas já vistos, temos: lim

x P +2 ~ lim

x+- 00

X+- 00

Mas

x2

lim x+-oo

lim

(x p

x P +2 ~ -

lim X+-oo

x 2 ) ~ Iim

xp

X7- 00

X· lim x+-oo

x ~ + 00 e

lim

anx O - 1

x2 xP

-

00

portanto,

lim

x+-oo

x++ 00

00.

x+-oo

lim

(~+ anx

As demonstrações para o caso em que n é par e da parte I ficam como exercícios.

+ ... + lim

x.".+ 00 X

1

~

l

a n' l

anx

1)

lim x++oo

+~ + ... + 1\ ~ anx

1 -n- + lim

ao. 11m

x++ 00 ao

74-H

+ ... +

an x O - 2

pois

X+- 00

lim

1)]

2

ai

X++ 00 ao

)

lim

1

li""=!

X++ 00 X

+ lim

1

a2. - . 11m

x++ 00 ao

X++

--n:2 00

+... +

X

1

x++oo

75-H


78.

Ho76

Teorema Se

+ b2 x

2

*

lim X++

00

0

n f(x) (a - = I'Im - - x n-m) g(x) X.H 00 b m

, I Im

e

f(x) -

x+- 00 g(x)

b) lim

x+- 00

f(x) -X'H 00 g(x)

lim x++

n

(

ao

anx

lim + 00

c) 11m

+

aI

a2

+

anx n - 1

an x n - 2

+

b1 b2 m ( - b-o- + + + bmx m b X m-2 b Xm-l bmx m m

(~). m bmx

lim x"*+

00

an lim -_. x++ 00 ( bm

aI

+

X++ 00

H.7B

(1 _ xn"

n E N*

( 2 ),cEIR* di 11m x+ -00 c

Encontre: Vx 2 - 2x + 2

a) lim

b) 11m

x++oo

H.79

a2 --+ + a xn- 2 anx n - t anx" n bo bl b2 + m + b b Xm-2 m-I X bmx m m

- I'Im 1 _

1) 1)

+

Ic • xl, c E IR*

x++oo

x+-oo

n

18 _ x 3 )

X+~OO

(x n - 11, n E N*

bl lim

00

ao

X+

fi 11m

Encontre: ai Iim

lim 00

15x 2 - 4x + 31

x++oo

, 11m

(3x 3 - 4)

x+- oo

X-++DO

Ho77

x++

e) Iim

14 - 5x)

c) tim

bm

(4 - x 2 )

x++oo

x+-oo

n x n-m) (a-

I'Im

=

d) lim

x++oo

Demonstração

anx

(2x + 3)

a) lim

n f(x) = ao + alx + a2x2 + .. + anx , a n O, e g(x) = bo+b1x+ m + .. , + bmx , b m O são funções polinomiais então

*

Encontre:

Vx 2 - 3x + 5

x+-oo

Encontre:

5x 2 - 4x + 3 3x + 2 x+-oo

+1

3x + 2 a) lim x++oo 5x - 1

cl lim

+ .. + 1

5 - 4x bl lim x+-oo 2x - 3

4x - 1 di 11m 3 2 x+-oo x + 5x - 2

+

"O

o

( -a n- Xn-m) bm

Solução: 3x + 2

3x

a) 11m - - - = 11m x++oo 5x - 1 x++oo 5x

11m x+oo

~ 5

3

'5

EXERCICIOS

bl 11m 5 - 4x = 11m -4x - 11m (-2) = -2 x+-oo 2x - 3 x+-oo ~ - X-r-OO

H.75 Encontre:

cl 11m

ai lim

(4x 2 - 7x + 3)

(5x 3 - 4x 2 - 3x + 21

c) 11m x+-oo

x++oo

b) 11m

(-3x 3 + 2x 2 - 5x + 3)

H.BO

a) 11m (4x 2 - 7x x++oo

76-H

+ 3) = 11m

(4x 2 ) = + 00

+ 5x - 2

= 11m

4x _ 11m

x-+-oo 3x 2

~=O

x+-oo 3x

(-3x 3 + 2x 2 - 5x + 3) = 11m

(-3x 3 1

c) 11m x+-oo

di lim

(3x 4 - 7x 3 + 2x 2 - 5x - 4) = 11m

=-00

~ di 11m 2 x+-oo x + 1

4x - 3 b) 11m - - x+-oo 3x + 2

e) lim

x2 - 4 ---

fi lim

x+-oo x+-oo

(3x 4 )

+00

3

a) 11m

_00

x++oo

(5x 3 )

Encontre:

3 - 2x --X++ 00 5x + 1

x++oo

(5x 3 - 4x 2 - 3x + 2) = lim

x+-oo

4x - 1

x+-oo 3x 2

X+-oo

Solução:

x++oo

d) 11m

(3x4 - 7x 3 + 2x 2 - 5x - 4)

d) 11m

x++oo

bl 11m

5x 2 - 4x + 3 3x + 2 x++oo

clllm X-H.oo

x

+ 1

x++oo

x2 - 3x + 4 3x 3 + 5>.2 - 6x + 2

x2 + 4 3 - 1 8x x+-oo

77-H


X2 + x + 1 gl 11m I 1)3 - x 3 x+-oo x +

j)

x++oo

(2x - 31 3 hl lim X7+ 00 xix + 1IIx + 21 il

11m

kl lim x+~oo

(2x - 3)313x - 21 2 xS

H.83

Encontre

(~~

11m

Ix + 21 4 - (x - 11 4 12x + 3)3

x++oo

- xl

Solução

13x + 2)3

Observemos que

lim X7-00 2x(3x + 1 )(4x - 1I

Vx 2 - 3x + 2

lim

+00 e

11m

H.81

Encontre:

bolo V x 2 - 2x + 2 bl 11m x + 1 x+-oo

V x 2 - 2x + 2 ai 11m x + 1 x++oo

por

Vx 2

+ 3x + 2

V x 2 - 2x + 2

11m

+ 00,

11m

x+-oo

x++oo -00

Ix +

11

+ 00.

.vX2+3x+2 +x

x++oo

+00 e +00

e não têm significado os símbolos

x+-oo

+00 _ 00

V x 2 - 2x + 2

+ 1

+ 00, lim

(3x + 21

lim

Notemos que

(1

_2 + .l_I 2

x

+

x 1

x

2

Ixl

2

--+x x2

x(3 V x 2 + 2x + 3

- x

+2 1

V x 2 - 2x + 2 x + 1 x++ oo

j1 --x +-;2 2

x Iim x+-oo

x(1 +~)

V x 2 - 2x + 2 x + 1 x+-oo

lim

-x lim x+-oo

j1 --;-+;2 2

lim x"" 00

lim x+-oo

x

-

1 +x

lim

(V x 2 + 2x + 3

V x2 + x + x + 1 x..:,.+oo

V x2 + x + bl lim x + 1 x+-oo

11m

x++oo

2

+-;+;2+1

-j1 - -; + 1 +x

x++oo

)1

2x 2 - 3x - 5 ~ x +1

x2 e) 11m ---,---:-;= x++ 00 1 + x '\{"X

ti lim

X+-oo

x+~ x2 + 1

x

x2 -1

H.84

~2

2

+--;-+-xr

+ 1

Encontre:

ai lim

(Vx 2 + 3x + 4

x)

IVx 2 + 3x + 4

- x)

el lim x++oo

x++oo

c) lim x++oo

d) lim

f)

lim

(~

~1)

IV x 2 - 4x + 5 - V x 2 - 3x + 4)

X++OO

I~

-

v;-=-;)

g)

lim x++oo

(Vx 2 - x + 1

- xl

hl Iim

(x-~I IVx 2 + ax + b - xl

x++oo

x++oo

H.85 Encontre:

a) 11m

2x 2 - 3x - 5

~

3

x

3

2

x++oo

78-H

- xl

x+-oo

ai 11m

x+-oo

x

3

1

3+2-

1

bl Iim Encontre:

di lim

x

+~ +.2-2 +

2

2

2

x(1 +2- 1

j1 -x-+-;z2 2

2

x

x++ oo

3 +2-

x

portanto

lim

cl 11m

+00 eosimboh

+00

portanto:

H.82

=

+ 00 não tem significado. Fazemos então

x

e

(Vx 2 + 3x + 2 + xl

x++oo

X-'Jo+OO

Notemos que

x

- x

3x+ 2

V x 2 - 2x + 2 Ix + 1 I

(+ 001.

1+ 001

Observemos que

11m

mas, carece de significado o sím·

(V x 2 + 3x + 2 - xl

Solução

11m

+ 00,

Ixl

x++oo

X++ OO

b) lim

x +

.J x 3

- 5x 2 - 2

~

v':-

x++ oo VX + 2

cl lim x++ oo

V x 2 + 2x + 4 - x x -

V x2

- x

+ 1

~ - ~ 79-H


H.B6

Encontre:

ai lim x-?+

e

v-;

iJx

+

Jx

+

Jx

+

Vx x

+ V;

-

vÍxl

> 0,

2

x++ oo

00

b) lim x++oo

:=

> 0,

"I M

H.87 Mostre pela definição que: x2

N

então considerando

V4x + 1

a) Iim

>

==g(x)

V;+~+\f;

c) lim x++oo

3N

max {N 1 , N 2 },

=

>°I

x

>

N

temos:

M 2

temos:

== f(x)

+ g(x)

M

M

>2

+2

=

M

Faremos a apresentação dos enunciados dos demais teoremas e deixaremos a b) lim x2 x+-oo

+ 00

x++oo

H.as

M

+ 00, isto é, se tomarmos

g(x)

lim

= + 00

cargo do aluno as demonstrações,

Mostre pela definição que: x3 =

ai lilTl

b) lim x3 = x+ _00

+ 00

x++oo

ao.

00

Teorema

Se lim

f(x)

x?'-+

-

g(x)

e Iim

00

-

então

00,

+

(f

Iim

g)

-

00,

x++ oo

X+T 00

00

Observação

IV. PROPRIEDADES DOS LIMITES NO INFINITO

Se

lim

f(x)

= + 00,

lim

x++oo

Veremos em seguida dez teoremas cujos enL!nciados serão apresentados com o símbolo "x -->- + 00" e não perdem a validade se esse símbolo for trocado ~'or "x -->- - 00". Estes teoremas são basicamente os apresentados nas propriedades dos limites infinitos, com adaptações para aplicações de limites no infinito.

e lim

i(x) = -

x++

= + 00,

g(x)

lim

x++oo 00,

h(x)

= - 00

x++oo

não podemos estabelecer uma lei geral para os seguintes limites

00

lim

(f - g),

x++oo

lim

(h - i)(x)

e

x+-+oo

lim

(f

+ h)(x)

x++oo

Por exemplo, consideremos as funções f(x) = 3x - 2 e g(x) = 3x + 5 definidas para todo x real. Observemos que

79.

Teorema Se

lim x++

lim

+ 00 e lim g(x)

f(x)

x++

00

+ 00, então lim

(f

+ g)(x)

x++oo

00

(3x - 2) =

x++

lim

Para provarmos que

"I M

> 0,3

N

>

lim

°I

+ g) (x) = + 00 devemos provar:

>

N

=

(f + g)(x)

>

f(x) = + 00,

lim

isto é, se tomarmos

aO-H

M

2 > 0,3

N[

>°I

x

> N[

=

>

2

f(x)

= lim

0,

>

M 2

(-7) = -7

x++oo

M

2

+ 2x - 3 defi-

nidas para todo x real, teríamos: (3x 2

-

7x

+

1) =

temos: mas

lim

(f - g)(x)

lim x++oo

[(3x 2

(2x 2

+ 00 e lim x++

lim

+ 2x - 3)

=

+ 00

00

[f(x) - g(x)]

x++oo

x++ oo

"1

+ 5)]

x++ oo

M

+ 00

X-"+ 00

[(3x - 2) - (3x

lim

00

5) =

Se considerarmos as funções f(x) = 3x 2 -?x + 1 e g(x) = 2x

Temos, por hipótese

x++

+

[f(x) - g(x)]

(f - g)(x) = lim 00

x++oo

x++oo

x

(3x 00

e calculemos x++

(f

x-++

= +00.

lim

Demonstração

+ 00 e lim

00

-

7x

+

1) - (2x 2

+ 2x - 3)]

(x 2

= lim x++

-

9x + 4)

+00

00

81-H


81.

83.

Teorema Se

I) se

= + 00

f(x)

lim x"*+

e lim

g(x)

=

b

x~+ 00

00

então

b>O

(f. g)(x)

lim

"* O,

Teorema

Se

então

lim

=

f(x)

e lim

+ 00

=

g(x)

então lim

+ 00,

(f • g)(x)

=+

(f· g)(x)

= - 00.

(f· g)(x)

= + 00.

00.

X++OO

x++oo

::::;+00

x~+oo

11) se

então

b<O

(f· g)(x) = -

lim

00.

x++oo

84.

Teorema Se

lim

=

f(x)

e lim

+ 00

x++oo

82.

x++

g(x)

= -

então

00

lim x++oo

00

Teorema Se

X.:;.

+

= - 00

f(x)

lim

lim

e

g(x)

=

b

x~+oo

00

"* O,

então 85.

II se b>O então lim

(f • g)(x)

Teorema

= - 00

x~+oo

11) se

b<O

então

Se

(f • g)(x) = + 00,

lim

lim

= - 00

f(x)

e

lim

x++oo

= - 00,

g(x)

então

lim x++oo

x++oo

x++ oo

Observação Se

Iim

f(x) = + 00

(ou -

00)

e

x++oo

lim

g(x) = O,

Observação

x++oo

Se lim

onde 9 não é a função nula, então não podemos formular uma lei geral para lim (f • g)(x). x++oo

Por exemplo consideremos as funções f(x) em IR e a função g(x) =

definida em

x - 1

= 2x + 1

e h(x)

= x2

IR - {1 } .

lim

f(x)

lim x++

4 definidas

lim x++

(ou -

e lim

00)

= x

2

g(x)

=

+ 00

(ou -

00)

não podemos esta-

x++oo

lim

(..!.) (x). 9 Por exemplo, consideremos as funções f(x) - 4x + 3 definidas em IR. 00

= 2x

- 3, g(x)

= 3x

- 4 e h(x)

=

Notemos que

lim

(2x

+

1)

=

lim

+00

x++

x++

-

lim

4) = + 00

=

x++oo

g(x)

Iim

1 --=0 x - 1

x++

(2x - 3) = + 00

lim

=

(3x - 4)

lim

= + 00

X..,. + 00

x++ 00

00

00

f(x)

x++oo

(x 2

lim

g(x) = Iim 00

+ 00

x++

x++ 00

h(x) 00

=

=

belecer uma lei geral para -

Observemos que x++ 00

f(x)

~+oo

h(x)

=

(x 2

lim x++

00

-

4x + 3)

= + 00

00

mas mas 11m

(f. g)(x) = lim

x++oo

(h • g)(x)

Iim x++

82-H

00

[f(x). g(x)] = lim

x++oo

=

lim x++

[h(x). g(x)] 00

2x + 1 -- = 2

x++oo

= Iim x++ 00

X -

1

x2 X -

-

4 1

::::;:+00

f

x++oo

lim x++

I" Im

(-) (x)

lim

00

9 ( !l)(x) 9

x++

=

00

Iim x++oo

f(x) -g(x) h(x) g(x)

2x - 3

lim x++

=

3x - 4

00

lim X-1'-+oo

x

2

2

=3

- 4x + 3 3x - 4

::::;+00

83-H


86.

Teorema

Se

lim

Dados f(x); + 00,

então

x++ oo

Iim _1_; O. x++ 00 f(x)

f(x)

lim

+00

;

f(x)

-00

;

Teorema

f(x)

Iim

lim

f(x)

- 00,

então

x++oo

Iim _1_; O. x++oo f(x)

+00

;

x++

88.

f(x)

+00

;

f(x); O,

então

lim x++ OCI

X~ + 00

If(~) I; + 00.

f(x)

+00

;

:;

-

00

;

b*O

g(x) ; b * O

g(x)

lim

f(x)

;

+00

1

11m - ; Iim x++ 00 f(x) x++ 00

1 f(x)

x++oo

- 00

b>O b<O

lim

(f • g)(x)

;

[~:

se se

b>O b<O

(f • g)(x)

;

+00

;

- 00

(f • g)(x)

;

- 00

(f • g)(x)

;

+00

Iim

;

- 00

lim x++oo

.

1

11m --; O X++ 00 f(x)

+00

;

rIm

- 00

1

-;0

x++oo f(x) .

.

/1

I

11m - - ; +00 X-H 00 f(x)

f(x) ; O

lim x++oo

ou se existir N > O tal que para todo x> N tenhamos f(x) < O, então: Iim

se se

x++oo

; +00.

1 f(x)

(f • g)(x) ; {+oo -00

-

f(x)

lim

Se existir N > O tal que para todo x > N tenhamos f(x) > O, então:

x++oo

lim

lim

00

=

- 00

x++oo

g(x)

lim

;

x++oo

g(x)

lim

(f +g)(x)

x++oo

x++oo

Observação

. 1 -IIm x++ 00 f(x)

lim

x++

x++oo

lim

.

g(x)

lim

(f + g)(x) ; +00

x+-oo

X++ 00

f(x)

lim

lim

00

x++oo

x++oo

Se lim

-

x++oo

00

lim

Teorema

- 00

;

x++oo

lim

:;

x++oo

f(x)

lim

Iim x++oo

g(x)

Iim

x++oo

Se

+00

;

x+-oo

x++oo

87.

g(x)

lim x++oo

x~+oo

lim

Conclusão

Não podemos estabelecer uma lei para os seguintes casos: lim

f(x)

;

+00

lim

lim

g(x) =+00

f(x)

=-

00

lim

g(x)

;

- 00

(f - g) (x) ; ?

Iim x++oo

x++oo

x++oo

(f - g)(x) ;?

lim x++oo

x++oo

x++oo

-

lim

89.

Resumo

lim Faremos agora um resumo dos teoremas apresentados, lembrando que as proposições continuam verdadeiras se trocarmos o símbolo "x .... + 00" por "x ~ -00".

84-H

f(x) =+00

x++oo

lim

g(x)

;

- 00

f(x)

;

+oo(ou- oo)

lim

g(x)

;

O

;

+ 00 (ou - 00)

lim x++oo

00

lim

(f'g)(x);?

x++oo

x++oo

f(x)

(f + g)(x) ; ?

lim x++

x++oo

x++oo

x++oo

lim

g(x)

;

+ 00 (ou - 00)

lim x++oo

f (-)(x) 9

;

?

85- H


y

Exemplos

CA PÍ TU LO IV

1Ç») A fun ção f(x) = cos x é limitad a em IR, poi s -1" ;;; cos x ,.;;; " x E IR.

COMPLEMENTOS SOBRE LIMITES I.

90.

TEOREMAS AD IC IO NA IS SOBRE LIMITE S

x

y

2

3 2Ç») A fun ção flx ) = x + 1. não é lim ita da em IR mas é lim ita da no Intervalo [-1 ,1] poi s _2,.;;;x3 + 1"; ;;2 par

Fu nçã o lim ita da

x

1

a tod o xE [-1 ,1 ]

Definição

Dizemos qu e um a fun ção f, def ini da em A, é lim ita da em B C um nú me ro M > O A se exi stir tal qu e, par a tod o x per ten cen te a B, tem isto é, -M < f(x) < os I f(x li < M, M. Em sím bo los :

- 2

91.

Te ore ma Se lim flx )

f é lim ita da em B <= (3 M > O I x EB

=

If( x)!

=

x'"

< M)

lim ita da em

b,

en tão exi ste um int erv tal qu e f é alo ab ert o I co nte n d o a,

I - {a} .

Demonstração y

De vem os pro var qu e . se Iim f(x) = b en tão exi ste m M > O e Ó > O taiS x'" qu e se O < Ix - ai < ó en tão If( x)i < M. De fat o, se Iim flx ) d € = 1 na def ini ção de = b, tom an o lim ite , temoS:

M

x'"

€ =

x

1, 3 ó

>OI

O<

De cor re da def ini ção qu e, se f é lim ita da em B, en tão exi ste m tai s qu e, par a tod o a e b reais x E B, vale a < f(x } < b.

Ifl x) - b\

<

1

mas lf( x) - bl ;;;' \flx ) I -

-M

Ix - a I < ó = Ib\

po rta nto lf( x) I-l bl "; ;;l \f(x ) - bl < 1 = po nd o M = Ibl + 1, tem os 3M O, 3 ó > O I O < Ix - ai < ó

>

== lf(x)l,.;;; Ibl == \f\x}1 ,.;;; M

+ 1

86 -H 87 -H


92.

x++

Se lim f(x) X+a

~

b

*- O

g(x) ~ lim

"Se lim

Teorema da conservação do sinal então existe um intervalo aberto I contend o a, tal que

x++ 00

00

todo x E la, + oo[

I·Im f() x

_a

b , toman do e

~

Ibl

~

e ~ 2,3 6 > O I O < Ix - ai < 6 ~ b -

2

< f(x) < b

. - de I'Imites, . 2Ibl na d ef'IOlçao temos;

=

If(x) - bl <

Ibl 2

=

Ibl

11

+

g(x) ~ lim

x+a

x E I - {a},

~

lim h(x) x+a

~

b e se f é tal que g(x) < f(x) < h(x) para todo

onde I é intervalo aberto que contém a, então Iim f(x) ~ b. x.. a

h(x) ~ b, então para todo

e> O,

existem NI > O

00

e N2 > O tais que x> N ==> Ig(x) - bl < e = b - e < g(x) < b + e I x> N ==> Ih(x) - bl < e ~ b - e < h(x) < b + e 2 Sendo N ~ max {N , N }, temos para todo e> O, existe N >

==

2

x > N ~ b - E < g(x) ,,:;; f(x) ,,:;; h(x) < b + e b - e < f(x) < b + e = If(x) - bl < e

isto é,

lim

f(x)

x++ oo

~

O tal que

=

b

b I Ulr . mos "x Obs: O teorema continu a va'I'd I o se su st't la, + oo[ por l- 00, ar.

Sendo lim g(x) = lim h(x) x+a

94.

-+

+ 00" por "x ~ -

00"

e

Teorema Se Iim f(x) ~ b e lim g(x) = c, com b < c, então existe um intervalo x+a

Demonstração x+a

~

b então, para todo

e> O,

x+a

aberto I contend o a, tal que f(x) < g(x) existem 6 I > O e

8 2 > O tais que

em

I - {a}.

Demonstração

0<lx -al<8 1

==

O < Ix - ai < 8 2

==

Ig(x)- bl<e ==*b -e<g (x)<b +e Ih(x) - bl < e ==* b - e < h(x) < b + e

Sendo 8 ~ min {8 I, 8 2 }, temos para todo e > O, existe 8 > O tal que O < Ix - ai < 8 b - e < g(x) ,,:;; f(x) ,,:;; h(x) < b + e ==> b - e < f(x) < b + e If(x) - bl < e isto é lim f(x) ~ b x"a 88-H

x++

00

Teorema do confron to Se Iim g(x)

==

x++

I

Se b > O, então, para todo x tal que O < Ix - ai < 6, temos Ibl b b f(x) > b - 2 ~ b - 2" ~ 2" > O = f tem o mesmo sinal de b. Se b < O, então,p ara todo x tal que O < Ix - ai < 6 temos Ibl b . f(x) < b + 2 ~ b -"2 2"b < O = f tem o mesmo 'slOal de b.

93.

f(x) ~ b".

Demonstração

Sendo lim

Ibl

x+-+- oo

I - {a}.

Demonstração

Sendo

então Iim

'

f conserva o mesmo sinal de b em

h(x) ~ b e se f é tal que g(x)":;; f(x) ,,:;; h(x) para

==

==

c-b df'-dl' Sendo lim f(x) = b e lim g(x) = c e tomand o e = -2na e Inlçao e Ix+a

x+a

mites, temos que existem 8 I > O e 6 2 > O tais que c - b 3b - c O < Ix - ai < 8 1 = If(x) - bl < -2-~---Y- < f(x) <

O < Ix - ai

< 82

c - b b + c < g(x) < =* Ig(x) - cl < - 2 - = -2-

b

+ C

2

3c 2- b

89-H


96. 3ô>OIO<lx-al<ô =>

b + c 2 <

==> f(x) <

g(x)

Teorema

=

~

lim cos x

cos a, 'lfa E IR

x+a

f(x) < g(x)

A demonstração deste teorema,que é feita de modo análogo à do anterior,fi· cará como exercício.

11.

97.

LIMITES TRIGONOMI:TRICOS

Teorema lim tg x

95.

x+a

Teorema

~

'Ir

tg a, 'lfa 0/= -2 + k'lr, k E

~

Demonstração lim sen x x+a

~

sen a, 'lfa E IR ~

lim tg x x+a

. sen x 11m - x+a COS x

lim sen x x+a

~

."....--Iim cos x

sen a cos a

tg a

x+a

Demonstração Para demonstrarmos que

lim sen x

~

sen a demonstremos que

x+a

Iim (sen x - sen a) x+a

~

O,

já que

98.

lim sen x x+a

~

sen a

~

Teorema (limite trigonométrico fundamental)

Iim (sen x - sen a) = O. x+a

. sen x IIm - - =

Temos, da trigonometria, O.;;; Isenx-senal

=

x-a x+a 12 sen - 2 - ' cos -2-1

x-a I x-ai e Isen -2-1.;;; 2

mas

x+o

=

x+a x-a 12 cos -2-1. Isen -2-1

Demonstração Da trigonometria, temos

x+a 12 cos -2-1 .;;; 2

O Isen x - sen a I .;;; ==>';;;

Ix - a I

'Ir

a) O < x <

"2

1 sen x

>

então 2 O .;;; Isen x - sen ai.;;; 2l x-al

X

~-­

..

=lo

x

sen x < x < tg x

>

tg x

=

(I)

Considerando as funções g(x) = O, f(x) = Isen x - sen a 1 e h(x) = Ix - a I e notando que Iim g(x) = lim O = O x+a

X+a

b)

Iim h(x) = lim Ix - ai = O x+a

x+a

lim Isen x - sen ai = O,

9O-H

< x < O ==> sen x > x

x-;.a

Segue-se pelo teorema do confronto que lim Isen x - sen ai = O e, portanto, x+a

'Ir

-"2

ou seja,

~

sen x

< -x1 < -tg1x

> tg x

==O-

(11)

lim sen x = sen a. x+a

91-H


Multiplicando as desigualdades

a) o < ~

2" >

1T

b) -

sen x > x

<x<O

2 ~

sen x sen x >

(sen x> O)

1T

x <

sen x > x

resulta

sen x 19 x

sen 3x b) 11m x+O se" 5x

==

1T

<x<

2

1T

2

"*

O

sen x > x

sen x cos x < - - < x

g(x) = cos x, f(x)

Considerando

lim g(x) = lim cos x x~o . x..,.b

sen x x

sen x tg x

==

lim

X

=

x+O

f)

b) lim x+O

sen 2x sen x

g) lim x+o

c) Iim x+O

sen ax bx

hl

lim

j)

lim

e

h(x)

sen ax

e notando que

1

1

3

=5"

( (sen x)2 " __1_ _ ) 1 + COS x x2

1

=2"

j}

3'i<"

tg ax bx 1 - cos x

x

1 -sec x x2 x+O

x+O

tg 2x

e) 11m x+O

lim

x+o

x+O sen bx

pelo teorema do confronto, temos x+o

lim

sen 3x 2x

d) lim

H.91

x+o

sen x

(1 - cos xl(1 + cos x) x 2 • (1 +cos x) x+o

11m

a) 11m x+O

1

cos O = 1

lim h(x) = lim 1 = 1 x+o

(.:!. • sen 3x "~) 3 sen 5x =5"" 5 3x

H.90 Encontre:

sen x sen x >

cos x

e X

x+o

x2

Temos, portanto: para

11m

=

1 - cos x

c), lim x+o

cos x

(sen x < O)

sen x > x

1>

CD e ® por sen x,

tg x + sen x x

- cos x x+O x • sen x

lim

Encontre: Ii m x+a

~s.:.e,,-n-,-x,----,-s:..:e-,-n,-=a

x - a

1

Solução Da trigonometria, temos:

sen x. - sen a

=

2 sen

x - a 2

x +a

• cos - 2 -

então EXERCICIOS lim

x - a

x+a

H.89 Encontre:

a) 11m x+o

se" x - se" a

1 • cos a

=

cos a

2

1 - cos x x2 x+o

H.92

Encontre:

a) lim x+a

Solução

92-H

x+a

t_

c} lim

sen 2x x

lim

x - a x +a . cos 2 2 x - a

se n x - a ~ _ _--,2=-_. cos _x_;_a = lim x-+-a,\ x - a

sen 2x x

sen 3x b) 11m x+o sen 5x

ai lim x+o

2 sen

(2· sen 2x ) 2x x+o

11m

2 •1

2

c) lim x+a

cos x - cos a x - a

secx-seca x - a

b) lim x+a d) lim

x..,.

I

tgx-tga x - a

sen x - cos x 1 - tg x

93-H


e) 11m X~O

g) 11m x~o

II 11m

tg x - sen x sen 2 x

x~o

p) 11m

x

-~~.e~i)x 2

t)

1 - x

lim

<

e

1,

<

Ioga (1 - e)

então

< aX <

e <= 1 - e

1 + e <=

< O e Ioga

(1 + e) >

O

Assim, para todo O < e < 1, existe 8 ~ min {Ioga (1 + e), -Ioga (1 - e)} Ia X - 1I < e o ~

1,

tomamos

e'

<

1 ,,;;; e

e determinamos

tal que

O

Deixaremos a carga do leitor a demonstração para o caso

x2

vi 1 + sen x - vi 1 -

tal

O< Ix I < 8 =

8' = min {Ioga (1 + e'). -Ioga (1 - e')} 0< Ixl < 8' = la x - 11 < e' < e

- cos x

x+O

O<

Se a > 1 e e

x

x~o

11x cos 2

que

cos ax - tos bx

ri 11m

1 temos:

Ioga (1- e) < x < Ioga (1 + e) <=> X < Ioga (1 + e) e -x < -Ioga (1 -e) <=> <= Ixl < Ioga (1 + e) e Ixl < -Ioga (1 - e)o

1 - cos 3 x sen 2 x

x~O

x - sen 2x x + sen 3x

x~1

mas se a> 1 e e, portanto:

1 - x2

n) lim

sen ax - sen bx

s) lim

x

Ilm--x-H sen 1TX

cos 2x cos x - sen x

x~o

x~o

I)

11 - 3x

X+~

sen(x+al-sena

1

<

la x - 11 < e <= - e < a X - 1 <= Ioga (1 - e) < x < Ioga (1 + e)

~~---~--

1 - 2 cos x

m) 11m

q) 11m

~

/i) 11m

Supondo a > 1 e O < e

sen 3x - sen 2x sen x

x~o

x

f

o) 11m

h) 11m

cos(x + ai - cos a

k) lim

11m x~o

cos 2x - cos 3x x2

x~o

x.:,.

f)

<a<

1o

sen x

x

100. Teorema Encontre:

H.93

a) 11m x • sen x~o

b) 11m x • sen x++oo

Se a E IR e O < a

c) lim

x

* 1,

então lim a X

d) 11m

x

cotg 2x • cotg

11 ("2 -

x)

=

abo

x~b

x~1

Demonstração

x~o

Iim a X

Para provarmos que

=

ab

provemos que

lim

x~b

(a x - a b )

00

x~b

lim a x - b =

Provemos inicialmente que

isto é:

x~b

111.

LIMITES DA FUNÇÃO EXPONENCIAL

Ve

> O, 8 > O I O <

Fazendo 99.

Ve

Teorema

Se a E IR

e O

<a

* 1,

lim a X

1,

94-H

::3

w,

=

la x - b

-

11

< e

temos:

> O, 8 > O I O < Iwl < 8 =

Mostremos agora que

x+O

la w

-

11

< e

lim (a X

-

a b ) = O.

De fato:

x~b

lim la x - a b ) = lim [a b • (a x - b - 1)] = lim a X

x~b

=

1 devemos provar:

8 > O I 0< Ixl < 8 =

la x - 11

x~b

b = a b • Iim (a x - b - 1) = a b • [Iim a x - b - 1] = a • [1 - 1] = a b • O = O

x~o

Ve > O,

=

< 8

que é verdadeiro pelo teorema anterior. então

Demonstração

Para provarmos que

x - b

Ix - bl

< e

x~b

x~b

95-H


101. Teorema Se

2) Dado

aE IR

e a>1,

então

aX

lim

+00 e

aX

lim

O.

x+-oo

x~+oo

O < f2 < 1,

O < Ix - bl < 02

=

aX

lim x++

=

a

X

> M aX > M

M > O temos

=

x+-oo

x < N

Notemos que x X la I < f a < f

=

=

=

laxl < f

então existe o = min {o 1, 02} tal que

l)SeO<E<l,

x > 109aM.

Se M > 1, tomamos N = 109aM > O e temos que, para todo M> 1, existe N = 109aM > O tal que x > N = a X > M. Se O < M < 1, tomamos M' > 1 > M, determinamos N = 109aM' > O e temos que, para todo M < 1, existe N = 109aM' > O tal que x> N ~ a X > M. Para provarmos que lim a X = O devemos provar:

'tE> O, 3 N < O

O<lx-bl<o = ~ 1 - f < af(x) < 1 + f

que

109a(1 - f) < f(x) < 109a(1 + e) = = - f < af(x) - 1 f ==<> laf(x) - 11 < E

<

2) Se E > 1, então tomamos O < f' < 1 < E e existe o' > O tal 0< Ix-bl <o' = laf(x) -11 <f'<E

Assim provamos que lim af(x) = 1 para a> 1, deixamos a car90 do leitor a x+b demonstração para O < a < 1, que é feita de modo aná109 0 .

< 109af

X

Se O < f < 1, tomamos N = 109af < O tal que x < N = I a X 1 < f. Se f> 1, tomamos f' < 1 < f, determinamos N = 109af' O e temos que, para todo f > 1, existe N = 109af' < O tal que x < N 1a X I < f' < f.

<

=

104. Teorema Se

a E IR,

O < a =1= 1 e lim f(x) x+b

f(x)

lim

lim af(x) x+b

102. Teorema Se a E IR

temos:

109a (1 - E2) < 0< 109a(1 + fd Então, para todo f > O, temos:

+ 00 devemos provar

00

't M > O, 3 N > O 1 x > N Notemos que para todo

=

-109a(1 - E2)

EI > O e O < E2 < 1,

Notemos que, para

Para provarmos que

=

lf(x)1 < -109a(1 - f2)

=109a(1 - E2) < f(x) <

Demonstração

existe 02 > O tal que

e O < a < 1,

então

aX = O e

lim x++

lim

a X = + 00.

x+- 00

00

A demonstração deste teorema ficará a car90 do leitor como exercício.

=

c,

então

= a C.

= a x+b

Demonstração Por hipótese, temos

lim f(x) = c, x+b

isto é,

lim [f(x) - c] = O. x+b

Pelo teorema anterior 103. Teorema

=

lim [f(x) - c] = O

Se a E IR, O < a

"*

lim a[f(x) - c]

x+b

1 e lim f(x) = O,

então

x+b

lim af(x) x+b

x+b

1. Para provarmos que

Iim af(x)

= aC

provemos que lim [af(x) - aC]

pb

= O.

pb

Demonstração Considerando que

Iim f(x) x+b

1) Dado

existe

<

EI > O,

=

=

O e supondo

OI > O tal que

O < Ix - bl 01 If(x)1 < 109a(1 +fd = -109a(1 + fi) < f(x) < 109a(1 + fi)'

96-H

=

a > 1, temos:

Então lim [af(x) - aC] = lim a C • [af(xl-c - 1] = x+b x+b = aC lim [af(x)-c _ 1] = a C [Iim af(x)-c - 1] x+b x+b =

aC

(1 - 1)

=

aC

O

=

O

97-H


EXERC(CIOS

IV. LIMITES DA FUNÇÃO LOGARliMICA

H.94 Complete:

a) lim 3 x =

c) lim

x~2

105. Teorema

eX =

x~2

(.2.)x 2 x+-]

b) lim

=

Se

I~)x =

d) lim

a E IR

e 0< a

* 1,

então

e

x~3

lim (l09aX) = O. x~1

Demonstração H.95 Complete:

a) lim

2

x

=

x

=

di lim

x~+oo

b) lim

x+-oo

2

Para provarmos que

1.2.)x _ 3 -

V x e

e) lim

~-oo

x++oo

(.2.)x x++oo 3

c) 11m

=

f)

eX

11m

H.96 Complete:

a) 11m 22X2 - 3x+1 x~3

=

x+-oo

cI lim e

<=*

3x + 2 x - 1

<=*

x~o

3x2+6X+ 2

b) lim

x+-2

di lim

10

H.97 Complete:

a) lim 3 x~2

1 - x2 x- 1

2

x~1

c) lim x~

x~2

(.!- )

b) 11m

2 I

dI lim

(.!- )

<=*

e) lim e.;x - 1

> °I

3 8

>1

1I09axl < E

<=*

°< E

=

1x - 1 i < 8

> 0,

1I09aX I <

E

temos que

-E < 109aX < E

<=*

a-E < x < aE

°

<=*

>

a - 1 mas a-E -1 < e aE - 1 0, portanto: E E a-E - 1 < x - 1 < a - 1 <=* X - 1 < a - 1 e 1 - x < 1 - a-E <=* 1x - '11 < aE - 1 e Ix - 11 < 1 - a-€ Assim, para todo E > 0, existe 8 = min {a E - 1, 1 - a-E} tal que

°<

E

=

lIo9axl <E

a < 1 e

> 0,

temos que E < 109aX < E <=* a E < x < a-E <==>

E

1I09axl < E <=* E a - 1 < x - 1 < a-E - 1 mas aE - 1 < e a-E - 1 0, portanto: E a - 1 < x - 1 < a-E - 1 <=* X - 1 < a-E - 1 e 1 - x < 1 - aE <=* 1x - 1 I < a -E - 1 e Ix - 1 I < 1 - aE Assim, para todo E > 0, existe 8 = min {a- E - 1, 1 - aE} tal que

°

1x - 1 1 < 8

=

>

1I09aX 1 < E

106. Teorema

x~1

x 3 - 3x + 2 x 2 + x -2

e

a-E - 1 < x - 1 <

°<

3

X- 1

> 0,

Supondo

<=*

x 3 - 6x 2 + 11 x - 6 x 2 - 3x + 2

E

Supondo a

0< Ix -11<8

4x 2 +6x - 2 3x + 4

x+-2

x 2 -4 x-2

lim (109aX) = O devemos provar x~1

Se

x 2 - 5x + 4 fI lim (.!- I JX - 2 e ~4

a E IR

e

° < a * 1,

então

lim (109aX)

109ab onde

x~b

b

> O.

Demonstração Para provarmos que lim (l09aX) "b =

= 109ab provemos que lim (109aX - 109ab) = "b

O. Provemos inicialmente que

lim (l09a ~)

=

0,

isto é

x~b

V

98-H

E

> 0,

3 8

> ° 1 ° < Ix -

b 1< 8

<=*

1I09a

t

I< E

99-H


Fazendo

x

b;

isto é,

w,

x; bw

Ix - bl; Ibw-bl ; Ibl. Iw - 11,

108. Teorema

que

Se

temos:

I~I

\>'E>0,31)'>0 10<lw-ll<

e notando

e O < a < 1, então lim x++

e

=1)'

lim

(Iogax) =

-00

e

00

(logax) = + 00.

x+o+

que é verdadeira pelo teorema anterior. Mostremos agora que

a E IR

A demonstração deste teorema, que é feita de modo análogo à do anterior, ficará a cargo do leitor.

Iim (Iogax - logab) = O. x .. b

De fato: x lim (logax - logab) = lim (Ioga -b) = O. x.. b x.. b

109. Teorema Se

a E IR,

*-

O< a

1 e

lim f(x) x.. b

1,

então

lim [Iogaf(x)] = O. x.. b

Demonstração

107. Teorema Se

a E IR

e a> 1,

então

(loga x ) =+00 e

lim x++oo

lim (loga x ) =

-00.

x+o+

Considerando que

lim f(x) = 1 e a > 1, x.. b

1) Dado

existe

EI

> O,

=

O < Ix - bl < 1)1 ~ 1 - aEI < f(x) - 1 < aEI

Demonstração

-

I) I

> O,

temos:

tal que

=

If(x) - 11 < aEI - 1 1 = 2 - aEI < f(x) < aEI

Para provarmos que lim

2) Dado Ez > O,

+00

(loga x )

devemos provar

=

\>' M > O, 3 N > O I x > N Notemos que, para todo

logax > M.

M > O,

temos

logax > M

<=;>

x > a M.

Assim, tomando N; a , temos que para todo M > O existe N = a M > O tal que

=

Para provarmos que

lim

logax > M

(logax) = -

tal que

00

devemos provar

;>

a- Ez _l

<

Notemos, que para EI > O e EZ > O, temos O < a- Ez < 1 < aEI , então, para todo E > O, existe I) ; min {I) I, I)z} tal que

M

x > N

existe I)z > O,

0< Ix-bl < I)z ==> If(x) -11 < 1 _a- Ez < f(x) - 1 < 1 - a- Ez ;> a- Ez < f(x) < 2 _a- Ez

x++oo

;>

O < Ix - bl < Ilogaf(x) I < E

I)

=

a-E < f(x) < aE

= -E<

log a f(x) < E==>

Com isso provamos que lim [Ioga f(x)] ; O para a > 1. Deixamos a cargo x.. b

do leitor a demonstração para O < a < 1.

x+o+

\>' M < O, 3

I)

> O I O< x <

==> logax < M

I)

Notemos que logax < M Assim, tomando

I)

<=;>

x < aM

= a M , temos que para todo M < O existe li = a M > O tal

que O< x <

100-H

I)

==> logax < M

110. Teorema Se a E IR, O < a

*-

1 e lim f(x) ; c > O, x.. b Iim [Iogaf(x)] = Ioga [ lim f(x)]= logac. x.. b x.. b

então

101-H


Demonstração

V.

Por hipótese, temos

lim f(x) = c,

isto é,

Iim f(x)

x+b

1.

c

x+ b

LIMITE EXPONENCIAL FUNDAMENTAL

111. Teorema

Pelo teorema anterior, f(X)] =Iim [ 109a -c- = O

lim f(x) = 1 x+b

C

Na função

f(n) = (1 +

x+b

Para provarmos que

lim [109a f(x)] = 109aC,

*

)n"definida em

Jl'

temos:

~ t ~__---

• li (1) f é crescente em til' (2) 2 .;;; f(n) 3, \f n E N (3) existe lim f(n). _..• _._+-~._--

provemos

<

x+b

lim [109a f(x) - logac] = O.

'fi

n++ 00

x+b

Temos:

,

Demonstração de (1)

lim [109a f(x) - 109aC] = Iim

~09a

x+b~

x+b

f(X)] = O

(1 + -~)n

Desenvo Iven do

C

pelas fórmulas do Binômio de Newton Iveja no

n

livro 5), temos: f(n)

EXERCíCIOS

= (1

n 1 n 1 n 1 n 1 + ( ) ' - + ( 2 ) ' - +2( 3 ) ' - 3 + " ' + ( ) 1 n n n n

+-l)n n

H.98 Complete:

Qn x

a) 11m 1093x = x+2

c) 11m x+e 2

b) Iim log 1 x = x+4 "2

d) 11m log x X+lOOO

~

f(n)

=

H.99 Complete:

b) I Im x+-+oo

c) lim x++oo

d) 11m 10go,I x x++oo

log I x ~

e) 11m

Qn x =

f)

2'

Qn x

=

b) 11m Qn (3x 2 x+3

+ 5)

+ 4x - 2)

=

n(n - 1) 21

• -2

1 n

temos: +

n(n-1)(n-2) 1 • -n3 + ... + 3!

n(n-1)(n-2) ... 2 · 1 1 I • -nn n.

+~ 21

(1 _.!.) n

2

1

1

1

+~ 31

(1

-~) n

(1

-~) + ... n

+

n-1

Indicando

6x + 2 cl lim log 4x + 3 =

1

(1 - - ) n

3x 2 - 5x + 2 d) Iim log I 2 x+4 "2 2x - x + 2

2

(1

(1 - - )

n

-~) n

0'0

(1

-~) n

• (1

_..!-) n

temos

+ 3x + 2 + 5x + 4

x - x3 b) 11m log - 2 x+O x + x

102-H

1

. -;- +

i';;; n,

+ ... +- (1 --)(1 --)(1 - - ) ... (1 - - I nl n n 3 n

H.l0l Complete: x2 1093 x2

+ 11

f(n) = (1 +.!.)n = 1 + 1 n

11m log I x = x+o+ 7:

x+3

=

n

1

para

ou seja:

x+O+

a) 11m 1092 (4x 2 - 7x x+-l

=

=

H.l00 Complete:

a) Iim x+-l

1 n (1 + -;;- )

+ ... +

a) 11m 1092x ~ x++oo

n! i! (n - i)!

lembrando que

=

'r;n

~

c) 11m x+3

Qn

d) 11m log x+-2

x - 3

v'X+T -

2

3-~ ~-2

=

n-I 1 f(n)=2+L(i+1)! i~1

I

n

j ~I

j (1 - - ) n

103-H


(1 + _1_ )n+1 n+1

Desenvolvendo de modo análogo f(n + 1)

1

n

f(n+1)~2+L

j=l

Para provarmos que

>

f(n + 1)

1

n~

(1 - n

f(1) = (1 ++)1 = 2

f(n)

.

~ fi+iTf j~I,

a)

Considerando que em (1) provamos que f é crescente em N *, segue-se que f assumirá o menor valor para n = 1, então

(1 - - j - ) n+ 1

11

(i+1)!

i=l

~ 1) >

Demonstração de (2)

encontramos

devemos provar:

portanto f(n);;' 2 para todo n E N* Provemos agora que f(n) < 3 para todo n E N* Notemos que, para todo j E N, 1"; j .,; n - 1, temos:

t; n~

n-l

j

1

i

1 -n

j

-»O n +1

fi ( 1 - -

b)(n+1)!

j=1

e, para todo (1

n

<1 =

j=1

i E N,

+ i)! ;;.

i

j n+1

==>

<

n j

j=1

n+ 1

1

n-]

L

(1 n-I

II

j=1

1

n

(1 + i)!j=1

1

.

(1 _ _J_)

n-]

>

n+1

n-I

L

j=l

j (1 - - ) n

1

1 - (n + 1)! 1 (n+1)n+1

104-H

(1+i)!

'" 2'

n-1

1

<~

n

2'

==>

L

1

(1+ij!

1=1

n-1

j~]

.

(1 -

~) <

n-]

L

temos: 1 2i

1=2

é a soma dos termos de uma progressão geométrica, portanto.

1 (1

+ i)!

1 1 22 +

23

1 + ... + 2n- 1

i=l

n-I

11

1

2' = "2 +

(1

)',.1

.

logo

n

1 I : (1:i)!nri ( 1 - 1 ) < I : ; i =1 j=1 n "=1 "

_1.)

< 1'"*f(n)=2 + I : j;o;l

~il(1+1)<3 n

1 (1 + i)! H

Demonstração de (3)

n

fi (1

J'

-

n+1

1

) =

n

fr1-t1TI j~I,

(n+1- j ) n +1

(n + 1)n O pois

11 (n + 1 - j)

j=1

nEN*

(n + 1)!

(n + 1)n

Considerando que f é crescente e limitada em

N *, seja L, 2"; L < 3 tal

1l?) f(n) < L para todo n E N* 2l?) se f(n) < K para todo n E N *

K;;' L

que:

n

>

(')

1

---,;::~

1 2;

n-1 1

L

==>

Prova de b

(n + 1)! H

temos:

i=l

> - (+')111 1 1 . J=1

n+ 1

L n

Mas

1

1

i E N, j E N, 1 .,; i .,; n - 1 e 1 .,; j .,; n - 1,

1 n-I j - - 11 (1 --I

==>

==>

n

---I

(1+i)!j=1

i"'l

n

_1)

(1

j

n j=1

+ i)!

j

=1---> n+1

.

> n

n-l

1 (1

n-l

portanto, para todo

1"; j .,; n - 1, temos:

e

>

=-n+ 1

11 (1 - - - I

==>

=

j

n -I

=

j E N

<

1"; i .,; n - 1,

Prova de a Notemos que para todo

.

J (1 - - ) n

então

• n!

( *)

Fica como exercício provar por indução finita que (1 + i)!;;' 2 i , \f i E N*

105-H


Mos trem os que

Iim f(n) = L. n++oo De fato , para todo € > 0, exis te nl E N', tal que f(nl ) > L - €. Tom and o M = nl, tem os para todo € > O e n > M L - € < f(nd < f(n) < L < L +.€ isto é, para todo € > O, exis te M > O tal que n> M If(n ) - LI < €

Mas:

n~+oo

=

lim n++ oo

Cha mam os de e o limi te da funç ão f(n) = do n tend e a + 00 •

(1 +2) " n

~',

defi nida em

quan -

2) lim n+ + 00

=

lim n-++

(1 +

n++o o

2)"

OO

e

1 )" (1 +n

n++ oo

lim x++o o

(1 +.2) X

x) ,

X

n<> ;x< n+1

==> -

1 n

~

1 x >

n+1

a + 00,

ou

x

>

O}

por

X

Faze ndo x = - (w + 1) e nota ndo que se x tend e a tem os:

lim

1

=1+-~

1

+1

n

1 1+->1+

x

x+

_00

(1 +.2) X = lim (1 x w++oo

lim w++ oo

Con side rand o que

+ 1

e

Demonstração

+--

(1 + __ 1 _ )" <

x+- oo

e.

Seja m n e n + 1 dois núm eros inte iros posi tivo s e cons ecut ivos . Para todo x tal que n <>; x < n + 1 tem os:

n

(1 + 2)x x

Seja f a funç ão defi nida em {x EI Rl x< -1 f(x) = (1 + 1 x (1 +...!.)X = e. lim enta- o

Demonstração

n

e

114 . Teo rem a

Seja a funç ão f(x) = (1 +2) x defi nida em {x E IR I x < - 1 ou x> O}, enx 00

+~)J

entã o pelo Teo rem a do Con fron to, tem os

n

113 . Teo rem a

x++

• (1

1 (1 + -) = e • n

lim n-++ oo

núm ero e é um núm ero irrac iona l. Um valo r apro xim ado de e é 2,71 828 182 84.

lim

[(1 +~)"

Iim

o

tão

e

(1 + - - ) 1+n

(1 + 2 )"+1 n

lim

=

n++o o

,

n++o o

(1 + _1_ )"+ 1 n+1 1 +-n+1

= lim

(1 + _1_ )"+1 n+1

lim

112 . Def iniç ão do núm ero e

e

(1 + _1_ )" n+1

1) fim

(1

( ~ )-(W+I) W

=

+1

W

1_)_ (w+ I) +1

00

entã o w

tend e

=

lim (w + 1 )W+I w.++oo W

n <>; x < n + 1, resu lta: +.2 )X < (1 +.2 ) n+1 x

n

lim w++ oo

lim w++ oo

1 w

(1 +- )

e • 1

e

106 -H 107 -H


Notando que, se x tende a zero, então w também tende a zero, temos:

115. Teorema

.

Seja a função definida em

{x E IR I - 1

< x *-

aX

1

-

11m - - O} por

f(x)

x

HO

~

w • Qna

lim

w.. O Qn(1 +w)

~

Qna· lim w.. o

1

2.- Qn(1

+ w)

~

Qn a Qn e

w

1

+ xIx ~ e.

lim (1

então

Qn

X.. O

a ·Iim w ..

Qn

o

Demonstração Fazendo

x ~-

1

l

(1 + xIx

obtemos

y

a

1

...!..-

Qn(1+w) Vi"

Qn[lim (1+w)W] w+o

~

Qn a -1-

Qn

a

+ 2.-)Y e notando que

(1

y

EXERCICIOS H.102 Calcular:

a) 11m

temos

x++oo

l

( 1 + xIx ~ lim

lim x" 0+

Y70+00

! ( 1 + x)X

lim

1 ( 1 + -)Y Y

e

( 1 +.2-)Y Y

e

lim

x+o-

y.::,._OO

b) 11m x+- oo

Solução a) lim

x

x++oo

portanto l lim (1 + xIx

bl Fazendo

e

~

(1 +.!..)2X

w

11m x++oo

[(1 +

~)x]

2

~1 +~ IX] x++oo

x temos: 3

+~)x = 11m

--O<)

x

> O,

então

lim

x.. o

~ x

bl Iim

Demonstração

lim x+O

aX

~ -

c) lim

1 temos

x++oo

1x - 1 lim - - -

1

x+O

X

Supondo O

< a*-1

X

~

lim O

~

O

~

e fazendo aX

Notemos que

.1 11m

[(1 +

~)w] = .3

-

1 ~ w, temos:

x

w

Qn a Qn(1 +w)'

.~---"----"--

34" =

(1 +-)

x

x+-oo

(1 + .!..)X+2 x

f)

(1 +~)x x

g)

d) 11m (1 +2)3X x x+-oo

Qn

x+o

11m x++oo

=

11m x+-oo

=

(1 +-!.I x x (1 +~)bx x

= = =

hl 11m

(_x_)x

cl 11m

(1 _ .!..)3X

=

_2 )2X

=

X++~

x

+1

H.104 Calcular:

Qn(1+w) Qna

108-H

--O<)

Qna. x+-oo

Para a

w

3

= 11m

x

=

(1 +.!...) 3x x x++oo

a) lim

a

(1 +~ )3W

H.103 Calcular:

116. Teorema Se

= e~

=-

x.. o (1 lim x+-oo

2

= 11m

a) Iim

(1 _.!..)x

x

x++oo

b) lim x+-oo

(1

=

_~)x = x

x+- oo

(1 d) lim x++oo

x

x

109-H


H.10S Calcular:

11m x++oo

I~)x x - 1

Solução

lim

lim x++oo

x++oo (

(1 + 2. l x x

X + 1)X

__x_

X:

1

11m

_~Ix

X++OO (l

x

lim x++oo

(1 + 2.)x x

(1 _ 2.)x x x++oo lim

=,= o

02

o

CAPÍTULO V

H.l06 Calcular: a) 11m x++oo

b) lim X+-(X)

cl Iim x+- oo

( x + 4)x x - 3 ( x + 2 )x x+1

x++oo

(x-4 )X+3 x- 1

o) 11m x++oo

( x 2 + 1 )x2 x2 - 3

di 11m

CONTINUIDADE

' x - 3 IX \ x +2

I.

NOÇÃO DE CONTINUIDADE

'H.l07 Calcular: (3x+2)2X x+-oo 3x - 1

a) lim

cl 11m

x++oo

1 2x-1 IX x+-oo 2x + 1

bl 11m

I.

H.l0S Calcular: aI 11m "'o b) Iim

2x o - 1 x 2 3X _ 1 x

x+o

di

x o)

11m o x72

2 - o x - 2

x

a

fllim~ x+a

e 2X _ 1 cl Um 3X _ 1 x+o e

117. Definição

g)

.

x - a

2 x _ 2a

11m - - x+a x - a

3 2X _ 1 sX - 1 x+o 2

Seja f uma função definida em um intervalo aberto I e a um elemento de Dizemos que f é contínua em a, se lim f(x) = f(a),

Notemos que para falarmos em continuidade de uma função em um ponto é necessário que este ponto pertença ao domínio da função. Da definição decorre que se f é contínua em a então as três condições deverão estar satisfeitas: 10) existe f(a) 2C?) existe lim f(x) x+a

3C?)

lim f(x)

=

f(a)

x+a

lim

118. Definição

H.l09 Calcular: Qn 11 + x)

a) II m -'-'--'-" x x+o

b) j1m log(1 + xl x+O

x

H.ll0 Calcular: 11m x+o

Vl -2x

c) lim x+o d) lim

x+o

Qn(1 + 2x) x

log(1 + 3xl x

Seja f uma função definida em um intervalo aberto I e a um elemento de I. Dizemos que f é descontínua em a se f não for contínua em a. Observemos também que para falarmos em descontinuidade de uma função em um ponto é necessário que esse ponto pertença ao domínio da função. Da definição decorre que, se f é descontínua em a, então as duas condições abaixo deverão estar satisfeitas 1C?) existe f(a) 2C?) não existe lim f(x) ou lim f(x) f(a) x+a

x+a

*

110-H 111- H


119. Definição Dizemos que uma função f é contínua em um intervalo aberto se f for

2?)

A função

contínua em todos os pontos desse intervalo. f(x) definida em pois

120. Definição Seja a um ponto do domínio da função f. Dizemos que f é contínua à direita de a se

lim

f(x) ~ f(a) e dizemos

~ {2X4+ 1 :: : '* ~

IR

é descontínua em

lim f(x)

lim (2x

x~l

x-H

+ 1)

3'* 4

1, f(1)

x~a+

que f é contínua à esquerda de a se lim x+a-

x

f(x) ~ fIa).

121. Definição Dizemos que uma função f é contínua em um intervalo fechado la, b) se f for contínua no intervalo aberto la, bl e se também for contínua em a, à esquerda, e em b,

Observemos que f é contínua em IR - tl} pois para todo temos: lim f(x) ~ lim (2x

x+a

à direita.

+ 1)

x+a

~ 2a

+1

a EIR - {1},

~ f(a)

y

122. Exemplos

3?)

A função f(x)

~

{x + 1

se

x

1 - x se

x

-<

>

1 1

definida em IR é descontínua em 1, pois lim

f(x) ~ lim

x+}-

1?)

A função f(x) ~ 2x

+ 1 defi-

e lim

nida em IR é contínua em 1, pois lim f(x) x-H

~

lim (2x + 1)

~

3

~

f(x)

lim

x-H+

f(l)

(x + 1)

2

X+!-

(1 - x)

~

O

X+l+

portanto, não existe

x+l

lim f(x), x-H

x

Observemos que f é contínua em IR - {1} pois para todo a E IR - {1} , lim (1 - x) ~ 1 - a ~ f(a) temos: se a 1, então lim f(x) x->a x->a

>

Notemos que f é contínua em IR, pois para todo a E IR, temos lim f(x) ~ lim (2x + 1) ~ 2a + 1 ~ f(a) x+a

112-H

se

a

<

1, então

lim f(x) x->a

lim (x x->a

+ 1)

a

+ 1

f(a)

x-;.a

113-H


y

EXERCICIOS H.111 Verificar se a função definida por

4<:'1

Na função f(x)

,

~~ x

I{x) ~

defini-

da em IR não podemos afirmar que f é descontinua em x ~ 0, pois x ~ não - - - - - - - - - j - - - - - - - -

°

x

pertence ao dominio da função. ---

2 - 1 {X7-2x

é contínua em

=

11m

7 - 2 •2 flx) ~ lim

então

Observemos que

é continua em se se

a a

> 0,

< 0,

{1-1 ,

IR

se se

> <

° °

x x pois, para todo

então então

logo

a E IR', ~

lim f(x)

lim 1

x+a

x+a

lim f(x)

lim (-1)

x+a

x+a

1

~

~

flxl

~

lim

lim I{xl = 3 x+2

=

fia)

ai f(xl

=

g

se se

x + 2 {~

fia) bl I{xl

x2 - 1 x- 1

- - - de-

di f(x)

x - 1 {~

função.

lim f(xi x+a

114-H

x2

1 lim - - x+a X - 1 -

~

IR -

lim (x + 1) x+a

x

1

x *-2

se

x

se

x * 1

se

x

se

x *-1

se

x

a + 1

f(a)

x

no ponto

x

no ponto

x

no ponto

x

~

O -2

-2

=

1

~

~

=

-1

-1

contínua no ponto especificado

{3X+2 -2x

se se

x :;;:'-2 x <-2

no ponto

x

b) flx)

x2 - 3x + 2 { x2 +4x _ 5

se se

x> 1 x<1

no ponto

x

~

1

32x - 10

se se se

x>4 x = 4 x < 4

no ponto

x

~

4

se se se

x> 1 x = 1 x < 1

no ponto

x

cl f(xl

pois, para todo

no ponto

ai flxl x

f é continua em

+ {~

se

H.113 Verificar se a função

não pertence ao domínio da

Notemos que temos:

=

1

finida em IR - { 1} não podemos afi rmar que f é descontinua em x ~ 1, pois

1

x:=; 2.

x:;;:' O x <O

-2

~

3

f(2)

f é cont ínua em

c) flx)

x

~

(7 - 2xl

4

~

3 Ix 2 - 1) = 3

X+2+

y

Na função f(xi

1(2)

=

H.112 Verificar se a função f é continua no ponto especificado.

temos:

-1

11m I{xl x+2

x+2~

x~..2+

x

2

==

Devemos verificar se

.Q-1

x+2-

~ ~~

x

x < 2 x:;;:'2

Solução

a) 1(2) bl 11m

f(x)

se se

a E IR - {1} , di flxl

{ 10 -2x

2X2 - 3x + 2 2 { 2 - x2

-2

115-H


H.114 Verificar se a função f é contínua em

se

te: x

ai flx)

C

x*O

se

x

se

x * O

se

x " O

=

e) flx)

=

O

{

{~~ sen x

r

se

x*O

se

x " O

-sen x se x + sen x

x * O

1

se

x " O

1

d) flx) "

H.115 Verificar se a função f

ai f(xl = { : : 2

c) flx)

x * 2

se

x

~x

=

x

se

x = O

sen 2x

se

x

cos a

se

x

x

O

no ponto

x

O

=

{~X~ll

seja cont ínua em

x

-=

* ==

O O

O

é contínua no ponto especificado.

se

{XL- 11)2

bl f(xl

*

se

a

tg x flxl

{

cl flxl

~-1

H.117 Determine a para que a função

-c;s~

b) flxl

x " O:

=

no ponto

x = 2

2

11. se

x * 1

se

x -= 1

se

x * 1

no ponto

x

123. Teorema no ponto

se

PROPRIEDADES DAS FUNÇÕES CONTfNUAS

Se f

x

x = 1

ções

e 9 são funções contínuas em a, então são continuas em a as fun-

f + g, f - g, f . 9

e -

f

neste último caso, desde que

g(a)

9

* O.

H.116 Determine a para que a função seja conÚnua no ponto especificado.

ai flxl

{

~5~

{

x-1 3 : - x

x - 2

a

Demonstração se

x*2

·se

x = 2

se

x * 1

no ponto

x

-=

2

Demonstraremos como modelo a continuidade de f + g. Como f e 9 são contínuas em a, pela definição temos: lim f(xl = f(a)

bl flxl =

c) fi xl

{ V;-2 x-4

3x + a

di f(xl

no ponto

lim g(xl = gla) x~a

x

se

x= 1

Para provarmos que f + 9 é contínua em a devemos provar a igualdade:

se

x >4

x+a

se

x<4

lim (f + g)(x) ~ (f + g)(a) no ponto

{V02-vI: 3x 2 -:x + a

116-H

e

x~a

x = 4

De fato:

se

x>O

se

x<O

no ponto

x " O

lim (f + g)(x)

lim [f(x) + g(x)]

lim f(x) + lim g(x)

x+a

x+a

x~a

fiai + g(al

X~a

(f + g)(a).

Agora faça a demonstração para as demais funções.

117-H


111. LIMITE DA {Yf(x)

124. Teorema do limite da função composta

Se lim g(x) x+a

Como hav íamos prometido quando da apresentação da propriedade La de limites, vamos demonstrar essa propriedade, mas antes vejamos dois lemas.

b e se f é uma função contínua em b, então

=

= f(b), isto é, lim /fog) (x) = f(Iim g(x)).

lim (fog) (x) x+a

X-1-a

X+a

126. Lema 1

Demonstração

Se

O teorema ficará demonstrado se provarmos

==

VE > O, 3 o > O I O < Ix - ai < o

lim.çr;

I(fog)(x) - f(b) I < E

Sabemos que f é contínua em b, isto é,

lim f(V) = f(b),

x+a

Por outro lado,

Iim g(x)

= b,

=

(I)

isto é,

"101 > O, 3 o > O I O < Ix - ai < o ~ Ig(x) - bl < OI g(x)

=

Com base nas afirmações =>

(11)

e

(111),

VE>O, 30 >0 10< Ix-al<o I (fog)(x) - f(b) I < E.

(11 )

-+

a+" ou " x

então

If(g(x)) - f(b)1 < E (111)

yr;z

=.if;;-

devemos provar V E > 0,3 o > O I O < Ix - a I < o

If(g(x))-f(b)/<E

=

I yr;z

- yr;;- I <

E

Lembrando da fatoração Vn _ b n = (V _ b) • (Vn-I + bV n - 2 + b2 Vn-3 + ... + bn-2 V + bn-I)

= isto é

Este teorema continua válido se o símbolo

x

I im x+a

Observação IJ

a E IR~,

Faremos a demonstração para o caso em que n E N* e a E IR+. Deixaremos a cargo do leitor a demonstração para o caso n E N*, n é impar e a E IR~ . Para demonstrarmos que

temos

==

n é ímpar e

~.

em (1), teremos:

VE > O, 301> O 10< Ig(x) - bl < OI

n E N*,

ou se

Demonstração

If(V) - f(b) I < E

x+a

Se substituirmos V por

=

e a E IR+

portanto,

y+b

VE > 0,3 OI > O I O < Iv - bl < OI

n E N*

-+

"x

-7

a"

for substituído por

Vn - b n

a -".

n

bl •

= (V -

L

.

bi-l vn-I

i=l

z;r;-I

Iyr; = b; decorre x = Vn e a

podemos expressar

-era

125. Teorema

=

em termos de n b , então:

Ix - a I. Façamos ~ = V e

n

Se a função 9 é contínua em a e a função f é contínua em g(a) então a função composta fog é contínua em a.

Demonstração

Ix - ai

=

Ivn - bnl

=

I(v - b) •

L

bi-lvn-il

i=l

I~ - yr;;-I •

Considerando que 9 é contínua em a, isto é, lim g(x)

=

g(a) e f é contí-

n

L

i-I

a n

n-j

X

n

i=l

x+a

nua em

g(a),

lim (fog) (x) x+a

o que prova que

118-H

e finalmente temos

pelo teorema anterior temos:

=

lim f(g(x)) x+a

= f(lim

g(x))

x+a

= f(g(a))

(fog)(a)

I~ -~I

Ix - ai

fog é contínua em a.

119-H


o>

Considerando que desejamos encontrar O <

==

Ix - ai < o

podemos fazer com que o Ix - ai < a Fazendo

=

tal que

1

Ix - ai •

i -I

I~

n-i

n-l

<

I x - ai •

It

E

• xn-

an

seja menor ou igual a a,

o

I

t a'~1

x

Faremos a demonstração para o caso n par, deixamos a cargo do leitor, como exercício, a demonstração para n ímpar. Pelo lema 1, temos:

n-J

a n

lim h(xl ~ lim vr;-

Se • E

n

lim •

d

onde temos para todo

>

E

O,

x+a

existe

n-l

a.

lim f(xl

L onde L?< O e n EC N' ou L < O e n é natural ímpar

VfW = V lim

x+a

f(x) ~

VL.

Demonstração

·d <o

tal que

== I yr;; - Z!a"1 <

Sendo a função h definida por h(x) - ~ temos a composta hof definida E

(hof)(x) ~ h(f(xl) ~I. Pelo lema 2 a fu"ção h é contínua em da função composta, temos por

De fato

O < Ix - ai < a < a O<lx-al<o

=

O<lx-al<a n-l

Ix - ai

<a

n

n-l n

• E

.

E

=

Ivr;- -

11m x+a

ou n-l n

<

é contínua em

então

n-l

"" O

h

x+a

n

o ~ min {a, a

Ix - ai

~ h(a)

128. Teorema

!:.:!.

<

yr;;

E,

isto é

O

=

x+a

o que prova que

a n

min {a, aO

an

Demonstração

l-I

Como queremos que

tomamos

n-l

an

yr-;z-

x+a

Ix-al<a

'E'--~E

A função h(xl ~ definida em IR. se n é par ou definida em IR se n é ímpar,é contínua em a para a EC IR: se n é par ou a EC IR se n é ímpar.

temos

n~' I '

.~-----< Ix - ai • j-n n-j

1 ._-< n-l

i-I

127. Lema 2

a n •X n

I x - ai

n

<a

isto é

O < x < 2a

x ~ O em

Ix - a I .

O

Vf(xl

~ lim hlf(xl)

h (Iim f(xl) ~ x+a

L, então pelo teorema do limite

V 11m

x+a

f(x) ~

VL

= E

<

a

vai

120-H 121- H


f'(xo)

ou

fIxo + _llx) - f(xo) llx

lim llx+o

Quando existe f'(x o ) dizemos que f é derivável no ponto xo. Dizemos também que f é derivável no intervalo aberto I quando existe f'(xo) para todo Xo E I.

CAPÍTULO VI

DERIVADAS

130. Exemplos 1?)

f'(3)

I.

DERIVADA NO PONTO

= 2x

Calculemos a derivada de f(x) =

lim f(x) - f(3) x+3 X- 3

=

no ponto Xo

lim 2x - 6 x+3 X - 3

lim x+3

=

= 3.

-~~ _ 2 X- 3

.

Outra maneira de proceder seria esta:

Xo

f'(3)

=

lim f(3 + llx) - f(3) Llx+O llx

=

lim 2(3 + llx) - 6 Llx+O llx

=

lim 2 Llx+O

2

129. Definição Seja f uma função definida em um intervalo aberto I e Xo um elemento de I. Chama-se derivada de f no ponto Xo o limite

f'(x o ),

[df] dx

x

=

ou

Xo

Xo

Calculemos a derivada de f'(l) = lim

f(x) - f(xo) x - Xo

lim se este existir e for finito. A derivada de f no ponto gu i ntes notações:

2?)

incremento da função

=

é habitualmente indicada com uma das se-

=

f(x) - f(xo) = - - - - - - - --

x - Xo

relativamente ao ponto

lim x-+ Xo

~"'"'

f(xlx - Xo

'---------

122-H

no ponto

Xo

1.

f(l + llx) - f(l)

.6.x [(1 +llX)2 + (1 +llx)]

lim Llx+O

llx

lim (llx):_ + 3· llx llx l'.x+o

=

Calculemos a derivada de

. lly Xo· O quociente -ll-;;

7T

f'(.:r) 3

_ _ _ recebe o nome de razao IncrementaI de f relativamente ao ponto xo.

Frisemos que a derivada de i no ponto formas:

f'(xo)

x! + x

lim (llx + 3) l',x+O

3

Df(xo) 3?)

f

=

i\X+O

A diferença llx = x - xo é chamada acréscimo ou incremento da variável x relativamente ao ponto xo. A diferença lly = f(x) - f(xo) é chamada acréscimo

ou

f(x)

--

ou

Xo

psJe ser indicada das seguintes

[f() .

Xo- lim

lly

llx+o llx

=

lim Llx+O

sen x

em

xo

7T

3

7T

sen( - + llx) - sen 3 3 ------- llx -- - - - -

=

llx 7T llx 2 . sen • cos ( - + - . ) 232 lim llx h.x+o

.

ou

=

f(x)

IIm ;\x+o [

llx sen 2

~x

1

7T

cos"3

=

2"

-------'

123-H


4?)

Calculemos a derivada de f'(O)

lim x+o

=

.

1

11m. 3r:;y

x+o V x

=

f(x) - f(0) x - O

f(x)

=.vx

em

{fX

.

=O.

Xo 1

= 11m - - = lim3r::r x+o X x+o V x-

portanto, como

tg

não existe

f'(O).

H_123 f(xl = cos x,

Xo = 2

Xo

=

Xo =

4

H.124 f(x) =

..,r;:,

Xo = 1

Xo = -1

H.125f(xl =

~

Xo = 2

H.121 f(x) = Ix I.

Xo = 1

H.126flx) =

ij';,

Xo = O

H.122f(x) = Ixl,

Xo = O

H.127flx) = x .Ixl, Xo = O

11.

=

Como existe

1r

x3,

H.120f(x)

f(x) - f(xo)

= --'---_-'----''-C

portanto, tg a é a razão incrementai de f relativamente ao ponto xo. Se f é contínua em I, então, quando x tende a xo, Q desloca-se sobre o gráfico da função e aproxima-se de P. Conseqüentemente, a reta s desloca-se tomando sucessivamente as posições SI, S2, S3, .. , e tende a coincidir com a reta t, tangente à curva no ponto P.

Nos problemas que se seguem calcule f'(xol.

H.119 f(x) = x2 + 2x + 5,

a

X - Xo

+ 00,

EXERCICIOS

H.118 f(xl = 3x + 1,

A reta s é secante com o gráfico de f e seu coeficiente angular é:

f'(xo)

=

lim x+xo

conclu (mos:

f(x) - f(xo)

x - Xo

lim x+xo

tg a

~

tg(lim

a)

tg il,

x+xo

A derivada de uma função f no ponto Xo é igual ao coeficiente angular da reta tangente ao gráfico de f no ponto de abscissa xo.

132. Quando queremos obter a equação de uma reta passando por P(xo, Yo) e com coeficiente angular m, utilizamos a fórmula de Geometria Analítica:

INTERPRETAÇÃO GEOMI:TRICA

131. Seja f uma função contínua no intervalo aberto I. Admitamos que exista a derivada de f no ponto X o E I. Dado um ponto x E I, tal que x xo, consideremos a reta s determinada pelos pontos P(xo, fixo)) e Q(x, f(x)).

y - Yo = m • (x - xo)

'*

Xo

x

Em particular, se queremos a equação da tangente t ao gráfico de uma função f no ponto (xo, Yo), onde f é derivável, basta fazer Yo = f(xo ) e m = f'(xo)' A equação da reta t fica: y - fIxo)

= f'(xol

• (x - xo)

x

124-H

125-H


H.130 Determi nar, em cada caso, a equação da reta tangente ao gráfico de f no ponto xQ:

EXERCICIOS H.128 Qual é a equação da reta tangente à curva

y = x 2 - 3x

ai f(xl~x+1, bl flx) ~ x 2 - 2x, cl f(xl ~ sen x,

no seu ponto de abscissa 41

Solução xo ~ 4 '==> f (xo I ~ 4 2 -

3 •4

d) f(xl

16 - 12 ~

o

xo ~ 3 xo ~ 1 xo ~ O

~~, x

xo

~

1

04

el flx) ~

V-;,

xo

~

4

então P14,41 é o ponto de tangência.

fi f(xl

-Ç.0 ,

xo

~

2V2

~

f'lxol

f'(41

flx)

lim x-+4

o

-

~

f(41 4

x-

Ix 2 - 3xl - 4

lim

x+4 --x---4--

~41(x+1I ~

lim x~4

x - 4

lim (x + x+4

111.

11 ~5,

portanto, o coeficiente angular de t

é

x

5 e sua equação é ~

y - 4

51x - 4).

INTERPRETAÇÃO CINEMÁTICA

133. Do estudo da Ci nemática sabemos que a posição de um ponto material em movimento, sobre uma curva T' (trajetória) conhecida, pode ser determinada, em cada instante t, através de sua abscissa s, medida sobre a curva A expressão que nos dá s em função de t:

r.

s(t)

H.129 Determinar a equação da reta tangente ao gráfico de flx) ~ tg x no ponto de abscissa Xo

rr

4.

~

é chamada equação horária.

y

Solução

Xo ~ PI

4rr

f,

=>f(xo) ~ tg

rr

4

~

1,

então

11 é o ponto de tangência.

rr

x

2

senlx-.:r!.1 4 f'(xol

~ f'I.!T) ~ lim 4

rr

x+4"

C05

x.

COS

rr

'4

Sendo dado um instante to e sendo t um instante diferente de to, chama-se to e t o quociente:

rr x--

velocidade escalar média do ponto entre os instantes

4

vm = ~

2

e a equação da reta t é

fl.s fl.t

e chama-se velocidade escalar do ponto no instante

lim t+to

126-H

s(t) - s(to) t - to

vm

lim t+to

s(t) - s(t o ) t - to

. 11m

/\t+O

fl.s

~ = t

to

o limite:

s'() to

127-H


Daí se conclui que:

H.132 Calcular no instante

A derivada da função s; s(t) no ponto t ; to é igual à velocidade escalar do móvel no instante to.

equação horária s

= -

t =

1

3 a velocidade de uma partícula que se move obedecendo à

.

t

(Unidades: SI)

H.133 Um ponto material em movimento sobre uma reta tem velocidade v

t. Calcular a aceleração do ponto no instante to

=

=~

no instante

2. (Unidades SI)

Solução

134. Sabemos ainda que a velocidade v de um ponto material em movimento pode variar de instante para instante. A equação que nos dá v em função do tempo

A aceleração no instante to

=.

2 é igual à derivada de v no instante to:

v'(to) = v'(2) = lim v(t) - v(2) = lim ~2 t-2 ~2

t:

0 - {/2 = t-2

v ; v(t)

é chamada equação da velocidade do ponto.

{/2:2

+

02

=

Sendo dado um instante to e um instante t, diferente de to, chama-se aceleração escalar média do ponto entre os instantes to e t o quociente: a

; ~v~(t~)~-_v~(t-",o~) f:J.v t - to ; f:J.t

m

e chama-se aceleração escalar do ponto no instante lim a m t+to

lim v(t) - v(to) ; lim f:J.v t+to t - to 6t+O f:J.t

H.134 Calcular a aceleração de uma partícula no instante t obedece à equação v = 2 + 3t + 5t 2 . (Unidades: SI)

to

=

5 sabendo que sua velocidade

o limite: v'(to)

Daí se conclui que:

IV. A derivada da função v; v(t) no ponto t; to é igual à aceleração escalar do móvel no instante to,

FUNÇÃO DERIVADA

135. Seja f é uma função derivável no intervalo aberto I. Para cada xo pertencente a I existe e é único o limite f'(xo); Iim

fIxo + f:J.x) - f(xo) f:J.x ' portanto,

""'+0

EXERCICIOS

H.131 Um ponto percorre uma curva obedecendo à equação horária s a sua velocidade no instante to = 2. (Unidades 5.1.)

=

t 2 + t - 2. Calcular

Solução A velocidade no instante to

2 é igual à derivada de s no instante to: slt) - s(2) __ l'lm (t 2 + t - 21 - (2 2 + 2 - 2) s'(to) = s'(2) = lim -'--=-~.c... t+2 t-2 ~2 t-2 =

128-H

fim t+2

podemos definir uma função 1': I ~ IR que associa a cada xo E I a derivada de f no ponto xo' Esta função é chamada função derivada de f ou, simplesmente, derivada de f. df Habitualmente a derivada de f é representada por f', ou Df. dx A lei f'(x) pode ser determinada a partir da lei f(x), aplicando-se a definição de derivada de uma função, num ponto genérico x E I:

=

t2 + t - 6 It - 2)(t + 3) - lim t - 2 t+2 t - 2

=

5 m/s

f'(x) ; Iim 6x+O

f(x

+ f:J.x) - f(x) f:J.x

É isto que faremos logo em seguida para calcular as derivadas das principais funções elementares.

129-H


V.

DERIVADAS DAS FUNÇÜES ELEMENTARES

138. Derivada da função seno Dada a função

f(x) = sen x,

temos:

136. Derivada da função constante

flx sen(2 )

f(x + flx) - f(x) = ~ = O flx flx

fly flx

. f '() x = 11m

-fly

6x~O flx

2 • sen( flx) • cos(x + flx ) 2 2 flx

fly sen(x + flx) - sen x flx = - - - flx

Dada a função f(x) = c, c E IR, temos:

cos(x + flx ) 2

=~-.

2

O

Logo,

I

flx cos(x + 2 ) = cos X

lim 6x->O

f(x) = c

== f'(x) = O

~1

Logo,

I

f(x) = sen x

== f'(x) = cos x

139. Derivada da função cosseno

137. Derivada da fu nção potência

Dada a função

f(x) = x n , n E 1\1*, temos: fly f(x + flx) - f(x) (x + flx)n _ x n flx = flx flx Dada a função

f(x) = cos x,

cos(x + flx) - cos x flx

fly flx

( ~ )x n + ( ~ )x n-1 • flx + ( ; )x n-2.(flx)2 + ... + ( n )(flx)n _ xn flx

- - - - - - -n - - - - - sen (x +

( n, )xn-1 + ( n )xn-2 • flx + ( n )xn-3 • (flX)2 + 2

f'(x) = lim 6x->O

3

Ll.y Ll.x

I

000

flx

2 )

n

f'(x) = lim 6x->O

fly Ll.x

flx ) 2 flx

-2 . sen(x + -

flx sen(2)

+ ( n )(Ll.x)n-l

( n, )x n- 1

temos:

- lim A.x-+o

flx 2

sen(x + Ll.x ) . lim 2 6x->O

Ll.x sen(2 ) ---.---- = -sen x flx

~

Logo,

=1

Logo,

I 130-H

f(x) = x

n

==- t'(x)

n 1 n· x -

I

"-f(-X-)-=-c-o-s-X-==---f'-(X-)-=---s-e-n-x-

131-H


140. Derivada da função exponencial Dada a função f(x) vada. Temos:

=

H.138 Obter a equação da reta tangente ao gráfico da função f(x)

aX, com a E IR e O < a i= 1, calculemos a sua deri-

f(x + /::"x) - f(x) /::"x

f'(x)

=

/::"y

11m

_.

aX

lim

6X+O

==- f'(x)

= aX • Qn a

N"o instante

No caso particular da função exponencial de base e, f(x) sultado notável: f'(x)

=

=

eX, temos o re-

v(

.4 s

t:::'"6 s.

1T

4) =

= eX

==- ~os

1T

al-)

==> f'(x)

s, temos:

1T.J2

m/s 2

= v'(t) =

t. t

1T

==65,

temos:

V3 m/s2

1T

-cos -

= - -

H.14D Um móvel desloca-se sobre uma reta obedecendo à equação horária s = r4 (Unidades SI). Determinar: a) b) c) d)

H.135·Obter a derivada das seguintes funções: flx) = 5

g(X)

=

x6

h(x)

=

xiS

H.136 Obter a derivada das seguintes funções: flx) = c • x n (c E IR e n E N*) glx) = tg X hlx) = sec x H.137 Obter a equação da reta tangente ao gráfico de Solução

sua sua em em

velocidade no instante t = 25, aceleração no instante t = 35, que instante sua velocidade é 108 m/s , que instante sua aceleração é 48 m/s,

VI. DERIVADA E CONTINUIDADE f(x)

=

cos x

no ponto

141. Teorema

Sejam a função f:A -+ IR e Xo E A. Se f é derivável em Xo, então f é

O coeficiente angular da reta procurada é: =

=

v =

662

EXERC(CIOS

,1T

="41T

"4 = - 2

-sen

No instante

f(x)

t

bl A derivada de v nos dá em cada instante a ac.leração do móvel, isto é, a

eX • Qn e = eX

Logo,

-sen

1T

3" =

-

V3

-2-

portanto a equação da reta é:

contínua em xo.

Demonstração Notemos que: f(x) - f(xo)

132-H

1T

a) A derivada de s nos dá em cada instante a velocidade do móvel, isto é, ~ s'(t) ~ -sen t.

f(x) = aX

cos t

Solução

6X+O

Logo,

f ("3)

t ==

==

1T

b ) sua aceleraçao no Instante

Iim

A

6><+0 '-' X

eX no ponto de abscissa 2.

H.139 Um móvel desloca-se sobre um segmento de reta obedecendo à equação horária s (Unidades 511. Determinar:

aI sua velocidade no j nstante

/::"y /::"x

==-

f(x) - fixo) x - Xo

= ~:....-.-.:.'.:.:".:...

• (x - xo)

133-H


lim (f( x) - fIx o))

X+O

~

lim x+ xo

f(x) - f(x o) x - xo

lim x+ Xo

po rta nto : lim x+ xo

f(x )

f(x o)

e, po r def ini ção , f é

(x - xo )

~

f'(x o) •

co ntí nu a no po nto

o~ o

xo .

CA PÍ TU LO VII

142. No tem os qu e o rec ípr oco des te teo rem a é fal so, isto é, exi ste tín ua s em xo e não m fun çõe s con der ivá vei s em xo.

REGRAS DE

-

Exemplos 1?)

A fun ção lim

f(x) ~ Ixl

é co ntí nu a no po nto

Ixl ~ O ~ fIO}

X'" o

po rém , est a fun ção nã o é der ivá vel no po nto lim Ix l- O =_ l e lim Ixl - O x+ ox- O x - O x+o + en tão não exi ste

f'(O)

lim

O poi s:

cos2)~ O ~ HO} xo ~ O:

x. cos nO ) ~ Iim f(x ) - f ( O) ~ lim x X'" o e est e últ im o lim ite não exi ste .

DE RI VA DA DA SOMA

143. Sej am u ~ u(x ) e v ~ v(x) du as fun çõe s der ivá vei s em I qu e a fun ção f(x) ~ ~ la, b[. Pro vem os u(x ) + v(x) tam bé m é der ivá vel em I e sua der iva da é f'(x ) ~ u'( x) + v'( x) Te mo s:

X

ma s f não é der ivá vei no po nto

x - O

Ne ste cap ítu lo vam os sis tem ati zar o cál cul o das der iva das pro cur regras de der iva ção par and o ob ter a de ter mi na r a der iva da de um a fun ção , sem nec ess ari am ent e à def ter de rec orr er ini ção .

I.

x - O

1 A fun ção f(x ) ~ x • cos - =F O e f(O) ~ O é co nü nu a no po nto x xo ~ O:

Iim (x • x+O

DERIVAÇAO

O poi s:

Ixl - O

X'" o

2?)

xo

xo

X'" o

x

1 : lim co sX'" o

t::.y ~ f(x + t::.x) - f(x) ~ [u( x + t::.x} + v(x + t::.x )] - [u( x) + v(x )] : [u( x + t::.x) - u(x )] + [v( x + t::.x) - v(x )] ~ t::.u + t::.v En tão :

X

t::.y

lim

!\x "'o t::.x

~

.

11m 6x" 'O

t::.u t::.x

. + 11m DX"'O

t::.v t::.x

Co mo u e v são fun çõe s der ivá vei s,o s doi s lim ite s do seg un do me mb ro são fin ito s, po rta nto , lim ~y é fin ito , isto é, f é derivável DX"'O uX em I. Ca lcu lan do os lim ite s, tem os: f'(x ) ~ u'( x) + v'( x) Em res um o:

I

f(x ) : u(x} + v(x}

13 4- H

== > f'(x )

u'( x) + v'( x)

I


Notemos que esta propriedade pode ser estendida para uma soma de n funções. Assim:

Calculando os limites, temos: ~

f'(x)

u'(x) • v(x) + u(x) • v'(x)

Em resumo: ffxL= udx)

+ uz(x)

+ ... +un(x)

-

f'(x) = uj(x) + u2(x)

+ .., + u~(x) f(x) = u(x) • v(x)

sempre que x E I e UI. U2 •. ". Un sejam deriváveis em I. Notemos também que a derivada de uma diferença de funções pode ser obtida através de fórmula semelhante à da soma pois: f(x) ~ u(x) - v(x)

=

=

f(x) ~ u(x) + [-v(x)]

=

f'(x) ~ u'(x) + [-v' {x)]

u'(x)· v(x)

+ u(x)

• v'(x)

146. No caso particular em que f(x) ~ c • v(x), isto é, u(x) ~ c (função constante) e v(x) é uma função derivável, a regra precedente leva ao seguinte resultado: ~

f'(x)

f'(x) ~ u'(x) - v'(x)

===> f'(x)

u(x) • v'(x) + u'(x) • v(x)

~

c· v'(x) + O • v(x)

~

c· v'(x)

Logo,

I

144. Exemplos

=

10 )

f(x) x + 1 f(x) ~ x 2 + 3 = t(x) sen x + cos x f(x) ~ x 2 - eX

2C?) 3C?) 40 )

f'(x) ~ 1 + O ~ 1 f'(x) ~ 2x + O ~ 2x = f'(x) ~ cos x - sen x f'(x) ~ 2x _ eX

=

1C?)

DERIVADA DO PRODUTO

4C?)

145. Sejam u ~ u(x) e v ~ v(x) duas funções deriváveis em I ~ la. b(. Provemos que a função f(x) ~ u(x) • v(x) também é derivável em I e sua derivada é f'(x)

~

=

f'(x) = c • v'(x)

I

147. Exemplos

2C?) 3?)

11.

f(x) = c· v(x)

u'(x) • v(x) + u(x) • v'(x)

f(x) f(x) f(x)

3x' = f'(x) ~ 3(4x 3 ) ~ 12x 3 2 3x + 5x f'(x) ~ 6x + 5 (x 2 + 1)(x 3 + 2x) f'(x) ~ 2x • (x 3 + 2x) + 2 2 + (x + 1 )(3x + 2) ~ 5x' + 9x 2 + 2

=

=

f(x) ~ sen x • cos x ~ f'(x) ~ cos x • cos ~ cos 2 X - sen 2 x

X +

sen x. (- sen x)

=

148. Notemos que a propriedade da derivada do produto pode ser estendida para um produto de n fatores. Assim:

Temos:

t!.y

~ f(x +

t!.x) - f(x)

~ u(x +

t!.x) • v(x + t!.x) - u(x) • v(x)

~

f(x) ~ udxl • Uz (x) ..., • UnIx)

+

u(x + t!.x) . v(x + t!.x) - u(x) • v(x + t!.x) + u(x) • v(x + t!.x) - u(x) • v(x)

UI (x) • uí (x) •

=

f'(x) = uj (x) • Uz (xl·

• UnIx) + ...

+

un(x) + u~(x)

UI (x) • U2 (x)

lu(x + t!.x) - u(x)] . v(x + t!.x) + u(x) . [v(x + t!.x) - v(x)] ~

t!.u • v(x + t!.x) + u(x) • t!.v sempre que x E I e UI. U2, .... Un sejam dériváveis em I. Em particular. se UI (x) ~ U2 (x) ~ ... ~ UnIx) ~ u(x), esta propriedade se re·

Então:

. I Im

t!.y

~-~

,-,x+o t!.x

lim

t!.u

_ . lim

,-,x+o t!.x

,-,x+o

v(x + t!.x) + Iim

,-,x+o

.

u(x). 11m L>x70

t!.v t!.x

--

Como u e v são funções deriváveis, e portanto contínuas, os quatro limites

t!.y é finito. isto é, f é derivável em I. '-'X7 o t!.x

do segundo membro são finitos e, assim, lim

136-H

duz a:

I f{x)~ [u(x)]n= f'(x)

n.[u(xllo-r·u'(x)

I 137-H


149. Exemplos 1'?)

==

f(x)

f'(x) ; 2x • sen x • eX '-.----'

+

x2

'-.----'

4

f(x)

cos x • eX + x 2

'-----,.------'

ui

UI

2'?)

x2 + x + 1

bl fazendo

sen x ;

~

"---r'

U3

~

"---r'

U2

U3

g'lx)

+

c)

5 • [ulx)

Fazendo

o

ulx), vem glx)

sen x • eX

o

[ulx) jS,

então:

4

J4 • u'lx)

51x 2 + x + 11 (2x + 11

eX. cos x - x 2

h'lxl ~ 4 • [ulxlJ3 • u'lx)

u{x),

vem

h(x)::o lu(x)]4,

então:

4' lo' • cos x - x 2 )3 • Ie' • cos x - e' • sen x - 2x)

o

'=;-'-'J '---r-'

UI

U2

u]

f dada abaixo.

H.144 Obter a derivada de cada função

== f'(x)

~)4

o

ai b) cl d) e) f) g) hl

u

i)

EXERCICIOS

flxl flx) flx) flx) flx) f (x) flx) flx) flx)

i I flxl

13x 2 + x)11 + x + x 3 1 x 2 1x + x 4 111 + x + x 3 ) 12 + 3x + x 2 1s 12x + 31 s2 x3 • eX

x • eX + cos x x4 • a 2X

e 3X ex2+x+ 1

-= cos S X

=. sen 7 x . cos 3 x flxl = a • sen x + b • cos x

k) flx)

H.141 Calcular a derivada da função polinomial

f(x) = ao + atX + 82X2 + '.. + anx n .

I1

Solução

(a, b E !R)

H.145 Calcular a derivada da função

Trata-se de uma soma em que as parcelas têm a forma apx P, portanto, sua derivada é 1 P • ªpx P - . Assi m, temos: n-I f'lx) ~ aI + 2a2x + 3a3x2 +

+

n • anx

f(x) '"" {sen x

+ e X }2 (cos x + x 3 )3

no ponto

Xo

O.

H.146 Obter a equação da reta tangente ao gráfico da função f(xl --' (3· sen x + 4· cos x)5 no ponto de abscissa Xo = 1T. H.147 Obter a velocidade e a aceleração de um ponto material que percorre um segmento de reta obedecendo à equação horária s =- a • e-to CQS t, com a E IR. (Unidades: SI).

H.142 Calcular a derivada de cada uma das seguintes funções:

a) flx) - 8x ll bl flxl c) flx)

d) fi x) e) flx) fi flx)

_2.

.J3

x3 7 5 5 + x + 3x 2 3 + 5x 2 + x 4 x3 + x 2 + x + 5 n n '" 3 + 2x + x 2 , nEN

111. DERIVADA DO QUOCIENTE H.143 Calcular a derivada de cada uma das seguintes funções: a)

f(x) = eX • sen x + 4x 3

b) glxl cl hlxl

~ o

150. Sejam u; u(x) e v

Ix 2 + x + l)S (ex. cos x - x 2 )4

Provemos que a função

Solução a)

f

deve ser vista como soma de duas parcelas

(ex. sen x

e

4x 3 l,

portanto,

f'

f'(xl

é a soma das derivadas das parcelas, sendo que a primeira parcela é um produto, Df(x) - Dle' • sen x) + D14x 3 ) -

'D(e x ) • sen x + eX . D(sen x) -+ D(4x 3 ) """ eX. sen x + eX. cos x + 12x 2

138-H

c

duas funções deriváveis em I e v(x) "" O em I.

u(x~

v(xl

também é derivável em I e sua derivada é

u'(xl . v(xl - u(x) • v'(x) [v(x) 12 Temos:

então:

f'(xl

f(x)

v(x)

11 y

u(x + Lixl ; f(x + Lix) - f(x) - v(x + Lix)

u(x) v(xl

u(x + Lix) • v(x) - u(x) . v(x + Lixl v(x + Lixl . v(x)

139 -H


152. Conseqüências

u(x + 6x) • v(x) - u(x) • v(x) + u(x) • v(x) - u(x) • v(x + 6x) _v(x + 6x) . v(x) ---~_

....

lu(x + 6x) - u(x)] • v(x) v(x + 6x) • v(x) 6u .

1a )

u(x)· [v (x + 6x) - v(x)] v(x + 6x) • v(x)

v(x) v(x + 6x) . v(x)

Derivada da função tangente

Dada a função

u(x) . t.v v(x + 6x) • v(x)

f(x) ~ sen x cos x

e então podemos

aplicar a regra da derivada de um quociente: ~

u(x) = sen x

Então:

~

v(x)

6y 6u v(x) . u(x) 6v lim = lim - _ . lim - 11m ·Iim Ax+o 6x Ax+O 6x Ax+o v(x + 6x\ • v(x) Ax+O v(x + 6x) • v(x) AX+O t.x

Como u e v são deriváveis e continuas, os quatro limites·do segundo mem-

~

cos x

u'(x) v'(x)

cos x -sen x

portanto, cos 2 x + sen 2 x

u'(x) • v(x) - u(x) . v'(x) (V(X))2

f'(x)

bro são finitos e, portanto,

sabemos que

f(x) ~ tg x,

cos 2 x

Logo

6y lim - - é finito, ou melhor, f é derivável em I. . x+o 6x

flxl = tg x

===> f'(x)

Calculando os limites, temos:

2a )

, , v(x) u(x) f (x) ~ u (x) • Iv(xlj2 - [v(x)]2 • v'(x)

Derivada da função f(x) ~ [u(x)]-n,

1

Dada a função f(x) = [u(x)j"n = [u(x)]n' Em resumo:

n E ~. podemos aplicar a regra da deri-

vada de um quociente:

u(x) f(x) = v (x)

=- f'(x)

f'(x) = O • [u(x)]n - 1 • n· [u(x)]n-I [u(x) ]2n

U'(x) • vlxl - u(x) • v'(x) (v(x)

J2

• u'(x)

n • u'(xl [u(x)]n+l

r

-n • [u(x) ln +ll . u'(x) Em particular, se

x

u( x)

vem a importante regra:

151. Exemplos 1°)

eX f(x) - x 2

2°1

f(x)

----

f(x)

sen x aX

3°)

~

~

~

140-H

eX (x 2

-

2x)

x4

(2x)(x + 1) - (x 2 + 1)(1) (x + 1)2

f'(x)

x x cos x • a - se n x . a • logea (a X)2

(cos x - sen x • logea)

a

eX • x 2 _ eX • 2x (x2 )2

f'(x)

2

+1 x +1

x

f'(x) =

x 2 + 2x - 1 (x + 1)2

EXERCICIOS H.148 Derivar as seguintes funções:

f(xl

=.2.., x

glx)

=

x24 ' hlxl

= __ 1_,

senx

jlxl

=

-e

fx

Solução flx) = x-I

=f'(xl

1-11 •

x- 2

1

:=

--;2

X

141-H


g(x)

=

2 •

X-

4

=> g'lx)

hlxl = (sen xl- 1

=

2 • 1-4) • x -s

IV. DERIVADA DE UMA FUNÇÃO COMPOSTA

=> h'(x) = (-1) • (sen x)-l • cos x => i'lxl

=

7, 1-2) • le x l- 3 = -

cos x

(REGRA DE CADEIA)

sen 2 x

= -

l~x

153. Seja f:A --> B uma função dada pela lei V = f(x). Seja g:B --> C uma função dada pela lei z = g(V). Existe a função composta F:A --> C dada pela lei z = F(x) = = g(f(x) I.

e

H.149 Derivar as seguintes funções:

2 x7 bl flxl = 3x- s

ai fi xl

cl fix)

o

f)

gl fix)

1

x2 + x + 1

di fi xl

x +1 x- 1

e) f(x)

x+3 x+2 --+~~ x - 1 x+1

x 2 + 3x + 1 x-2

flx)

Supondo que f seja derivável no ponto x e 9 seja derivável no ponto V tal que V = f(x), provemos que F também é derivávei em x e sua derivada é F'(x) = g'(V) • f'(x) .

x 2 • sen x eX

Temos inicialmente: t1z t1z t1V ~ = !:::.y' t1x

_ cos x h) f(xl ~~

Notemos que se t1x tende a zero, então t1v também tende a zero pois a função V = f(x) é derivável e, portanto, contínua no ponto x. Assim, para valore, próximos de x (t1x --> O) a função f assume valores próximos de V = f(xl (t1V --> O). Então, temos:

H.150 Obter a derivada de cada uma das seguintes funções:

ai b) cl di el

f)

fix) = cotg x flxl = sec x flx) =: cossec x fixl = tg 2 x fix) = sec x - t9 x

Ix 2 +1) • tg x

flxl

11m f\x+O

tg x gl flx) sen x + cos x ex hl flx) = l __)2 tg x

t1z

--~ 11m

~x

I\X+O

Xo

-=

f(x) =

x

+

L:i.y

lim

t1v

. 11m

t1x

:W+o t1V

-~=

L\X+O

t1z

lim '\x+O

!:::.Z

!:::.V

_

Como z = g(V) e V = f(x) são deriváveis, l i m , e lim ,- . sao ambos y+o w.y .x+o uX finitos, portanto, lim

H.151 Obter a equação da reta tângente ao gráfico de

t1z --o

eX

~z

também. Assim z

I\X-:l>O uX

no ponto de abscissa

F'(x)

-1.

~

F(x) é derivável e sua derivada é:

g'(v) • f'(x)

Em resumo: H.152 Calcular o valor da derivada da função

f(x)

1 = Xl +

e

-x

+

2

sec x

quando

Xo

1r

4~.

F(x) = g(f(x)) H.153IMAPOFEI-7D) - ~ dada a função

y

a) Determinar a derivada. bl Calcular lim y.

10)

g'(f(x)) • f'(x)

Derivar

F(x) = cos 2x.

Fazendo y ~ f(x) ~ 2x e z ~ g(V) ~ cos y, temos: y' ~ f'(x) ~ 2 e z' = g'(vl ~ -sen y, portanto, vem: F'(x) ~ g'(vl . f'(x) ~ (-sen vi . 2 ~ -2 . sen 2x

Determinar os pontos do gráfico em Que a tangente passa pela origem.

20

142-H

F'(x)

154. Exemplos

X-*+ 00

c)

==-

)

Derivar F(xl ~ sen 3 x. Fazendo V = f(x) = sen x e z = g(V) = V3 , temos: V' = f'(x) ~ cos x e z', g'(V) = 3 V2, portanto, vem: F'(x) = g'(vl • f'(x) = (3y') • cos x ~ 3· sen 2 x • cos x

143-H


30)

j)

Derivar

F(x) = e7xL2X Fazendo y = 7x 2 - 2x e z = g(y) = eV, temos: y' = f'(x) = 14x - 2 e z' = g'(y) = eV, portanto, vem: F'(x) = g'(y) • f'(x) = eV • (14x - 2) = (14x - 2) • e7x2-2X

jI

X2 +SX+I

k) F(x) = tg 3 2x I) F(x) = e sen 2x

F(x) = a (aE IR.) F(xl = tg(cos xl

H.156 Calcular o valor da derivada da função f(x) = cos 3 x + sen 3 x no ponto Xo = H/2. H.157 Calcular o coeficiente angular da reta tangente ao gráfico da função flx) = .'2. SX ponto de abscissa -1 .

no

EXERC(CIOS H. 158 Obter a equação da reta tangente à curva

no ponto de abscissa -2.

H.154 Utilizando a regra da função composta, obter a derivada de cada função abaixo:

bl Flxl=senxn(nEI\I'1

cl F(xl =a

(x 2 1

H.159 (EPUSP-61I As derivadas dos termos da seqüência:

d) F(xl = (flxl)n(n (01\1') e) F(x) ~ cos (sen xl fi F(x)=sen 3 3x

ai Flx)=cosnxlnEr,J')

H

sen x, sen(x + 2\' sen(x •

m, ..., sen(x

nH

+ T)'

(aEIR.1

também são termos da seqüência? Solução n a) Fazendo y = f(x) = cos x e z = g(v) = v , temos: y' = I'lx) = -sen x e z' _ g'lv) = n • v n - l , portanto, vem: n l F'(x) = g'(yl • f'lxl = Invn-IH-sen xl = -n • cos - x • sen x n e z = g(yl = sen V, vem: v' = f'(x) bl Fazendo V = flx) = x e z'=g'ly)=cosv e dai: n l F'lx) = g'(y) • f'lx) = Icos v) • (nx n - I ) .~ nx -

cos x

V.

e z = g(vl = vem: v' = I'lx) - 2x cl Fazendo V = flxl = z' = g'(v) = a V • logea e dai: ( 21 F'(xl = g'lyl • f'(x) = (a V • logea)(2xl ~ 2xa x • logea n di Fazendo V = f(x) e z = g(v) = v , vem: v' = f'(xl e z' = g'(vl = nvn-I, e dai: F'lxl = g'lyl • f'(xl = nvn-II'lx) = n[Hxil n -! I'(x) el Fazendo y = flxl = sen x e z = glvl = cos v, vem: V z' = g'(vl = -sen V logo: F'lx) = g'lv) • f'(x) = (-sen v)(cos x) = -cos x • sen (cos x)

f'(x)

°

155. Seja a função y = f(x) bijetora e derivável em I tal que f'(x) i= para x E I. Provemos que a função inversa x = ri (y) é derivável em f(l) e que -I " 1

e

(f

cos x

=

)(y) = f'(x)

hl!}(f(xl)), isto é, F é a composta de três funções. Como a regra =

Sendo f derivável e, portanto, contínua, se também tende a zero. Assim, temos:

.6.x

cl Flxl = a • sen bx (a. b E IR) d) F(xl = cosl3x 2 + x + 51

144-H

w )(y) = lim l

~x

6v+0"'y

H.155 Obter a derivada de cada uma das seguintes funções:

°

i=

(3z 2 1(cos v)(3) = (3' sen 2 vllcos v)(3) =

ai F(xl = sen 4x cos 7x bl F(xl - - - x

y = f(x),

Como f é bijetora e derivável temos que .6.x demos escrever:

da composta pode ser generalizada para a composta de n funções, vem:

F'lx) = h'lzI • g'(V) • f'lx) = 9· sen 2 3x • cos 3x

sendo

=> .6.y

i= 0,

portanto po-

e

fi Fazendo V = f(x) = 3x, z = g(v) = sen y e t = hlz) ~ z3, temos y' = f'(x) = 3, z' = g'(y) = cos V e t' = h'(z) = 3z 2 . Notemos que F(xl

DERIVADA DA FUNÇÃO INVERSA

n

aV ,

x2

n l n' x -

= lim

} , 6x+0"'y .6.x

tende a zero, então .6.y

1 .

1 .6.y = f'(x)

I Im - 6X+0 .6.x

Logo,

x e) Flxl = sen e f) F (x) = x + 3 • tg 4x sen x (a E IR.) gl F (x) = a hl F(x) = cotgl3x - 1)

145-H


156. Conseqüências

Já vimos que:

x

1. Derivada da função !ogar(tm ica

Sabemo s que a função logar(tmica é a inversa da função exponen cial: y ~ logax = x ~ aY

=

x' ~ aY

sen y

=

x'

,1

cos y

-..; 1 - sen 2 y

Em resumo,

Qn a

Empreg ando a regra ora deduzid a, vem: y' -

~

1

y ~ ~,- ~ cosy

Já vimos que:

x ~ aY

~

Empreg ando a regra da derivada da inversa, vem:

arcsen x

y

===> y'

x'

4. Derivada da função arc cos

Em resumo,

I

y

~ Ioga x

===>

y'

y

~

arc cos x

~

x

cos y

~

Já vimos que:

No caso particul ar em que

a Qn x

y

Sabemo s que a função y ~ arc cos x, definida em I ~ [-1, 1] com imagens em [O,7T] é a inversa de x ~ cos y:

e,

temos:

x

~

cos y ===> x'

~

- sen y

Empreg ando a regra da inversa, vem:

===> y'

x

..; 1 - x 2 2. Derivada da função potênci a com expoen te rea!

Dada a função

y ~ xO<,

onde O< E IR

e x

Em resumo,

>

O,

temos:

y ~ xO< ~ (eQr x)O< ~ eO<' Qn x

arc cos x

y

=

y'

Aplican do a regra de derivação da função logar(tmica, obtemo s: ,

O< . Qn x

y~e

1

O<

-1

'O<'x ~x'O< 'X

~O<'X

0<'1

5. Derivad a da função arc tg

Em resumo, fica generalizada para qualque r O< real a seguinte regra:

Dada a função

y

=

arc tg x, de IR em

y = arc tg x

x

~

tg Y

~

7T

7T

]- 2" ' 2[' sabemos que:

então: 1

y'

3. Derivada da função arc sen

em

Sabemo s que a função y ~ are sen x, definida em I ~ [-1, 1] com imagens 7T 7T [- '2 ' "2 J. é a inversa de x = sen y: y

are sen x

=

1

1

='7 = sec2 y =1+ tg 2 y

Em resumo,

y

x

=

are tg x

===>

y' =

1

Ti-7

sen y

146-H 1 A"'7

••


EXERCrCIOS

H.162 Determin ar a função derivada das seguintes funções:

a) f(x) ~ 1092x b) f(x) = 1092cos x c) f(x) = '\IX d) f(x) =

e) f(x) ~ ~ fi f(x) = arc sen x 2 g) f(x) = arc cos eX h) t(x) = arc tg (Qn x)

var-:> x2

Solução a) f' _

~ x 4h

a) f(x)

H.160 Determin ar a função derivada das seguintes funções:

bl t(x)

~

W

c) t(x)

~

x

d) t(x)

~

f!;

e) flx)

~

{Ix ...;;

yÇ7

V = cos x

~ ~

g) t(x)

h) t(x)

b) Vamos aplicar a regra para funções compost as: e

1 z'(v) • V'(x) = ~ • !-sen x)

= 1092V então f'(x) =

z

i)

f(x)

j)

t(x)

I)

f(x)

sen x

c)f(x) =y';= x

f'(x)

=

V

= sen

x

1

V = x2

e

f'(x) = z'(V) • V'(x) g) Fazendo

V = eX

f'(x)

e

z

= V;,

2:;;rv'

x3

v,

z = are eos V,

=

x

m) f(x)

-3

z ~ are tg V,

x

VI2x_x4

~~+ x

Qn x

b) f(x) = x n • Qn x (n E FlJ) c) f(x) ~ (ax + b) • Qn x d) t(x) = sen x • Qn x

~x2+1

(x 2

+ 1)

o) t(x) p) flx)

~

= - _/

V 1 - e2x

(a, b EIR) (a, b, c

E

IR)

~

3 +

x + 1 - ----

~

V;

r)

f(x) ~ ~

s)

t(x) = Qn

t)

f(x) ~ loga ~

eX

~

Vl-~

temos:

H.163 Determinar a função derivada das seguintes funções:

x

a) flx) b) f(x)

H.161 Determin ar a função derivada das seguintes funções: a) f(x)

q) t(x) ~ eos

1

+ V2

~

V; ~ -2~V;

.

(a, b E IR)

~ ~(1 + x + x 2 )4

nl t(xl

temos:

V 1 - V2

f'(x) = z'(V) • V'(x) = 1

temos:

1 = ~ • 2x

e

2

temos:

f'(x) = z'(v) • v'(x) = - _ ~ • e

V = Qn x

1 3

=:: -

-i- =2\7X 1

eos x = -2-'y=;'s"'e-n-x-

1

h) Fazendo

1 =T' x

la, b EIR)

ax 2 + bx + c ex 2 + bx + a

cos x

z = are sen

e

-I

l-I

='3'

1

z'(v) • v'(x) ~

Fazendo

J

1 =f'(x)=T

= ~ = x l/3 =

e) Fazendo

f)

1/2

_x- 2

~ ~ax2 + bx + c ~ .Ja+b...r;

= - cos x • Qn2

d) f(x)

.ç;;

fi t(x) ~..;; +

1

-~

e) t(x)

Qn x cos x

f) f(x) = Qn (ax 2 + bx g) f(x) = Qn sen x h) f(x) = loga109b x

= are sen 3x = are eos x 3

c)

i)

x1

c) f(x)

=::

are tg

d) t(x)

=::

x 2 + are sen x

e) f(x) = are cos x -

+

h) t(x)

fi f(x) g) f(x)

~

x • are tg x

(f;

j)

V;

= Qn are eos x

f(X)=~

f(x) ~ x • arc sen x 2 - e x

kl flx) = arc eos I)

3

V; ex

t(x) = Qn arc sen x

are cos x

are sen x

148-H 1Aft

.1


H.16 4 Obter a equação da reta tangente à curva Xo

~

3.

x • ~ no pont o de abscissa

y

H.1G5 Obter os pont os em que a reta tangente à curva y

==

x + 1. . d ---,==-=: e parale Ia ao eixo os x. V x- 1

H.16 6 Obte r o valo r da deriv ada da inversa da funç ão t(x) ~ x 3 + x no pont o Xo ~ 1. Solu ção y ~ x3

+

dy

=

x

dX

~ 3x 2 + 1

H.16 7 (EPU SP-6 7) Dada a funç ão Y ~ x 3 no pont o Yo ~ 6. H.16 B Dada a funç ão

f(x)

~

[u(x ) ]v(x) .

=

dx dy ~

+ x 2 + 4x,

1

-3,,2+1

para

x

1,

temo s

VI. DE RIV AD AS SUCESSIVA S 157 . Seja f uma funç ão con tínu a em um inte rval o I e seja 11 o con junt o dos pon tos de I em que f é deri vável. Em 11 já defi nim os a funç ão f', chamad a funç ão derivada prim eira de f. Seja 12 o con junt o dos pon tos de 11 em que f' é derivável. Em 1 pod emo s defi nir a funç ão deri vada de f' 2 que cham arem os de derivada segunda de f e indi care mos por f". Rep etin do o proc esso , pod emo s defi nir as deri vada s terc eira , qua rta, etc de f. A deri vada de orde m n de f repr esen tare mos por f(n). 158 . Exe mpl os

calcu lar a deriv ada de sua funç ão inversa

1 '?) calcu lar sua deriv ada.

f'(x) = 6x + 5 f"(x ) = 6 f'''(x ) = f(4) (x)

Solu ção

~

~ [e.\'.n u(x) ] v(x) ~ eV(x) • Qn u(x) Aplicando a regra de derivação da funç ão com post a, temos: y ~ v(x) • Qn u(x) e z ~ Y e t(x)

[u(x )]v(x )

2'?)

então: f'(x)

~

z'(y)

y'(x)

~

eY • [v'(x ) • Qn u(x)

+ v(x)

Calcular as deri vada s de f(x) Tem os:

= f(s) (xl

Calc ular as deri vada s de Tem os:

=

f(x)

=

O

sen 2x.

f'(x} = 2 • cos 2x

,~W· u'(x )]

e finalmente:

f"(x }

~

[ f'(x) ~ [u(x) ]V () u'(x) ] x • v'(x) • Qn u(x) + v(x) • ~Txf

f'''(x )

=

H.16 9 Obte r a deriv ada da funç ão f(x)

3x 2 + 5x + 6.

-4· sen 2x

=

2

2

-8 • cos 2x = 2 3

rr • cos (2x + 2) •

cos (2x + rr)

f(n) = 2 n • cos (2x + (n- 1 )rr

~ (c os xIx.

2

Solu ção Emp rega ndo a regra que acab a de ser dedu zida , vem: f'(x) ~ (cos x)x. = (cos xix •

[1 • Qn cos x + x.

-sen

X] ~

cos x

H.17 0 Obte r a deriv ada de cada uma das seguintes funç ões: 2 a) f(x) (sen x) (x ) 3 b) f(x) x (x )

cl t(x)

)e

EXE RCrC IOS

(Qn cos x - x • tg x)

X )

d) t(x) = (ex)t g 3x

H.17 1 Calcular as derivadas suce ssivas para cada uma das seguintes funções: a) t(x) = x 4

+ 5x 2 + 1

b) t(x) =.1-.

x cl f(x) =e x d) t(x) = e-x e) t(x) = cos x

150 -H 151 -H


H.172 Um ponto móvel sobre uma reta tem abscissa s dada em cada instante t pela lei s = a. cos(w t + <,O) onde a, W e <,O são números reais dados. Determinar:

a) bl c) d)

a lei que dá a velocidade do ponto em cada instante; a velocidade no instante t = O; a lei que dá a aceleração do ponto em cada instante; a aceleração no instante t = 1.

CAPÍTULO VIII

H.173 (EPUSP-67) A função y = A • sen kx, com A> O, e sua derivada segunda y" satisfazem identicamente a igualdade y" + 4y = O. O valor da derivada primeira y', para x = O, é 12. Calcular as constantes de A e k.

ESTUDO DA VARIAÇÃO DAS FUNÇÕES Neste capítulo mostraremos algumas aplicações das derivadas. Veremos que, a partir da derivada de uma função, muitas conclusões podem ser tiradas sobre a variação da função e, portanto, sobre seu gráfico.

I.

MAxlMOS E MrNIMOS

159. Definições

I) Seja a função f: D ..... IR e seja Xo E D. Chamamos vizinhança de Xo um intervalo V = ]xo - fi, Xo + fi[, onde fi é um número real positivo. 11) Dizemos que Xo é um ponto de máximo local de f se existir uma vizinhança V de Xo tal que

=

(Vx)(x E V f(x) ~ fIxo)) Neste caSl), o valor de f(xo) é chamado máximo local de f. 111) Dizemos que Xo é um ponto de mínimo local de f se existir uma vizinhança V de Xc tal que ('Itx)(x E V

== f(xl

;;. fIxo))

Neste caso, o valor de f(xo) é chamado mínimo local de f. IV) Dizemos que .'0 é um ponto extremo ou um extremante se Xo for um ponto de máximo local ou de mínimo local de f. Neste caso, o valor de f(xo) é chamado valor extrer'o de f. 152-H 153-H


160. Exemplos

4':1)

f(x)

f(x I

a, X2, X4 e b são pontos de mínimo lo· eais de f, enquanto Xl, X3 e Xs são pontos de máximo locais de f.

x ~ O é ponto de máximo local da função f(x) ~ 1 - x 2 ; O máximo local de f éf(0)~1.

x

Os pontos de máximo ou minimo locais que não são extremos do intervalo em que a função está definida são chamados pontos de máximo ou mínimo locais interiores. No 4<;> exemplo, X2 e X4 são pontos de minimo locais interiores. f(x)

As noções de máximo e m(nimo locais referem-se a uma vizinhança do

f(x)

ponto considerado. Na função represen· tada ao lado, existe uma vizinhança V, de x, em que f(x) <:;; f(x,), 'ifx, por outro lado, existe uma vizinhança V 2 de X2 em que f(x);:;" f(x 2 1, 'ifx. Isto leva à conclusão (aparentemente contraditória) de que x, é ponto de máximo local, x: é ponto de rn(nimo local e f(x,) <

x ~ O é o ponto de m(nimo local de f(x) ~ Ixl; om(nimo local de f é f(O)~ ~

O.

<

f(X2)'

161. Definição 11

X

="2

é ponto de máximo local de f(x)

sen x, o máximo local de 11

sen"2

x

~

f(x) f(

154-H

3" 2 c

}11 ) 2

=

11

é f(-) 2

= =

1. é ponto de m(nimo local de

sen x; sen

o m(nimo local de

311

2

=

-

1.

f

é

Dizemos que f(xo) é um valor máximo absoluto de f se f(xo);:;" f(xl para todo x do dominio de f, isto é, f(xo) é o maior valor que f assume. Dizemos que f(x o ) é um valor mínimo absoluto de f se fIxo) <:;; f(x) para todo x do dom(nio de f, isto é, fIxo I é o menor valor que f assume. Voltando aos cinco exemplos anteriores, 'temos: 10 ) o valor máximo absoluto de f(xl ~ 1 - x 2 é 1; 20 ) o valor m(nimo absoluto de f(x) ~ I x I é O; 3 0 ) f(x) ~ sen x tem um máximo absoluto que é 1 e um mlnimo absoluto que é -1; 40 )

f(xsl

e

f(b)

são, respectivamente, o máximo e o m(nimo absolutos

de f.

155-H


Interpretação geométrica Observemos que são muitas as funções que têm máximos ou mlnimos locais mas não apresentam um máximo ou mlnimo absoluto.

O teorema de Fermat garante que num extremo local interior de uma funcão . derivável f, a reta tangente ao gráfico de f é paralela ao eixo dos x.

y

f(xo)

Por exemplo, observando o gráfico

é máximo local interior

fixo) é mlnimo local interior f(x)

t(xl

ao lado, vemos que a e c são pontos de máximo local, b e d são pontos de mlnimo local, porém, a função não tem máximo absoluto nem mínimo absoluto.

Xo

162. Teorema de Fermat

Se f:D

->

é uma função derivável no ponto Xo E D e Xo é ponto exf'(xo) = o.

IR

tremo local interior de f, então

Demonstração Suponhamos que Xo seja ponto de mlnimo local interior de f. Existe uma vizinhança V de Xo tal que, para todo x E V, tem·se: f(x) - fixo) f(x o )

~

f(x)

O para

x

<

Xo

flx) - fixo) - - - - - ;:;, O para x - Xo

x

>

Xo

x - Xo

=

Sendo f derivável em lim

f(x) - f(xo)

x+ Xo

X -

xo,

~

Xo

que coincide com os limites laterais à esquerda e à direita de xo. teorema da conservação do sinal para limites, temos: f(xl - fIxo I x -- Xo

=

f'(xo)

~

O

lim

Lembrando do

J

f'(xo) f(x) - f(xo)

O

Observemos, porém, que o reei' proco do teorema de Fermat é falso, isto é, existem funções f deriváveis no ponto Xo do seu domínio, f'(xo) = O e Xo não é ponto extremo de f. É o caso, por exemplo da função f(x) = (x- 1)3. Sua derivada é f'(x) = 3(x _ 1)2, então f'(1) = O e 1 ní\'o é ponto extremo.

x

Observemos ainda que o teorema de Fermat não exclui a possibil idade de Xo ser ponto extremo sem que se tenha f'(xo) = Isto pode ocorrer se f não é derivável em xo. Por exemplo, O é pon· to de mínimo de função f(x) = I x I e não existe f'(O).

t(x)

= Ix

I

o.

x

f'(xo);:;'O

x+x~

x - Xo

Se

for ponto de máximo local de f, a demonstração é análoga.

Xo

x

existe e é finito o limite:

f'(xo I

=

x

157-H 156-H


)

11.

DERIVADA - CRESCIMENTO -

DECRESCI MO

f(x)

163. Neste item vamos provar alguns teoremas que terminam por estabelecer um elo de I igação entre a derivada de uma função e crescimento ou decréscimo desta.

f(al=f(b)

164. Teorema de Rolle Se f é uma função continua em la, bJ. é derivável em la, bl e f(a) então existe ao menos um ponto Xo E la, b[ tal que f'lxo) ~ O.

=

f(b), i

Demonstração f é constante em la, bl Neste caso f'(x) = O em la. b[, isto é, para todo f'(xo)

=

20

Xo E la, b[ temos

caso:

Interpretação geométrica

x

165. Teorema de Lagrange ou Teorema do valor médio Se

f(xl

flxl

f é uma função contínua em

existe ao menos um ponto

f(al

x

=

O

e, pelo teorema de Rolle, existe

1a, bl,

então

fia) cf flb) ~

Observemos que: I) 9 é continua em

[a, bl

111)

Ix - a)

portanto,

f(x) - f(al -

9 é derivável em

la, bl

nos extremos do intervalo

la, bJ.

=

f(bl - f(al fia) - fia) - - b _ a "

g(b)

~

f(b) - f(al

=

g(bl =

(x - a)

f(xl - fiai

e

la, b];

e sua derivada é g'lxl

g(al

g(a)

f(bl - f(al b - a

~-----

por ser a diferença entre

que são contínuas em

i

-+-----+l-+-lo-L--x

Xo E la, bl

b - a

glx)

i

a

tal que

f(bl - f(al b - a

f(b)

Consideremos a função

11)

I

e derivável em

0= f(b)~

f'lxo)

2'? caso:

b - a

fiai = flb

=

f(b) - fia) b - a

Neste caso

~=--!l~1 .

tia) '= f(bl -

Xo E: la, bl

la, bl

Demonstração

tal que

O teorema de Rolle afirma que se uma função é derivável em la, bl, contínua em la, bl e assume valores iguais nos extremos do intervalo, então em algum ponto de la, bl a tangente ao gráfico de f é paralela ao eixo dos x.

158-H

I

b

o.

f não é constante em la, bJ Neste caso existe x E la, bl tal que f(xl cf f(a) = f(b). Como f é continua em la, bJ. f tem um mínimo e um máximo em la, bl. Se existe x E la, b[ tal que f(x) > fia) = f(b), então o valor fia) = f(b) não é O máximo de f em [a, bl, por· tanto, f assume valor máximo em algum ponto Xo E la, bl e, sendo f, derivável em )a, bl, temos f'(x o ) = O. Se existe x E la, bl tal que f(x) < f(a) = f(b), a prova é análoga.

\

Xo

a

10 caso:

~

f'lx) _ flbl - f(a) ; b - a

temos:

Ia - ai ~ O

f(b) - f(al • (b _ a) b - a

O

o. 159-H


Nos exercícios H.175

que

Sendo assim, é válido para g o teorema de Rolle: existe g'(x o ) O, isto é, f'(xo)

g'(x o )

Xo E la, b[ tal

2x 2 - 3x + 1 3x 4

H.175I1xl

f(b) - f(a) - O - --b ---a---

ou ainda,

a

H.177 verificar que hipóteses do teorema de Relle estão sa-

tisfeitas pela função f no intervalo I.

X+3

se

{ 7- x

se

e

<2

x

H.176 flx)

f(b) - fia) b - a

f'(xo)

e

H.177flxl = 1 - Ixl

[-3, 7]

1= [-1, 1]

e

H.178 O recíproco do teorema de Rclle não é válido. Dar exemplos de funções para as quais a tese do teorema é válida, porém, uma das hipóteses não é. Nos exercícios H.179 a H.la1 verificar que as hipóteses do teorema de Rolle são satisfeitas pela função f no intervalo I. Em seguida, obter um c E I que satisfaça a tese do

Interpretação geométrica Segundo o teorema de Lagrange, se f é função contínua em [a, b] e derivável em la, bl, então existe um ponto Xo E la, b[ tal que a reta tangente ao gráfico de f no ponto P( xo, f( Xo )) é paralela à reta determinada pelos pontos A(a, fia)) e B(b, f(b)), por terem coeficientes angulares iguais.

teorema.

a

Xo

b

x

H.179 f(x)

x 2 - 6x + 8

H.180 flx)

x 3 - 2x 2 - x + 2

H.181 f(xl

x 3 - 16x

e

I

=

[2, 4) 1=[-1,2)

e

[o, 4]

e

H.182 Dada f(x)::::: x 3 + 3x 2 - 5, verificar que as condições para validade do teorema do valor médio estão satisfeitas para a ""- -1 e b.::= 2. Encontrar todos os números (k, f(21 - f(-1) [ tal que f'(ol = (2) _ (-11 oE ] -1,2, Solução

EXERCICIOS H.174 Dada

Notemos que f(x)

4x 3 - 9x 2 + 5x, verificar se estão satisfeitas as condições para validade

do Teorema de Rolle em cada um dos seguintes intervalos:

5

lo, 2].

Determinar um número

(k

[O, 1], [1,

~) e 2

em cada um desses intervalos de modo que

f'(oo = O.

f

é derivável e continua em IR,

portanto, também é no intervalo

[-1, 2]. f'lxl = 3x 2 + 6x. Então:

Sua derivada é

f'loo = 1(21 - li-li 2 - 1-11

=

=

ou

O

c

..j2

-1 +

Como queremos

(k

302 + 60 = 15-(-31... 2 - (-1)

=

30 2 +60-6

0=

0=-1-V2.

no intervalo ]-1,2(, só convém

a=

-1

+..j2 .

Solução Notemos que f é derivável e contínua em IR, portanto, também é nos intervalos dados.

5

Temos 1(01 = O, f(11 = O e f("2) = tervalo

[O, 1]. Determinemos

O

75 4'

E [O, 1] tal que f'(ol = O:

I'(x) = 12x 2 -18x+5 = O = x =

portanto,

160-H

a -::

9 -

vS1

12

Assim, o teorema de Rolle é válido só no In-

Nos exercícios H.183 a H.186 verificar que as hipóteses do teorema de Lagrange são no intervalo I. Em seguida, obter um c E I que satisfaça a satisfeitas pela função tese do teorema.

9+-../21 12

ou

x =

9--../21 12

H,183flxl = x 2 + 2xH.184f(xl =

W

e

e =

I

=

[O, 1)

[O, 1)

161- H


H.185 f(xl

~

H.186 f(x)

H.187 Dada

v' 100 - x 2 x 2 + 4x x f(x)

x

=:;

e

I

=

[-a,

e

I

=

[2, 6]

2/3

166. Lembremos agora os conceitos de função crescente e de função decrescente num intervalo I.

6]

y

esboçar o gráfico de f e mostrar que não existe um número

a,

f(3) - f(-3) (31 _ (-31 . Qual a hipótese do teorema do valor médio

a EC ]-3, 3[ tal que f'(al que não se verificou?

Solução

Uma função f: D -> IR é crescente num intervalo I (I C Dl quando. qualquer que seja X, E I e X2 E I, temos: Xl

<

X2

=

f(X,)

<

f(X2)

I(xl

Xl Dando valores a

x

X

e calculando os

correspondentes valores de tlxl, podemos obter pontos e esboçar o gráfico ao lado.

y

Uma fu nção f: D -> IR é decrescenquando, qualquer que seja XI E I e X2 E I, temos:

te num intervalo I (I C D) T emas f '( X 1

2 =3'

X

f(3) - 1(-31

f'(al

3 - (-3)

e não existe

a

enta-o·.

-1/3

2 J

2

3 /

= -3-~ =

2 3 _ (_31 /

3 - (-31

O X

satisfazendo esta última igualdade.

A função f é contínua em IR mas não é derivável no ponto x:: O que está no intervalo ]-3,3[. Isto invalida uma das hipóteses do teorema do valor médio. Nos exercícios H.18B a H.190 verificar que hipótese do teorema de Lagrange não está satisfeita pela função f na intervalo I.

4 H.188 tlxl = (x _ 3)2

e

I ~ [1, 6]

H.189 f(x)

e

I

=

se

X

{ 8 - 3x

se

x;;:>

e

H.190 f(x)

162-H

1

x, - X 2

Xl, X2 E I, com x, *- X2' pois numerador e denominador têm necessariamente sinais iguais. Podemos também dizer que f é uma função decrescente num intervalo quando, aumentando o valor atribuído a x, diminui o valor de f(x).

[1, 2]

<1

3X + 2

Podemos também dizer que f é uma função crescente num intervalo quando, aumentando o valor atribu ido a x, aumenta o valor de f(x). f(xtl - f(X2) _ _ Notemos ainda que se f e crescente entao > O para todos

I ~ [-2,4]

X2) - .f(XI) f(. d a que se f e'd ecrescente, entao - - -- - < O para todos N otemos aln Xl - X2 Xl, X2 E I, com Xl *- x2, pois numerador e denominador têm necessariamente si· nais contrários.

163-H


167. Teorema y

Seja f uma função contínua em [a, b] e derivávei em ]a, b[. Então: I) f'(x) ;;. O em la, b[ <=<> f é crescente em [a, b] 11) f'(x) ,;;; O em la, b[ <=<> f é decrescente em [a, b]

Demonstração 1é! parte:

-==

11) Uma função f ser decrescente em [a, bJ. quando f é derivável, equivale a f'(x),;;; O para todo x E la, b(. isto é, os coeficientes angulares das retas tangentes ao gráfico de f são não positivos. x

I) Seja Xo E I = la, b[. Dado um outro ponto x E I, consideremos o quo-

I . f(x) - f(x o ) C f . . . f . ciente . on orme vimos no Item antenor, se e crescente em ,este x - Xo quociente é positivo. De acordo com o teorema da conservação do sinal, decorre f(x) - f(xo) que lim f'(xo) ;;. O. X-1> X o X - Xo 11) Pode-se provar analogamente.

2é! parte: I) Sejam

168. Exemplos 1Çl)

te em IR. Vx E IR.

= Xl, X2 E [a, b]

f(x)

A função f(x) = 2 é constanSua derivada é f'(x) = O,

com

x

Como [Xl, X2] C [a, bJ. f é contínua em [Xl, X2] e derivável em ]x l , X2 [. De acordo com o teorema de Lagrange, existe Xo E ]Xl' X2 [ tal que f(X2) - f(XI) = (X2 - Xl) • f'(xo). X2 - Xl Sendo f'(x);;. O em la, b(. decorre f'(xo);;' O. Como X2 - Xl > O, vem: f(X2) - f(XI) ;;. O, isto é: f(X2);;' f(XI) e, portanto, f é crescente. 11) Analogamente.

f'(xo)

= f(X2) - f(XI) , isto é,

2Çl) A função f(x) = x 3 é crescente em IR. Sua derivada é f'(x) = 2 = 3x ;;. O, Vx E IR.

x

Interpretação geométrica O teorema acaba de mostrar que: I) Uma função f ser crescente em [a, bJ. quando f é derivávei, equivale a f'(x) ;;. O para todo x E la, b(. isto é, os coeficientes angulares das retas tangentes ao gráfico de f são não negativos.

f(x)

3Çl)

A função f(x)

=..!.X

é decres-

cente em qualquer intervalo que não contenha o zero. Sua derivada é X

f'(x)

=-2.<0 x2 '

"txE IR'.

\

~

x

164-H 165-H


H.193 Para que valores de x é decrescente a função f(xl

f(x

4C?) A função f(x) = x 2 - 4 é decrescente em qualquer intervalo contido em IR _ e crescente em qualquer intervalo tal de IR+. Sua derivada é f'(x) = 2x que:

Solução Vamos definir f através de várias sentenças. Como primeiro passo, temos: f(x)

x

f(x) =

5C?l A função f(x) = x 3 - 3x 2 tem derivada f'(x) = 3x 2 - 6x, então:

f(x)

f'(x) = x

portanto:

f é decrescente

x ,,;;;; O ou x;;;' 2

<=*

O ,,;;;;

X ,,;;;;

2x-4 -2x + 4, { 2x +4, -2x - 4,

2, se

=

<=*

{12X -41. se x;;;' O \-2x-4L se x <O

se x;;;'2 seO";;;;x<2 se -2 x O se x ,,;;;; -2

< <

A derivada de f é, portanto:

x ,,;;;; O ou x;;;' 2 f'(x) ;;;. O O ,,;;;; x ,,;;;; 2 = f'(x) ,,;;;; O

é crescente

~

e finalmente vem:

f'(x) ,,;;;; O se f'(x) ;;;. O se

f

~ 12 .lxl-41?

{

-2

< x < O ou

x

>2

se O < x < 2 ou x < -2

-2,

e não é definida para x = O ou 2 ou -2. Assim, f é decrescente para x pertencente ao conjunto [0,2) U )-

00, -

21.

H.194 Determinar o conjunto dos valores de x para os quais cada função abaixo é decres· cente:

2

f(x) ~ 3x 4 + 4x 3 g(x) ~ eX - x,

-

12x2 + 1;

h(x) = 3x - 2

x+ 1 '

j(x)

EXERCICIOS

H.191 Determinar o conjunto dos valores de x para os quais a função flxl

~

x2

-

~

arc sen x.

lagex é Em cada um dos exercícios de H.195 a H.203, determinar os intervalos em que f é crescente e os intervalos em que é decrescente.

crescente.

Solução Devemos calcular a derivada de f e determinar em que conjunto a função f' é não negativa. Temos:

H.195flx) = x3

1 2x 2 - 1 f'(x) = 2x - - = - - -

H. 196 f(x) = x 4 + 4x

x

f '(x) ::;"0 -

x

=- 2x 2x -1 -::;"= O --

..[2 "'" <;;: < O -2x ou

..[2 x ::;" '" -2-..[2

Lembrando que D(f) = IR;, vem a resposta: f é crescente para x;;;' -2H.192 Determinar o conjunto dos valores de x para os quais cada função abaixo é crescente. f(x) g(x) h(x)

~ ~

~

i (x) =

~66-H

2x 3 - 15x2 - 84x + 13 2 • cos x - x + 1 sen x - cas x

Iix I -

1I

9x 2 + 15x - 5

-

H.197 flx) ~ x s - 5x 3 H.198f(x)

+ 20x - 2

~ x +-! x

H.199flx) = x ~ H.200 flx) = 2 -

~

H.Z01 f(x) = {2X + 1, se x";;;; 2 7 - x, se x 2

>

167-H


H.2D2 f(x)

~

2 - 1, se {x2 x + 2x - 3,

2 + x, x H.2D3 flx) = { x, 2x - x 3 ,

x ~ 1

111.

se x > 1

se x < O se O<x<l se x> 1

H.204 Estudar a função f:IR+ ~ IR tal que f(x) = logex + 109 Ix + 21 determinando os intervalos em que é crescente ou decrescente. e, ~

f:IR

H.20G Descrever o crescimento e o decréscimo da função = 2

+ 3 • sen(2x

f:

IR

[O.21r]

tal que:

-+ IR

_!!.) 3 .

H.207 Determinar para que valores de é x crescente ou decrescente a função f: f(x) = eCos x.

tal que

~ -+

f(x)

R tal que

DETERMINAÇÃO DOS EXTREMANTES

169, Dada uma função f, definida e derivável em ~ la, bL o teorema de Fermat garante que os valores de x que anulam f', isto é, as raízes da equação f'(x) ~ O são possivelmente extremantes de f. Assim, por exemplo, os possíveis extremantes da função f(x) ~ x 4 - 4x 3 são as raízes da equação f'(x) ~ 4x 3 - 12x 2 ~ O, isto é, O e 3. Em princípio, tanto O quanto 3 podem ser ponto de máximo ou ponto de mínimo ou não ser extre· mante. Com toda certeza nenhum número diferente desses dois é extremante por não anular f'. A questão agora é saber qual das alternativas é correta para O ou para 3.

170. Mais geralmente, dado um número Xo E la, b[ tal que f'(x o ) ~ O, como determinar se Xo é ou não extremamente de f e ainda, sendo extremamente, como saber se Xo é ponto de máximo ou de mínimo?

Ti! resposta:

Xo é ponto de máximo local de f se existir uma vizinhança Xo tal que f'(x) é positiva à esquerda e negativa à direita de x o . De fato, se eXistir uma vizinhança V de Xo com a propriedade citada, temos para todo x E V:

V de

H.20S Provar que se

então a função

H.209 Provar que o polinômio

f(xl

Sugestão: calcular

f(x),

lim X~-OO

'=

f(x) = -xsen x e, decrescente.

3x s + 5x 3 + 1

lim x++

H.210 Esboçar o gráfico de uma função f

f(x)

x < Xo X> Xo

admite um único zero real.

e estudar

f'(x).

e então,

00

H.211 Provar que se f é uma funça"o crescente em I, enta-o

> O =< O ==

f(x) crescente f(x) decrescente

9

=

-f é decrescente em I.

< x2- Temos:

=-íix,1 ;;'-flx2)

== f(x)

,,;; f(xo)

-=- f(x) < f(xo)

é um ponto de máximo local. y

o gráfico ao lado mostra que, numa vizinhança de um ponto Xo de máximo local, as retas tangentes à curva passam de coeficiente angular positivo (à esquerda de xo) para negativo (à direita de Xo l. E o coeficiente angular é justamente a de· rivada de f.

Solução

Xl <x2 =f(xt) <flx2)

f'(x)

== f'(x)

• para a qual sao verificadas as seguintes hipóteses:

a) f é continua em IR b) f(3) = 2 c) f'(x)=-l Se x<3 f'lx) = 1 Se x > 3

Sejam Xl E I e X2 E I com x,

Xo

-=-

=g(xl) ;;'g(x2)

então 9 é decrescente em I.

xo H.212 Provar que se f é uma função crescente em I ' 1 e h e definida em I pela lei = f0J ' então h é decrescente em I.

x

h(x) =

2i! resposta:

Xo é ponto de mínimo local de f se existir uma vizinhança Xo tal que f'(x) é negativa à esquerda e positiva à direita de xo. De fato, se existir uma vizinhança V de Xo com a propriedade referida, te· mos para todo x E V: V de

H.213 Provar que se f é crescente num intervalo I g é crescente em I e existe tOg é crescente em I. '

168-H

to g,

enta-o

..

169-H


x

< Xo

=> f'(x)

Xo

=> f'(x)

x> e, então,

Xo

<o = > o =>

f(x) decrescente f(x) crescente

=> f(x) ;;. f(xo)

o

=> f(x) ;;. f(xo)

v

3

x

+

+

+

x-3

-

-

+

f'(x)

-

-

+

x

é um ponto de mínimo local.

2

o gráfico ao lado mostra que,

numa vizinhança de um ponto Xo de mínimo local, as retas tangentes á curva passam de coeficiente angular negativo (á esquerda de xo) para positivo (á direita de xo).

x

xo

3'! resposta: Xo não é extremamente de f se existir uma vizinhança V de tal que para todo x E V e x 1= Xo tem-se f'(x) sempre com mesmo sinal. A figura 1 mostra que se existir uma vizinhança V de Xo tal que f'(x) > O para todo x E V e x 1= xo, então Xo não é extremamente. A figura 2 mostra, analogamente, para o caso em que f'(x) < O, que Xo não é extremamente. Xo

v

Existem vizinhanças de O em que f'(x) < O, portanto, O não é extrema· te de f. Há vizinhanças de 3 em Que f'(x) passa de negativa a positiva, isto é, 3 é ponto de mínimo local. O gráfico ao lado ilustra como varia a função f.

v =

2<:» Quais são os extremantes da função f: ]0,27T[ ..... IR dada por f(x) 2 sen x + cos 2x? Calculando a derivada:

=

f'(x) = 2 • cos x - 2 • sen 2x = 2 • cos x - 4 • sen x • cos x = = 2 • cos x • (1 - 2 • sen x) Os valores de x que anulam 1

e sen x

.

7T

37T

o

x

171. Exemplos Verificar se f(x) = x 4 - 4x 3 tem extremante. Já vimos que f'(x) = 4x 3 - 12x 2 tem raízes O e 3. Analisemos a variação de sinal da função f'(x) = 4x 3 - 12x 2 = 4x 2 (x - 3): 1<:»

170- H

57T

= 2"' Isto e, 2"' ""2' 6 e 6'

Analisando o sinal de x

são as raízes das equações cos x = O

f'(x)

7T

o

f'(x),

7T/6

temos: 7T/2

37T/2

S7T/6

27T

cos x

+

+

-

-

+

1-2 sen x

+

-

-

+

+

f'lx)

+

-

+

-

+

x

171-H


'f'

Ven Icamos que

6"11

e

6511

11

311

são pontos de máximo local, enquanto 2 e 2

são pontos de mínimo local. O gráfico da função f confirma nossa análise.

H.222 flxl = x • Qn x H.223 flx) = Qn Ix 2 + 11

H.225 Calcular o valor máximo assumido pela função Solução f'(x) ~ -2(x-a) • e-lx-a)2 f'(x) ~ O = -2Ix-a) • e-lx-a)2 Como

e-(x-a)2 > O para todo ==>x-a<O =x-a>O

x <a x >a

O

x E IR,

==> x

=

a

temos:

==>f'(xl >0 ==>f'(xl <O

a é um ponto de máximo local de f. O valor máximo de f é: 2 fia) = e-la-aI = eO = 1. Assim

x

=

flxl = e- lx - a )2.

=

Nos exercícios H.226

a

H.231 , calcule os valores extremos de f:

H.226 flx) = x2 - 4x - 1 =

H.228f(x)

=~

H.229 flx)

=

x

EXERCICIOS

a

H.224, determine os extremantes da função f.

H.214 flx) = _x 2 - 5x - 4 H.215 f(x) = 2x 2 - 8x + 11 H.216 f(x) = x 3 - 27x + 1 H.217 flx) = _x 3 + 6x 2 - 12x + 8 H.218 flx) = (x_8)3 (x_6)4

=

1 x2 + 5x _ 6

H.220 flx} = cos 3x H.221 f(x) = sen (2x -

172-H

x2e

x

x

Nos exercícios H.214

H.219 flx)

x 4 + 8x

H.227 f(x)

!!4 )

-x

e - e H.230 flx) = eX + e-x H.231 flxl

=

(x _ 1)2/3

Nos exerclcios H.232 a H.235 determinar as coordenadas dos pontos extremos da função f.

H.232 flx)

=

x3

-

9x

1 + x - x2 H.233 flxl = 1 _ x + x2 1 - x3 ---;(2

H.234 flx)

=

H.235 f(x)

=--;;z

Qn x

H.236 Calcular a e b de modo que a função flx) = x 3 + ax 2 + b tenha um extremo relativo em (1, 5),

173-H


H.237 Obter os extremos absolutos de I(x) = x 3 + x2 - x + 1 no intervalo [-2,

H.241 Um móvel desloca-se sobre um eixo de modo que sua abscissa s no instante t é dada por s = .a • coslkt + .1'), sendo a, k, .I' constantes dadas.

~ ].

Determinar:

Solução Como f é derivável em [-2, ~ f'(x)

= 3x 2

+ 2x - 1

= 3(x

l. apliquemos o teorema de

+ l)(x

1 -"3)

a) instantes e posições em que é maxima a velocidade do móvel; b) instantes e posições em que é mínima a aceleração do móvel.

Fermat:

e os zeros de f' são os números -1 e

H.242 Um triângulo está inscrito numa semi-eircunferência de raio R. Seus lados medem a,

3"1

b e 2 R. Calcular a e b quando a área do triângulo é máxima.

Analisando a variação de sinal de f', temos -1

f'(x)

+

Solução 1/3

I

I

O

O

I

I

x

+

b 2R

2R

2R

então -1 é ponto de máximo interior e 1/3 é ponto de ml'nimo interior. Calculemos o

valor de f nesses pontos críticos e nos extremos do intervalo [-2, f(-2) = -S +4 + 2 + 1 = -1, fl-l) 1111

f(

o

3')

27 + 9

- 3"

22 + 1 = 27

=

-1 + 1 + 1 + 1

2"1 ]:

Notemos primeiramente que numa semi-circunferência de raio R é possível inscrever diferentes triângulos, todos retângulos. Observemos que a e b, medidas dos catetos, variam de um triângulo para outro e percorrem o intervalo jo,2RL isto é, O a 2R e O b 2R. Para um mesmo triângulo são verificadas as seguintes relações:

= 2,

= ~ + ~ - .!- + e fi ~) 2 S 4 2

7

8"

S y

valor máximo absoluto de f

tervalo

1

1

e

fH

L

só la ou b). Eliminando b, pois S

portanto, é

f H) = 2.

a2 + b2

e

= 4R2

1

=- • 2

ab

1

=- • 2

t.:~~

a V 4R2 - a 2

b = Y 4R2 - a 2 , temos: 1

=- • vi 4R2 a 2 2

- a4

Provemos que 5 tem um ponto de máximo:

O valor mínimo absoluto de f no in-

e

1 1("31.

portanto, é

1(-2) = -1. O gráfico da função ilustra o exposto.

SR2 a - 4e 3

1

tervalo [-2, ~] é o menor dos números

1 1(-2), 1(2')

ab

=""2

onde 5 é a área do triângulo. Para determinarmos o máximo de S devemos colocar S como função de uma variável

no in-

[-2'"2] é o maior dos números

1(-21. f( 2')

< <

< <

2R2 a - a 3 - ,4

S'

="2 • 2Y4R2 a 2 - a4 = Y4R2 a 2

S'

=O

=-2R2 a - a 3

=O

=-a

=

RY2

< a < R Y2 =- a 2 < 2R2 =- a 3 < 2R2 a =- S' > O R Y2 < a < 2R =- 2R2 < a 2 = - 2R2 a < a 3 =- S' < O

O

x

e, então, Conclusão:

H.238 Obter os extremos absolutos de f(x) = (x - 2)2/3 no intervalo [1,5]. 1

=

a

=

R

Y2 é um ponto de máximo

local.

o triângulo de área máxima é aquele em que

y 4R2 - 2R2

~ R.../2,

a

RY2

e

b

isto é, é o triângulO isósceles.

x _ 2' obter os extremos absolutos de f no intervalo [2,6].

H.243 Um retângulo de dimensões x e y tem perímetro 2a (a é constente dadal. Determinar x e y para que sua área seja máxima.

H.240 Uma pedra é lançada verticalmente para cima. Sua altura h (metros) em relação ao solo, é dada por h = t 3 - 3t 2 - 9t + 1, onde t indica o número de segundos decorridos

H.244 Calcular o perímetro máximo de um trapézio que está inscrito numa semi-circunferência de raio R.

H.239 Dada a função

f(x)

após o lançamento. Em que instante a pedra atingirá sua altura máxima?

174-H

175-H


H.245 Calcular o raio da base e a altura do cilindro de volume máximo que pode ser inscrito numa esfera de raio R.

Solução

H.248 Um fabricante de caixas de papelão pretende fazer caixas abertas a partir de folhas de cartão quadrado de 576 cm 2 , cortando quadrados iguais nas quatro pontas e dobrando os lados. Calcular a medida do lado do quadrado que deve ser cortado para obter uma caixa cujo volume seja o maior possível.

A figura ao lado é uma secção da esfera e do cilindro inscrito, feita por um plano contendo o eixo de simetria do cilindro. Observemos que numa esfera podem ser inscritos diferentes cilindros, portanto, h r e 2" são variáveis. Para um dado cilin-

H.249 Uma ilha está em um ponto A, a 10 km do ponto B mais próximo sobre uma praia reta. Um armazém está no ponto C, a 7 km de B sobre a praia. Se um homem pode remar à razão de 4 km/h e andar à razão de 5 km/h, aonde deveria desembarcar para ir da ilha ao armazém no menor tempo posslvel? H.250 Um fio de comprimento L é cortado em dois pedaços, um dos quais formará um círculo e o outro, um quadrado. Como deve ser cortado o fio para que a Soma das áreas do c(rculo e do quadrado seja máxima?

dro são verificadas as seguintes condições:

v

1Tr 2 h,

< h < 2R

O

e

h2 4

r2 + -

H.251 Um funil cônico tem raio r e altura h. Se o volume do funil é V (constantel, calcular a razão r/h de modo que sua área lateral seja mlnima.

R2 onde V é o volume do cilindro. Para determinarmos o máximo de V devemos colocar V como função de uma variável h2 sô Ir ou hl. Eliminando r, pois r 2 ~ R2 -""""4' temos:

H.252 Um fazendeiro precisa construir dois currais lado a lado, com uma cerca comum, conforme mostra a figura. Se cada curral deve ter uma certa área A, qual o comprimento mínimo que a cerca deve ter?

x

x

y

y

x

~ 1TR2

V'

O

31Th 2

4 31Th 2

==>~4-

O< h <

2R

4R2 ==> h 2 < -3-

Y3

~<h

<2R

e, então,

h =

Y3

Conclusão:

=

Q. Se os lados e o fundo têm largura Q/3, calcular o àngulo de forma que a

e

2R

h =

Y"'J 31Th 2 4

==>~-

calha tenha a máxima secção reta.

< 1TR2

=V'>O

=V'<O 2R

~

x

H.253 Uma calha de fundo plano e lados igualmente inc1 inados vai ser constru ída dobrando-se uma folha de metal de largura

Provemos que V tem um ponto de máximo:

V'

y

172. Um outro processo, para determinar se uma raiz Xo da equação f'(x) = O é extremamente da função f, consiste em estudar o sinal da derivada segunda de f no ponto xo. O teorema seguinte explica o processo.

é um ponto de máximo local.

o cilindro de volume máximo é aquele em que

h

2R = 3Y

e r

2R = 0"

173. Teorema H.246 Calcular o raio da base e a altura do cone de área lateral máxima que é inscritível numa esfera de raio R. H.247 Calcular o raio da base e altura do cone de volume mínimo que pode circunscrever uma esfera de raio R.

Seja f uma função contínua e derivável até segunda ordem no intervalo I = la, bL com derivadas f' e f" também contínuas em I. Seja Xo E I tal que f'(xo) = O. Nestas condições, temos: a) se f"(xo) b) se f"(xo)

< O,

> O,

então Xo é ponto de máximo local de f; então Xo é ponto de mínimo local de f.

176-H 177-H


175. Observação

Demonstração f"(x)

a) se f"(xo) < O e f" é contínua, existe uma vizinhança V de Xo na qual O, ';f x E V.

<

Se f"(x) < O, então f' é decrescente em V, portanto, como f'(x o ) = O decorre que em V, à esquerda de Xo temos f'(x) > O e à direita de Xo temos f'(x) < O. Concluímos assim que Xo é ponto de máximo local. b) Prova-se analogamente.

Determinar os extremantes de f(x)

= x4

f(x)

-

=

4x 3

As raízes da equação f'(x) números em f"(xl. vem:

=

O

f"(3)

12 . 3 2 - 24· 3

=

= 4x 3

-

12x 2

= O são O e 3. Substituindo esses

f"(!.-)

2

=

>O =

36

f(x)

=

3

2 • sen x + cos 2x,

==

2

f"( 51T)

=

31T f"( - )

= -2 • sen -

6

2

então

178-H

1T

6"

e

cos

-2 • sen 51T - 4

6

31T

2

=

=

2 • cos x - 2 • sen 2x

3

cos

=

-2 + 4 = 2

51T

""3 = -1

- 4 • cos 31T

-2

<O >

O

= -3

= +2 + 4

51T 1T 6 são pontos de máximo e 2 e

=

O são 1T/6, 1T/2, 51T/6 e 31T/2.

~ = - 1 - 2 = -3

cos 1T

no intervalo

=

6

se n é par e f(nl (xo) < O, então Xo é ponto de máximo local de f; se n é par e f(nl(x o ) > O, então Xo é ponto de mínimo local de f; se n é ímpar, então Xo não é ponto de máximo local nem de mínimo local de f.

A demonstração deste teorema não cabe num curso deste nível.

177. Exemplo Pesquisemos os extremantes da função Calculando as sucessivas derivadas de f, f'(x) = 5x 4 - 12x 3 + 9x 2 - 2x = xIx f"(x) = 20x 3 - 36x 2 + 18x - 2 = (x f'''(x) = 60x 2 - 72x + 18 f""(x)

=

120x - 72

f(v) (x)

=

120

f(n) (x)

=

O,

para todo

<O

As raízes de f'(x) Temos ainda:

>

f'(O) f'(1)

O

31T 2 são pontos de mínimo.

e

Nestas condições, temos:

11) 111)

é ponto de mínimo.

... = f(n-l) (xo) = O

f(n) (xo) =F O

I)

-2 • sen!.- - 4

=

f"(x)

O.

2 . sen x + cos 2x f'(x) f"(x) = -2 • sen x - 4 • cos 2x

6

= 12x 2 - 24x

-

=

-2 • sen !.- - 4

6

=

12x 2

f'(xo) = f"(xo)

4x 3

As raízes de f'(x) = 2 • cos x - 2 . sen 2x Testando cada uma em f"(x), temos: f"( !!.-)

4x 3 .

-

=

Achar os extremantes de f(x)

x4

=

f'(x)

nada se conclui sobre

f"(O)

2<;» [O, 21Tl.

176. Critério geral para pesquisar extremantes Seja f uma função derivável com derivadas sucessivas também deriváveis em la, b[. Seja Xo E I tal que

174. Exemplos 10 1

Devemos observar, nas condições do último teorema, que se f'(xo) = O e f"(xo) = O, nada pode ser conclu ído sobre xo. Um teorema mais geral que o ano terior estabelece finalmente um critério para pesquisar máximos e mínimos locais sem chegar a impasse.

f'(~) 5

=

n

>

O são

= =

,,2

O e f (5")

-

3x 4 + 3x 3

-

x2 + 1

1)2 (5x - 2) 1)(20x 2 - 16x + 2)

5. O, 1

O e f"(O) = -2 < O O, f"(1) = O e f'''(1)

=

f(x) = x 5 temos:

e

6

2/5.

'* O

18 25> O

179-H


portanto: O é ponto de máximo,

~

H.264 Achar o ponto Po situado sobre a hipérbole de equação xv = 1 e que está mais próxi-

é ponto de mínimo e 1 não é ponto de máximo

nem de mínimo.

mo da origem.

Solução Seja

Po = (x, V). A distância de Po à origem é d =

bre a hipérbole,

V

=

2..x

e, então, d =

J

x2 +

-L . x2

-J x2 + v·

. Estando Po so-

Calculemos x para que d se-

ja mínima. 1

1 -~ d' ="2. (x 2 +)(2) • (2x - 2· x-3 )

x -;3

1

j

EXERCICIOS Nos exercidos H.254 a H.259 determine os extremantes da função f, utilizando o critério da segunda derivada.

d'

=

=

O

e, portanto,

Po = (1, 1)

ou

x4 - 1

= O

x2

+J....2

=

x

x

±

1

Po = (-1, -1).

H.254 fi xl = xix - 2)3 x

H.265 Achar O ponto da curva (x_3)2 + (y_6)2 = 20 que está a distância mínima do ponto (-2, -4).

x2 eX

H.266 Um triângulo isósceles de base a está inscrito numa circunferência de raio R. Calcular a de modo que seja máxima a área do triângulo.

. H.255 f(xl - 1 + x.

H.256 f(x)

x

H.257 f(x)

e +e

-x

Solução 10ge(1 + x 2 )

H.258 flx) H.259 flx)

=

Seja ABC o triângulo isósceles de base a = AB e altura h = CE. Sua área é dada pela fórmula

Ix _ 1 )2/3

S

=

H.260 Calcular as coordenadas dos pontos extremos do gráfico da função

No triângulo retângulo BCD, a altura BE é média geométrica entre os segmentos que determina hipotenusa CD, então:

Qn~

f(x)

~_2~ =

2..2 ah

(Qn x)2 -

D

H.261 Dada a função [-2,3].

f(x)= -Ix - 1)2, determinar os extremos absolutos de f no intervalo

(BE)2 = (EC)(ED) 1

H.262 Obter os extremos absolutos de flxl = x2 - 4x + 8 em [-1,3].

a2

=4"

.r=:-

S = - ah = h V 2Rh - h 2 = 2

= h •

~ a = 2 -J 2Rh - h 2

(2R - h)

-J 2Rh 3 - h4

Procuremos o valor máximo de S para

O

< h < 2R:

H.263 Dada a função f tal que: f(xl

<

=

se x 1 {x + 2 x 2 - 6x + 8 se x;;;' 1

determinar os extremos absolutos de f em

180-H

S' = O

[-6,

5].

Como

== 3Rh 2 - 2h 3 = O S = O

para

h = O ou

3R ==h=T h = 2R

e

181-H


h

3R

entao

a

2

81R4

==> S

2

Nos gráficos seguintes, a figura 1 mostra o gráfico de uma função que tem

16 h

3R 2

concavidade positiva em xo, enquanto a figura 2 ilustra uma concavidade negativa em xo.

. e. ponto de rnaximo para S e, neste caso,

~~ V!;R. Lt 2

- 9RT 4

R

v'3

y

y

H.267 Calcular o raio da base e a altura do cone de máximo volume que se pode inscrever numa esfera de raio R.

H.26B Determinar as dimensões do cone de área total rninima que pode circunscrever uma esfera de raio R.

H.269 Um fabricante de caixas pretende produzir caixas com tampa de um certo volume V, cuja base é um retângulo com comprimento igual ao triplo da largura. Calcular as dimensões mais econômicas que deve usar.

I

I

H.270 Uma página para impressão deve conter 300 cm 2 de área impressa, uma margem de 2 em nas partes superior e inferior e uma margem de 1,5 em nas laterais. Quais são as

x

dimensões da página de menor área que preenche essas condições?

H.271 Um fazendeiro tem 80 porcos, pesando 150 kg cada um. Cada porco aumenta de peso na proporção de 2,5 kg por dia. Gastam-se Cr$ 2,00 por dia para manter um porco. Se o preço de venda está a Cr$ 3,00 por kg e cai Cr$ 0,03 por dia, quantos dias deve o fa-

I

j - - 1_

-<10---

_

Xo

x

_

Um critério para determinar se um gráfico tem concavidade positiva ou negativa em Xo é dado pelo seguinte teorema.

zendeiro aguardar para que seu lucro seja máximo?

179, Teorema H.272 O custo de produção de x unidades de uma certa mercadoria é a + bx

e o preço de

venda é c - dx por unidade, sendo a, b, c, d constantes positivas. Quantas unidades devem ser produzidas e vendidas para que seja máximo o lucro da operação?

Se f é uma função derivável até segunda ordem no intervalo I ~ la, bl. Xo é interno a [a, b] e f"(xo)"* O, então: a) quando f"(xo) > O, o gráfico de f tem concavidade positiva em Xo; b) quando f"(xo) < O, o gráfico de f tem concavidade negativa em xo. Apenas mostraremos geometricamente que o teorema é válido.

v IV, CONCAVIDADE 178. Definição Seja f uma função contínua no intervalo I ~ [a, b] e derivável no ponto Xu E la, b[. Dizemos que o gráfico de f tem concavidade positiva em Xo se, e somente se, existe uma vizinhança V de Xo tal que, para X E V, os pontos do gráfico de f estão acima da reta tangente à curva no ponto Xo. Analogamente, se existe uma vizinhança V de Xo tal que, para x E V, os pontos do gráfico de f estão abaixo da reta tangente à curva no ponto xo, dizemos que o gráfico de f tem concavidade negativa.

Se f"(xo) > O, então f' é crescente nas vizinhanças de XO, portanto, as tangentes ao gráfico têm incl inação crescente e isto só possível sendo positiva a concavidade.

t--

·--+1----_ Xo

x

182-H 183-H


V.

v

PONTO DE INFLEXÃO

181. Definição Analogamente, se f"(xo) < O, então f' é decrescente nas vizinhanças de xo, isto é, as retas tangentes à curva têm inclinação decrescente, portanto, a concavidade é negativa.

Seja f uma função contínua no intervalo I = [a, bl e derivável no ponto Xo E la, b[. Dizemos que Po (xo, f(xo)) é um ponto de inflexão do gráfico de f se, e somente se, existe uma vizinhança V de Xo tal que nos pontos do gráfico f para x E V e x < Xo a concavidade tem sempre o mesmo sinal, que é contrário ao sinal da concavidade nos pontos do gráfico para x> xo. Em outros termos, Po é ponto de inflexão quando Po é ponto em que a concavidade "troca de sinal". Eis alguns exemplos: Xo

x

180. Exemplos 1'?)

Como é a concavidade do gráfico da função

f(x)

cos x, para

x E [0,211 I?

Temos f'(x) = -sen x Notando que:

f"(x) = -cos x.

e

11 O.;;; X < - ou 2

f"(x) < O

~

-cos x <O

~

f"(x) > O

~

-cos x > O

~-<x

11 2

311 2'< x .;;; 211 Retomando os exemplos do item 180, vemos que os pontos de inflexão de

311 2

<

f(x) = cos x no intervalo [0,2111 são os pontos de abscissas ~ e 3 11; os pontos de 2 inflexão da curva y = x4 - 4x 3 são os de abscissas O e 2. Os seguintes teoremas permitem localizar os pontos de inflexão no gráfico de uma função.

y

que

Concluímos 11 311 [O, 2" [ e 12" VI"d a d e

nos

intervalos

2111 a curva tem conca-

11 311 [ . negativa e no .Intervalo 1"2 ' 2"'

182. Teorema

a concavidade é positiva. Confira com o gráfico ao lado. 2'?) y

Seja f é uma funçao com derivadas até terceira ordem em I = [a, b]. Seja Xo E la, b[. Se f"(xo) = O e f'''(xo) O, então Xo é abscissa de um ponto de inflexão.

*

Como é a concavidade da curva y 4

=x

-

4x

3

=- y' = 4x

3

-

12x 2

=

x4

-

4x 3 ?

=- y" = 12x 2

-

24x

Notando que y" = 12· x • (x - 2), temos:

=-

=

y" > O concavidade positiva. x < O ou x > 2 O < x < 2 =- y" < O = concavidade negativa.

Confira com o gráfico do item 171.

184-H

Demonstrarão

Suponhamos, por exemplo, f"(xol = O e f"'(xo) > O. De acordo com o teorema do item 173, Xo é ponto de mínimo local da função f'. Assim sendo, existe uma vizinhança V ie Xo tal que: (xEV e x<xo) (x E V e x> xo)

=f"(xo)<O = f"(xo) > O 185-H


isto é. em Xo a função f" "troca de sinal". ou ainda. em Po Ixo. f(xo)) a concavidade do gráfico de f troca de sinal. portanto. Xo é abscissa de um ponto de inflexão.

Notando que f'''lx)

=

24x - 12. vemos que:

f'''(2) = 48 - 12 = 36 cF O e f'''(-ll = -24 - 12 = -36 cF O portanto.

2 e -1

são abscissas de pontos de inflexão e esses pontos são:

P = (2, f(2)) = (2. -29)

e

Q

= 1-1. f( -11) = 1-1. -26)

183. Teorema

Xo

Se f é uma função derivável até segunda ordem em I = la. bl. Xo E la. bl e é abscissa de ponto de inflexão do gráfico de f. então f"(xo) = O.

Nos exercícios H.273 a H.277 determinar onde o gráfico da função dada tem concavidade positiva, onde a concavidade é negativa e obter os pontos de inflexão, caso existam.

Demonstração Suponhamos

f"lxo) cF O;

f"(xo)

.!Jx) - f(xo).

=

lim x-+xo

EXERCICIOS

X -

por exemplo. admitamos

>

f"(xo)

> o.

Temos:

O

Xo

. então existe uma vizinhança V de

Xo

tal que

f'(x) - f'lxo) x - Xo

> O,

V x E V.

x cF xo. Assim. em V. a função f' é crescente, portanto, em V o gráfico de f tem concavidade sempre positiva. isto é, em Po Ixo. fi xo)) a concavidade não troca de sinal e Po deixa de ser ponto de inflexão.

x. + 9x

H.273 Hxl

=

H.274 fi x)

= ~

x

H.275Hxl =

~

H.276 flx) -

{! Ix 2

2x + 4).

x < 1 _ {Xx2. para 4x

H.277 Hxl -

3 _

2

+ 7x - 3. para x

~ 1

H.278 Determinar os intervalos em que x deve estar para que o gráfico da função := sen x - cos x tenha concavidade positiva.

184. Observações Este último teorema mostra que uma condição necessária para Xo ser a abscissa de um ponto de inflexão do gráfico de f é anular f". Entretanto, nem todas as raízes de f"lx) = O são abscissas de pontos de inflexão. Se uma raiz Xo de f"(x) = O não anular f· ... o teorema do item 182 garante que Xo é abscissa de ponto de inflexão. Se. porém, f"(xo) = 1''' (xo) = O. nada podemos concluir. usando a teoria dada.

H.279 Determinar as abscissas dos pontos do gráfico da função concavidade é negativa.

f(x)

==

xS - x

4

f(x)

=

nos quais a

H.280 Quais são os pontos de inflexão no gráfico da função flxl = Ix2 - 1 )(x2 - 41? Nos exercícios H.281 a H.283, supondo que f é contInua em algum intervalo aberto que contém c, faça uma parte do gráfico de f numa vizinhança de c de modo que fiquem satisfeitas as condições dadas.

H.281 Para

185. Exemplo Determinar os pontos de in!',exão do gráfico da função f:IR flx) ~ x 4 - 2x 3 - 12x 2 + 12x - 5. Temos:

-+

IR tal que

<c,

As raízes da equação

186-H

f"(x) ~ O, isto é. 12x 2

x> e. f'lxl > O e f"(x) > O e O f'lxl > O e f"(xl x

H.282 Para

<

<c.

para

H.283 f'(e)

f'(x) = 4x 3 - 6x 2 - 24x + 12 fU(x) = 12x 2 - 12x - 24

<

f'(xl O e f"(xl > O e f'(xl > O e f"(xl > O

x> e. x

para

=

f"(c)

=

O e

f"(x) > O para

x

<c

ou

x> e.

H.284 Esboçar o gráfico de uma função f tal que, para todo x real, tenhamos -

12x - 24 ~ O são 2 e -1.

f'(xl

<O

f(x)

> O,

e f"(xl > O.

187-H


H.285 Esboçar o gráfico de uma função f positivas,

H.286 Se f(xl

=;

\f x E

para a qual

f(x). f'(x)

e

f"(x)

existem e são

e) lim

IR.

f(x)

um extremo relativo em

(O, 3)

e um ponto de inflexão em

x+ -

(1, -1 I,

+ a3x3 + a4x4, calcular

aO

I

aI, a2' 33

e

34

x

~

- 35

186. Um dos objetivos da teoria deste capítulo é possibilitar um estudo da variação de uma função f. Para caracterizar como varia uma função f procuramos determinar: a) o domínio, b) a paridade, c) os pontos de descontinuidade; d) as intersecções do gráfico com os eixos x e y; e) o comportamento no infinito; f) o crescimento ou decréscimo; g) os extremantes; h) os pontos de inflexão e a concavidade; i) o gráfico.

3x 2 + 2x - 5

~

~

5 3(x - ll(x +3)

-

5

3

ou

~ x ~

g) f(x)

~

O

1

==> f'(x) ;;, O

X;;'

==> f'(x) ~

1 ou

==> x ~

5 3

x

>O 6x + 2 == { f"(- 3_) ~ -8 < O f"( 11

f"(x)

=

8

3

tem um mínimo em

então

h) f"(x) = 6x

_

Estudar a variação da função f(x) ~ x a) Seu domínio é IR. b) A função não é par nem impar pois: 3

(_X)3

+ (__ xI 2

-

+x

2

inflexao em -

5x.

==> f crescente

O ==> f decrescente

x = 1

e um máximo em

x

5 3

+ 2, então:

x

<-~

==> f"(x)

< O ==

concavidade negativa

x

> -~

==> f"(x)

> O ==

concavidade positiva

Como o sinal da concavidade muda em x =

187. Exemplos

fi-xl ~

- 00

então:

VARIAÇÃO DAS FUNÇÕES

10)

~

de modo que o

gráfico de f passe pela origem, seja simétrico em relação ao eixo y e tenha um ponto de inflexão em (1. -11.

VI.

x3

x+-oo

00

f) f'(x) a2x2

x 3 = +00

X++ 00

f(x) = lim

lim

ao + alx + a2x2 + a3x3, determinar ao,31,82 e 83 de modo que f tenha

H.287 Se f(x) '" ao + ai x +

lim

x~+oo

1

-3'

o gráfico tem um ponto de

1

-3 i) gráfico de f: f(x)

5(-x) = _x 3 + x 2 + 5x

não é idêntica a f(x) nem a -f(x). cl A função polinomial f é contínua em R. d) Fazendo x = O temos fW) = o. Fazendo f(x) = O temos x 3 + x 2 ou

x

~

-

5x ~ O, isto é, x = O ou x =

-1-V21 2

-1 +~21

--

-1-V21 --2-

2

- os pontos As intersecções com os eixos sao

(O, O) ;

(-1. - 2V21

, O)

e

21.. OI ( =-!.. +_~ 2 " 188-H

189-H


2.o )

i) gráfico de

- da f unçao - f() x -_15 . Estudar a variaçao x = 2x a) Seu domínio é

D(f) = IR - {

f: f(x)

~}

b) A função não é par nem ímpar pois:

-x -1

f(-x) = 2(-x) -5

não é idêntica a

1

f(x).

nem a

I

1

-f(x).

x c) Como g(x) = x - 1 e h(x) = 2x - 5 são contínuas, f(x) = 2 x -_ ~ é contínua em todos os pontos do seu domínio. Notemos que lim _ f(x) = -

x+t

lim

00

e

------~----

2

~IO~.1()_

-

12" I I I

f(x) = + 00. d) Fazendo Fazendo

x = O,

f(x) ~ O,

temos

temos

0- 1

fIO) = 2 • O _ 5 =

x - 1

2x _ 5 = O,

isto é,

1

5.

x = 1.

1

As intersecções com os eixos são os pontos

(0'"5)

I

--------+----+--=....;;::--+------+--1-5------.. ;

x+ .1.+ 2

I I

e

I

(1, O).

EXERCICIOS

1 e) lim x++

f(x)

lim

x

1

2x - 5

5

2

- - - = lim

X'H 00

00

x - 1

x++

00

2

Nos exercícios H.28B a H.297 determinar o domínio, a paridade, os pontos de descontinuidade, as intersecções do gráfico com os eixos, o comportamento no infinito, o crescimento ou decrescimento, os extremantes, a concavidade, os pontos de inflexão e ográ-

e

x

1

lim x+-oo

f(x) - 2

H.2BB f(x) = 2x 3 - 6x

1 . (2x - 5) - (x - 1) . 2 f) f'(x) =.

fico de f.

(analogamente)

(2x _ 5)2

-3 (2x _ 5)2

< O,

então f é decrescente em todo intervalo que não contenha

V-x

=F

5

H.2B9 f(x) = 4x 3 - x 2 - 24x - 1

2'

%

H.290 f(x) ~ 3x 4

g) f é derivável em seu dom rnia e f' nunca se anula, então f não tem extremantes.

h) f"(x) =

(2Xl~ 5)3

x

< '2- = 5

> 2' =

2x - 5

<O

2x - 5

=

> O =<>

f"(x) f"(x)

< O =>

> O =>

concavidade negativa

=

5

'2

(em que

é definida), concluímos que o gráfico de f não tem ponto de inflexão.

6x 2 - 4

x 1/ 3 + 2 • x 4 / 3

H.295 f(x) = x

concavidade positiva

+

(x - 1)2 (x + 2)3

H.294 f(xl = 1

5

Como o sinal da concavidade muda no ponto de abscissa

190-H

=

4x 3

H.292 f(x) = 3x 2 / 3 - 2x H.293 f(xl

então: x

H.291 f(x)

+

+

(x _ 2)1/3

~

x+1 H.296 f(x) = ~

não

9x H.297 f(x) = x2 + 9'

191-H


y

CAPÍTULO IX

-

NOÇOESDE CÁLCULO INTEGRAL

Em cada sub-intervalo podemos calcular, aproximadamente, a área sob o gráfico, calculando a área do pequeno retângulo que fica determinado quando supomos f(x) constante; a área procurada será, aproximadamente, a soma das áreas destes retângulos.

,I-

Jt!

p"P"

....

....10-'

a

x

b

Vamos descrever mais precisamente o procedimento acima relatado. A divisão de [a, b] em sub-intervalos é feita intercalando-se pontos XI, X2' ... , Xn- l entre a e b como segue:

I.

INTRODUÇÃO - AREA

188. Historicamente, foi da necessidade de calcular áreas de figuras planas cujos contornos não são segmentos de reta que brotou a noção de integral.

a = Xo < Xl < X2 < ... < Xj_l < Xj < ... < Xn _ 1 < Xn = b Os n sub-intervalos em que [a, b) fica dividido têm comprimentos Âjx = Xj - X;_l' i = 1, 2, ... , n. Escolhemos Xj E [Xj_1 , Xj] e supomos f(x) constante e igual a

y

Se f(x) fosse constante e igual a k em [a, bJ. a área procurada seria a área de um retângulo e teríamos: A = k· (b - a)

Não sendo f(x) constante, dividimos o intervalo [a, b) em sub·intervalos suficientemente pequenos para que neles f(x) possa ser considerada constante com uma boa aproximação.

192-H

em

i = 1, 2, ..., n.

[Xi_I, Xj J.

Graficamente, temos:

y

Por exemplo, consideremos o problema de calcular a área A da região sob o gráfico da função f: [a, b) ...... IR, onde f(x);;' O (ver figura). Admitindo conhecida uma noção in· tuitiva de área de uma figura plana, e ainda, que a área de um retângulo de base b e altura h é b· h, vamos descrever um processo para determinar a área A.

f(xj)

a

b

x

xo=a

y

x

A área A é aproximadamente a soma das áreas dos retângulos, e escrevemos: k

A

==

f(Xl)ÂIX + f(x2)Â 2 x + ... + f(xj)ÂjX + ... + f(xn)Ânx

ou seja: n

a

b

x

A

==

L f(Xj)ÂjX i=l

193-H


A soma que aparece no 2? membro das igualdades anteriores se aproxima mais e mais da área procurada à medida em que dividimos mais e mais [a, b), não deixando nenhum sub-intervalo grande demais.

EXERCICIOS H.298 Faça uma estimativa da área A sob o gráfico de dividindo o intervalo

[O, 50]

f(xl

~

250

2

x -10'

em sub-intervalos de comprimento

O';;;; x ,;;;; 50.

10.

Solução

189. De um modo geral, se f é uma função continua definida em [a, bl. o núme-

y

250 r - -__

n

ro do qual as somas

L

f(Xj)D.ix se aproximam arbitrariamente à medida em que

i=l

todos os D.iX se tornam simultaneamente pequenos é chamado integral de f em [a, b)

e é representado por lb f(x)dx.

pequeno,

i = 1, 2, ... n,

Assim, podemos dizer que, sendo D.iX

temos a igualdade aproximada:

x

Façamos xo ~ O, x\ ~ 10, X2 ~ 20, x3 ~ 30. X4 ~ 40, Xs plo, xl ~ 5, x2 ~ 15, x3 ~ 25, x4 ~ 35 e Xs ~ 45.

n

f(x)dx

0=

L f(Xi)~iX i~1

~

50 e escolhamos, por exem-

Graficamente, temos: y

No caso da área A que estávamos calculando, podemos escrever:

A =

i

b

f(x)dx

190. Em muitas outras situações não diretamente ligadas ao cálculo de áreas, somos levados através de um raciocínio semelhante ao exposto acima, a considerar uma função f definida em [a, bl. subdividir [a, bl. formar somas do tipo

A 0=

n

L

A área

f(Xi)D.jX

e determinar o número de que tais somas se aproximam à medida em

j=l

que os D.iX diminuem, ou seja, somos levados a um processo de integração. Estabelecer a noção de integral desta forma geral é o que pretendemos a partir do próximo item.

194-H

A terá o valor aproximado:

f(x\ID.\x + f(x2)D.2x + f(x31l>3'x + f(x4)D.4x + f!xslD.sx

Efetuando os cálculos, resulta:

A 0= 8375

o

valor correto, conforme veremos, é 8333

+,

sendo o erro cometido da ordem de

0,5%, apesar do número de subdivisões ser t:io pequeno.

195-H


H.299 Obtenha uma estimativa da área sob o gráfico da função t(x) =

2~O.

x E [1O.50j divi-

dindo o intervalo em 4 sub-inter valos de comprim ento 10. (O valor correto das áreas

procurada é 321.91.

Ouando isto ocorre, dizemos que a função f é integrável em [a, b j e I é a integral de f em [a, bj. Precisam ente. dizemos que f é integráv el em [a, b) se existe um número real I satisfaz endo à seguinte condiçã o: (0 Dado E > O. existe /j > O tal que toda partição QJ com norma J.l < /j n

temos

I

L

f(Xi)21 i x - II

< E,

qualque r que seja a escolha dos xi

em [Xi_I, x;l-

j""l

11.

A INTEGRAL DEFINIDA

Vamos agora estabele cer de um modo geral a noção de integral de uma função f definida em um interval o [a, bj.

195. Integral Sendo f integráv el em

191. PARTiÇ ÃO

com

=

Xo

<

Xl

o número I é chamad o integral de f em

[a, bj (ou integral definida de f em la. bj) e é represe ntado por

Uma partição de [a. bj é um conjunt o Xi E [a. bl, i = 1. 2 n e a

[a, bJ.

<

X2

<

<

Xi-l

<

Xi

JP = {xo.

< ... <

Xl, x2 • •.. ,

Xi-l, Xi, .... xn}

xn = b

ta que. dado

J.l

<

/j

E

> O,

=

existe

/j

I~ f(xj) 21 iX -

>O

f:

J:

f(x)dx; resul-

tal que

f(x)dxl

<E

Vamos. agora. estabele cer uma condiçã o geral de integrab ilidade.

192. Norma Chamam os norma da partição {21 1 x. ~2X • ...• 21 i x. "', ~nx} onde

/f 21iX

o número

=

iJ. máximo do conjunt o Xi - Xi-l. i = 1. 2.... n

193. Soma de Rieman n Sendo Xi escolhid o arbitrar iamente no interval o [xi_I, x;l. i = 1. 2.... n. a soma f(xI )21, x + f(x2)21 2 x + ... + f(xi)21ix + ... + f(x )21 n nx

L

196. Teorem a 1 Se f é continu a em [a, bJ. então f é integráv el em [a. bj. A demons tração deste teorema está além dos objetivo s destas noções iniciais e deixare mos de apresen tá-Ia. Ela pode ser encontr ada, por exempl o, no livro Cálculo e Álgebra Linear, Kaplan - Lewis - Vol. 1. Capo 4.26. Livros Técnico s e Cient. Edit.

n

ou seja.

f(Xi)21 ix

se chama soma de Riemann de f em la. b) relativa à par-

j=l

tição

{jJ e à escolha

feita dos Xi"

EXERCICIOS H.300 Calcule. pela definição. a integral de

Solução

194. Função Integráv el Devemos calcular

Sob certas condiçõ es bem gerais, que estabele ceremos a seguir, as somas de Rieman n se aproxim am arbitrar iamente de um número fixo I, quando a norma iJ da partição j? se torna cada vez menor, indepen dentem ente das escolha s dos Xi·

196-H

f:

flx) = 5x + 7 em

It.

5).

15x + 7)dx.

Como a função flx) = 5x + 7 é contínua em [1, 5], sabemos pelo Teorema 1 que a integral existe, Dividind o [1,5] em n sub-inter 4 valos iguais de comprim ento ~, temos:

197-H


Xo :::: 1

4

= 1 +~

Xl

f

4

Xi :::: 1 + i

4 n

1 + 2

x2

1 +

li - 11~,

Temos, então:

n

~:

... , x n :::: 5

n

Escolhendo, por exemplo, em cada sub-intervalo, XI como sendo o ponto médio, resulta: Xj_l

Xi

+~li-11+1

+ Xi 2

Segue que f(XjlÁjx

4 +-j n

n

4 n

5xj + 7 = 12

f("jl

112 + 20 i n

!QI~, n

20 n

+-j

n

-

De fato, calculando a área sob o gráfico de f(xl = 5x + 7 entre x = 1 e x = 5,

Temos:

2 n

+-j

2

(5x + 71dx = 88

y

10 n

ou seja,

f(XjlÁix Logo, n

L:

n

f(X)Ái X

L: I n

48 _ 40 + 80 il

=

n2

n2

1=1

i=1

=

48

x

5 n

Como

L:

n • In j

+ 11

2

,

resulta que

i=l n

L:

40 80 n' (n + 11 f(XilÁjX =48-,,+-;:;2' 2

i=l

48 _ 40 + 40 n

I~I n

H.301 Calcule, pela definição, conforme o exercício H.300, Como

Á1X

= Â:! x = ...

=

Áix

a norma

J1

quando J.1 se aproxima de zero, temos: 1) n cresce arbitrariamente 40

4

será igual a n

logo,

cada sub-intervalo

[Xi-I, xil

um ponto

b) Xi

=

41

L:

se aproxima de 1 Dado:

f(xjlÁjx

se aproxima arbjtrariamente do número 48 - O + 40.1

escolhendo em

Xi

H.302 Calcule, pela definição, conforme o exercício H.300, n

+ 7ldx,

Xi tal que

2) - - se aproxima de zero n

n

15x

a) Xi :::: xi-l

.

3) n + 1

!:

i: i==1

i2

~: x dx. 2

n • In + 1) • (2n + 1)

6

i=1

ou seja, n

f.l ê" O =

L:

f(xjIL\;x ê" 88

j,- I

198-H

199-H


111. O CALCULO DA INTEGRAL (Evitamos escrever A(x) = 197. Vamos agora procurar um processo para calcular a integral de f em [a, bl sem termos que recorrer à definição.

C

f(x)dx para poder destacar que a variável x

é um dos extremos do intervalo de integração). Com as hipóteses já admitidas anteriormente, vamos mostrar que a derivada da função A(x) é a função f(x).

v 198. Teorema 2

Consideremos f contínua e não negativa em

[a, b].

O número

f:

f(x)dx Se

representa a área A sob o gráfico de f(x) no intervalo [a, b].

A =

A(x) =

!:

f(t)dt,

então

A'(x) = f(x)

Demonstração Seja

C

x E [a, bl

e h

>O

com

x

+

h E [a, b].

f(x)dx Sendo f contínua em [a, bl, ela

a

b

x

o será em [x, x + hl. e portanto admite um ponto de máximo x e um ponto de M mínimo x em [x, x + h]. m

v

Observação: Naturalmente, a letra que representa a variável independente pode ser escolhida arbitrariamente, e vale que: A =

f:

f(x)dx =

J:

f(t)dt =

J:

Raciocinando geometricamente, em termos de área, na figura ao lado, segue que

f(u)du = '" etc,

f(x m ) • h";;;; A(x

+ h)

- A(x) ..;;;; f(xM) • h

Logo, f(x m ) ..;;;;

v Vamos chamar de A(x) a função que a cada x associa a área sob o gráfico de f no intervalo [a, xl (ver figura), Segue que

A(a) = O,

f(x)dx,

2OO-H

!:

l(

x+ h

b

x

f(x) enquanto que o quociente A(x + h) h - A(x) se aproxl'ma da derivada à direita

A(b)

de

e de um modo geral.

A(x) =

8

Quando h tende a zero, f(x m ) e f(XM) se aproximam simultaneamente de

b

(a

)

A(x + h) - A(x) ~ f(XM) h ~

A(x).

isto é:

f(x) ..;;;; A'(x+) ..;;;; f(x) A(x}

Resulta que A'(x+) = f(x): Analogamente, sendo h O, considerando o intervalo [x + h, xl, temos:

<

f(t)dt

a

x

b

x

f(x m ) • (-h) ..;;;; A(x) - A(x + h) ..;;;; f(xM) • (-h)

201-H


199. Teorema 3

y

Se F(xl é uma função qualquer que satisfaz à condição F'(x)~ f(xl em la, bj, f continua em [a, bJ. então F(x) ~ A(xl + c onde c é uma constante e f(x)

C

A(xl

f(tldt

Demonstração

a

x b

x+ h

De fato, mostramos que A'(x) ~ f(x) e sabemos por hipótese que F'(x) ~ flxl; segue que a derivada de função Flx) - A(xl é nula em [a,b] e então F(x) - Alx) é constante, ou seja, F(xl - Alxl = c. Sendo, então, Flx) tal que F'lx) = f(x), temos:

x

F(bl ~ Albl + c { F(a) ~ Ala) + c

Segue que f( f(

)

xm ~

A(x) - A(x + h)

-h

Logo,

f(x)

O+ c

~

c

e então

A(b)

F(bl - F(a)

)";;:: A(x + h) - A(x) h

~

A'(x-)

~

Isto mostra que

f(x),

isto é,

A'(x) ~ f(x)

A'(x-) em

~ ~:

Utilizando a notação A'(x)

~

d

d; (

f(x)

e lembrando que A(x)

~

f:

la, b] deve ser o seguinte: ai procuramos uma função F(x) tal que F'(x)

f(t)dt, temos

que A(a)

~Oe

~: f(x)dx,

A'(x)

~ f(x).

~ f(x)

podemos procurar uma função como A(x), tal

e teremos: A(b)

~

L

b) vale que

f(x)dx

~

f(x)

F (bl - F (a)

Observação: na justificativa do procedimento acima, utilizamos como hipótese o fato de f ser não negativa em la, b); caso flxl < O uma pequena alteração nos argumentos levaria à mesma conclusão, de modo que a única exigência efetiva para f é a continuidade em la, b].

bastante sugestivo, já que estabelece uma relação essencial entre dois conceitos que nasceram de forma independente: o de derivada e o de integral. Para calcular

onde f é uma

f(x)dx,

função continua em

la, b].

Jafx f(t)dt)

íJ

b

200. Resumindo, o procedimento para determinar

a

o resultado

201. Uma função F satisfazendo a condição F '( x) ~ f(x) é chamada primitiva de f ou ainda, integral indefinida de f. Se F é uma primitiva de f, então F(x) + c, onde c é uma constante, também é. De um modo geral, representamos uma primi-

Jf(x)dx. Assim, por exemplo, se f(x)

tiva genérica de f por }: .f(x)dx.

Este procedimento pode ser simplificado se atentarmos para o seguinte teore-

202-H

~

A(b) + F(a)

x m ""

e como no caso anterior, quando h tende a zero, resulta que a derivada à esquerda de A(x) é igual a f(x):

ma.

F(b)

~

de f as funções

.

x3

-3 ' x3

J x 2 dx =:3 +

c

x3 3 + 5,

=

x 2 , são primitivas

3

x ou, de um modo geral':3 + c,

e escrevemos:


EXERCf clOS

Outros exempl os: 1. Ixsdx 2.

Jxndx

3.

JldX

X6 c 6 x n+ l n +1

ai f(xl =

+ c (n

* - 1)

J~os x dx 5. J~en x dx

.y;:

1

di f(x) = 1 + x2

1

bl f(x)

~ Jdx ~ x + c

4.

6.

H.303 Determine primitivas para as funções:

~-+

=

cl f(xl

x2 - 1

x3 x-2/5

e) f(x) =~

sen x + c Solução

-cos x + c

(eXdx :;;;: eX + c

Lembrando das regras de derivação já estabelecidas, temos: 3 / x /2 2 a) f(x) xl 2. F(xl = - =-x 3/2

etc.

3/2

202. Como conseqü ência de proprie dades conheci das para as derivada s, temos ain"da:

b) f(x)

x- 3 ; F(x) =

c) f(x)

x- 2/ 5 ; F(xl

f(k. f(x))dx

If(x)dx +

If(x)dx

Ig(X)dx k constant e

(k *0)

1 + x2 ;

--=2 = 2,;2 x-2/5+ 1

-~+

F(xl

=

1

el f(x)

3

-1

5

1

d) f(xl

f(f(x) + g(x))dx

x-2

- -;2; F(x)

5

~3x

3/5

1

are t9 x

1

x +x

Em cada caso, F(xl + c onde c é constant e, também é uma primitiva de f(xl. Pode1/2 d 2 riamos escrever, genericamente: x x = 3 x 3/2 + c, e t c.

.

.

S

H.304 Determin e primitivas para as funções indicadas:

Seguem mais alguns exempl os que ilustram a aplicaçã o das proprie dades acima.

1. f(x 3 +cos xldx ~ fx 3 dx + fcos x dx 2. 3.

r5x

3

dx

~

(x 3 dx

5

f(3x + 7)dx

~

3

x2

'2

4. J(3 sen x + 4 cos x)dx

5.

204- H

J(x

2

-

5x + 3)dx

3

x4 5·-+ 4

x4 4

+ sen x + c

C

+ 7x + c -3 cos x + 4 sen x + c

x2 5+ 3x + c 2

a) f(x) = x 3 - 2x + 7 b) f(x) = sen x + 3 cos x cl f(xl = _x 5 + 3 x7

x3

d) f(x)

=3" +7

e) flx)

~

x3

+ 1

--7-

H.305 Determin e as integrais indefinid as indicadas : ai f(x 3 - 4x 2 - 2x + 11dx di b)

fsec 2 xdx

cl

S( :; )dx

S~3

dx

e)

205,...H


H.306 Calcule: ai

f{rx'dX

bl

fWdx

S~dX

H.310 Calcule:

cl di

a)

el

S~dX

f~dX

f~/2 cos x dx. cl

Uma primitiva de f(xl

f:

cos x dx

~

~ cos x

é

Flxl

~

fcos x dx ~ sen x.

f

sen ; - sen O = 1

: cos x dx = sen x

Ix S

-

1)x dx

4

x + cos x)dx

f

el

(1 +

1

IV; + J-x)dX

coS xldx

H.311 Calcule a área sob o gráfico de flxl

11r

: = sen

r

Solução

A área A será igual a } 1 flxldx

x2

-

5x + 9.

1';;;; x .;;;; 4.

Iver figura). Logo.

1r

"2 - sen O =

1

y

H.30S Calcule as integrais definidas: ai

J[01

d)

Segue que

Também costumamos indicar os cálculos como segue: -1r

l~isen f:

3x + 5)dx

-

2

Solução

1r

Ix 2

1r

b) H.307 Calcule

J:

f>

dx

di

el

f:

cos x dx

r

9

e então

t ~ dx

J

1

x

(x 2

2

-

5x + 91dx

~

F(4) - F(1)

~

52 3 A

H.309 Calcule:

a)

bl

cl

4

L

7 dx

(

x2 dx

l-x 2 ldx

di {

el

fi -1

r

x 7 dx

x

2x4 dx

H.312 Calcule a área sob o gráfico de f entre x = a e x a) flx) = 4 - x 2 [a. b] = [-2. 2] e b) flxl ~ x2 + 7 [a. b] = [O. 3) e

cl f(x)

=

3 + sen x

f

[a. b] = [O. ;]

d) f(x) = V; + 1

e

[a. b] = [0,4)

el flx) ~ __ 11 + x2

e

la.

~

b

b] = [O, 1]

20&-H 207-H


~:

H.313 Calcule

2 11

(-x 2

+ 5x - 9)dx

J

e interprete o resultado obtido.

senx dx

= -1 +

Solução Temos:

F(x)

S(-x 2

+ 5x - 9)dx 41

-

(-"'6)

+ 5x - 9)dx

x3

= - --

3

v

(--cos 211) - (--cos O)

63

6"""

=

-

-21/2

= O.

n]

e sen x ~O

[11, 211J.

em

= F (4) - F (1)

= -

1

Como sen x;)::O em [O,

5x 2

+ - - _ 9x 2

[sen x dx

AI

(ver figura)

- A2

(ver figura)

21

2

f11 1T

o número

=

0

sen x dx

é o simétrico da medida da área indicada na figura abaixo:

y Como por simetria, sabemos que

AI = A2,

(Lembramos que a medida de u ma área é um número sempre não negativo). De um modo geral, se f(x) <O em [a, bJ. resulta que

De um modo geral, se [a,

el. e f(x)

J: -f(x)

>O

[a, bl. de

f

J:

em

[a. bl

J

e

(-f(x))dx = -

ji(X)dX.

f(x)

<O

f(x)dx

.;;; O

em

AI - A2

f(x)

lc.

>O bJ.

em

então,

(ver figura)

em H.315 Justifique geometricamente, através de uma figura. as afirmações:

f(x)dx = -A

no intervalo

onde A é a área da região situada entre o eixo x e o gráfico

f

a) se

f

é uma função ímpar,

b) se

f

é uma função par,

[a, b].

211

H.314 Calcule

Logo, se

y

segue que

O sen x dx

fa-a

f:

f(x)dx

=

O

f(x)dx = 2

f:

f(x)dx

e interprete o resultado. H.316 Calcular as áreas da região compreendida entre as curvas

Solução

Temos:

20S-H

y

x2

e

y

_x 2 + 4x.

Solução

Jssn x dx

= -

cos x

Nos pontos de intersecção das curvas temos:

209-'"


x

2

=

=

= -X 2 + 4x x = ou

o

A área

A

2x 2 - 4x = x = 2

o=

IV. ALGUMAS Tt:CNICAS DE INTEGRAÇÃO

pode ser calculada assim

203. Até agora determinamos ff(x)dx utilizando as regras de derivação e algumas propriedades das derivadas. Entretanto, o cálculo de uma primitiva pode não ser uma tarefa simples ou imediata. Vejamos alguns exemplos:

(-x 2 + 4x)dx _ ou, equivalentemente:

[(-X 2 + 4x) - (X 2 )] dx

A = (

Ternos, então:

A =

f:

=_

2x 3

F (x)

=

2

2.

hx"

1..; (x 3 3

2

(-2x 2 + 4x)dx x

~

fx • cos x f x • eX dx

4.

a) V = x b) V = x 2 - 1 c) V = x 2 d) V = x 2

e e e

V = x2 V = 1 _ x2 V = 2x + 8

e

e) V = sen x

e

V = V = x 2 - 7Tx

a)

b)

c)

(i;-

sendo

+ c

x sen x + cos x + c

xe X - eX + c

Nestes casos, algumas técnicas são requeridas, a fim de determinarmos a integrai indefinida. Nestas noções iniciais sobre integral, examinaremos duas: a integração por substituição e a integração por partes.

~

Consideremos o cálculo de uma primitiva de f(x) = 2x • cos x 2 • Fazendo a substituição x 2 = u(x), teremos u'(x) = 2x, e então f(x) = u'(x) • cos u(x). Lembrando da regra da cadeia, do cálculo das derivadas, resulta que uma primitiva de u'(x) • cos u(x) é sen u(x). ou seja, que

H.317 Calcule a área da região limitada pelas curvas:

H.318 Calcule

=

1)3

204. Integração por substituição

e A = F(2) - F(O) = (- 16 + 8) _ O = 3

dF

dx =

dx =

+c

f(-2x 2 + 4x)dx =

+ 4x 2 2

3

f 2x • cos x dx = sen x

3.

O e segue que

1.

y-;

F(x)

fu'(x) cos u(x)dx

=

sen u(x) + c

De um modo geral, se f(x) pode ser escrita na forma g(u). u', onde u = u(x), então uma primitiva de f(x) será obtida tomando-se uma primitiva de g(u) e substituindo u por u(x). ou seja:

ff(x)dx = fg(u). u'(x)dx = G(u(x)) + c onde G(u) é tal que G'(u) = g(u).

igual a

f~ (5t + 2)dt

(V; f~ V;

f3x 2

205. No caso de dt

dt

u(x)

=

f3x

2

x

3

y x3

-

1,

~ dx, temos:

u'(x) = 3x 2

r.----: - 1 dx = + c

=

~

.;

f ~«u' dx = 3i2 + c 3 2 U /

(x 3 - 1) 3

+c

210-H 211-H


EXERCICIOS

206. I ntegração por partes

H.319 Determine as primitivas indicadas:

Sabemos que para a derivada de um produto

ai

f7 • sen 7x dx

d)

~x

b)

fcos 3x dx

e)

fa sen x •

c)

fe

X2

+ 1)17 dx COS

x dx

Solução a) Fazendo b)

ulx)

=

f7 sen 7x dx

=

7x,

u(x) = 3x,

Fazendo

, fcos 3x dx

=

temos

u'(x)

fu'. sen u dx

+f

temos

=

I(u(x) • v(x))'dx

7 e segue que

= -eos u

+c

+c

= --eos 7x

u'lx) = 3 e segue que

3 • cos 3x dx

=

f u' • cos u dx

~

=

+

2

ulxl

u(x)

r (x2)

1

=-

Je

2

= x + 1,

1

=-

f

u'(x)

=1

• 2x dx

temos

2

eu • u'dx

=

u(x)

=

sen x,

I esen x cos x dx

=

.!..- eU 2

e segue que (x

18

1)18

b)

c)

f

+c

eU • u'dx = eU

d)

Je dx 5, cos 5x dx f

e)

• 3

+ c

=..!.2 e (x 2)

b) c)

fe

x3

I u(x)

+

logo:

• v'(x)dx

• x2 dx

fu(x) • v'(xldx

I

e que, uma primitiva de v(x) • u'(x) pode ser obtida através de uma primitiva de u(x) • v'(x), caso isto seja conveniente.

= esan x + c

f3'

f

~ 1

dx

207. Por exemplo, procuremos uma primitiva de u'(x) = eX, temos:

fx • eXdx Como

u'(x)

=

eX

v(x)

=

x

(x + 1)2 dx

d)

J~dx

el

J(3X

fcosl3x + 1 )dx

2

~ 7)2 dx

eX.

Fazendo v(x)

=

x e

Jv(x). u'(x)dx

=

=

u(x)

=

v'(x)

=

eX

=1

Iv(x) • u'(x)dx = u(x) • v(x) -

f x • cos 3x dx fl5x -

Iv(x) • u'(x)dx

fvlx) • u'(x)dx = u(x) • v(x) -

+ c

H.321 Calcule: a)

é u(x) • v(x);

segue que

I(3x + 7)15'3 dx 3X

v(x))'

fu(x). v'(x)dx

+ c

H.320 Calcule as integrais indefinidas indicadas: ai

(u(x)

+

I

= x2 , temos u'(xl = 2x e segue que:

+ I Ix+ 1)l7dx = I u 17 • u' dx =-Ts- + c =-1-8el Fazendo

u'(x)dx

Isto significa que

)x • dx

d) Fazendo

fv(x)

u(x) • v(x)

sen u + c =

sen 3x

f e(x

=

Mas uma primitiva de

=-3-+ c c) Fazendo

vale a igualdade:

Assim, segue que uma primitiva de (u(x) • v(x))' é igual à soma de uma primitiva de u'(x)v(x) com uma primitiva de v'(x) • u(x) (a menos de uma constante), ou seja:

x dx

u(x) • v(x)

(u(x) • v(x))' = u'(x) • v(x) + v'(x) • u(x).

ju(x). v'(x)dx

segue que

Ix . eXdx

eX -

x • eX

Iexdx eX

1 ) 13 dx

+

c

H.322 Calcule: 3X

ai

Je dx

di

b)

f(sen xIs cos x dx

e)

c)

fsen 5x dx

212-H

fi3 - 2xl 4 dx

fx

208. Um outro exemplo: procuremos Fazendo

fx

v(x)

=

x

e

u'(x)

=

• cos x dx.

cos x,

segue que:

• cosx dx = Iv(x) • u'(x)dx.

213-H


Como

u'(x)

= cos x

v(x) = x

==> u(x)

~

Não sendo f constante, vamos dividir [a, b] em pequenos sub-intervalos e em cada um deles, aproximando f(x) por uma função constante, vamos calcular o volume da fatia do sólido gerado como se fosse o de uma fatia cil(ndrica:

= sen x

v'(x) = 1,

jv(x) • u'(x)dx = u(x) • v(x) -

ju(x) • v'(x)dx

segue que: Jx • cos x dx

= x • sen x -

j sen x

dx

= x • sen x + cos x + c

Assim, o volume V será, aproximadamente, a soma dos volumes das fatias cilíndricas consideradas, ou seja:

EXERCICIOS H.323 Calcule:

n

b)

f)(· sen x dx ji.3x + 7) • cos x dx

cl

jl2x - 1) • eXdx

,d

y

d) e)

V

!1-3x + 1) • cos 5x dx

==

L

11 •

[f(Xj)]2 • LljX

i=l

j(2x - 3) • e 1 -3X dx

onde

Xj E [Xj-I, x;] e LljX

Xj - Xj-l

Lembrando da definição de integral, resulta:

V.

UMA APLICAÇÃO GEOMETRICA: CALCULO DE VOLUMES

209. Consideremos o sólido de revolução gerado a partir da rotação do gráfico de f em torno do eixo dos x, sendo f(x) ~ O em [a, b]. (ver figura). Vamos descrever um modo de calcular {) seu volume V.

Se f fosse constante e igual a c em [a, bl. o sólido gerado seria um cilindro e teria volume V igual a 11C 2 • (b-a):

y

f(x)

214-H

V =

11C

2

(b - a)

=-...':.-x

Logo: y

h

V

=

11 •

f: (~

y

x)2dx

=

c

o o

I

210. No caso de um cone circular de raio da base r e altura h, Podemos ter:

o

x

I 215-H


211. No caso de uma esfera de raio r. podemos ter:

RESPOSTAS

y

x

- r

CAPfTULO I

,I H,1

f(x) ;

v' r2

-

fi

x2

bl

LO\!lJ: V;

rr fr (v' r

2 - x 2 )2dx ;

-r

rr

fr

/

(r2 - x 2 )dx

/

;

/ I

1

-r

I

H.2

,I

cl

EXERCICIOS

H.324 Determine o volume do tronco de cone gerado pela rotação do segmento de reta AB. em torno do eixo dos x. sendo A = O. 1) e B = 12. 3). H.325 Calcule o volume do sólido obtido pela rotação do gráfico de f(x)

= x2 , x E

[1.3).

em torno do eixo dos x.

H.326 A curva y

=-2.... x

x E [1.4). ao ser girada em torno do eixo dos x determina um sólido

de volume V. Calcule V.

216-H

\

df

tv

""~)L/ -+ ,

bl

217- H


d) g(x) = x 2 e) g(x) '" 2)(

fi

e e

g(x) '" sen x

H.a

f(x)

H.9

a) f-I

=

f(x} f{x)

x

+

=

{ta', aI, (b', bl,

3, g(x)

=

c) h-l (x)

= ~~ = -Ç./;+2

el j-1(xl

=

(xl

p-l

H.38 a) 2a

tg x cos x

2)(

e

c)}

(c',

-

H.40

a)

~

cl não existe

bl 3

cl não existe

bl O

c) não existe f) não existe il não existe

di O

,I 1

,I 2

hl 1

la

= -

H.67 a =

1

bl

1 4

oi

24

H.58 a

-4

=

di -8

CAPITULO 111

quando x ~ 1 quando 1 x ~ 2

<

,I 3

H.l1

H.'4 ,I

10 2 )(

H.13 f-I(x) = 2'arcsenx

5

011

2

di 2

H.46 a) 3

b)~

1 H.47 a) 3

bl

1 6

2

5 H.48 a) 2

H.17 O <ó <0,01

H.50

a)

+

bl

01

dl +00

,I

fi

H.72 a)

b) +00

c)

di

+00 h) +00

fi

b) +00

c)

,I

f)

jl

e)

cl

~

bl26

c)

- 3

bl~

c)

~

3

-'8

2

di

< Ix

H.3

H.28

- ~ I < O e 0< ó 3

2 bl •

a)

c)

g)

<

QEQ9~ 3

H.51

~ 4

hl

8

-"3

b)

aI 3a

~

c)~

o

bl +00

H.78 aI 2

2

bl.i3

cl +00

di

m

f{;

n,

,I O

fi O

gl -.!..-

hl 8

i)

jl

k)

3

m

dI -12

j)

H.52

-2

fi 1

bl 5

a)

c)

não eXiste

H.53 aI 5

b)

o}

5

H.54

bl -11

c)

não eXIste

H.82 a) 1 di 2 fi O

8

H.' H.5

'Of = {Il.n. 12. 31. 13. 51} (güf){x) '" )(3 (fog) (xl

=

(x

+

,I

72

22

bl 4

f)

2

a)

o} 6

d)

5

H.55 al 1

b) -3

c) não existe

,I

hl 3

H.56 a) 2

bl 2

c) 2

H.57 aI

b)

c)

H.59 a)

b) -1

c) não existe

H.84 a)

t

d) -

1~

1

+ 1)3

b) -1 +QQ

o}

91

hl ; )

bl +00

01 O

e)

1 2'

e) O

g) O

h)+

(fOf) (x) '" )(9

(909) (xl H.6

=

thOgOfl (xl

x

+2

=

2(X+ 2)2

H.32

5

H.33

- J

(fOgOh) (X) '" 2 2X + 2

H.35 aI H.7

bl g/xl '" sen x c) g(x) '" tg x

218-H

4 -"5

e P.

f(x)

=

x2 + 4

f(x),.,)(3

H.37 a)

bl~ 19

1

"2

1

b} -

5"

0}1

o} 8

H.60 a) -1

bl

c) não existe

H.61

b) 3

c) não existe

b) -7

cl não eXIste

b) 1

01 2

blo

c)

H.85 a) H.86 aI

f(x) = x 2 + 1

ai g(x) '" Ix I e

-ª-8

il 2

,I O

H.3D a) 2

5

di

e) _~.: myt ~_::-m

v5 3

+00 +00

dl~ H.80 aI

H.18 O

;}

H.77 a) +00 se n for par e +00 se n for ímpar bl cl +00 sec>Oe sec<O se c>O e +00 se c<O di

3

3

+00

+00

H.76 aI +00

0,01 -3-

H.16 0< ó ~ 0,0005

H.70 a) +00

,I

bl -4

14 1

CAPfTULO 11

1

bl

H,43 a) - 3 -

<Ó <

bl 1

H.64 a) -3

H.56 a

2..;2

H.15 O

dI -1

~ quando)( >2

{

,I

4"

y';

X

=

c)

2

fI.J2 4

,I

~

f-I (xl", 2)( - 1

H.12 f-I (x)

1

J....

b)

di H.10

H.63 a) -1

H.65 a)

h(x) " cos x

5

=

dl~ o

oi o

bl

f(><) '" 3)(

e

dI i -\ (xl

f)

= =

T

1

"2

CAPITULO IV dl

1 8

a)

H.62 a)

-3

H.90 a)

%

b) 2

219-H


d)+

e)

~

H.1l2 aI descontinua b) descont(nua

il 2

91 O

o

H.135f'(x) '" g'(x) "'- 6x 5 h'(x) "'- 15x l4

CAPitULO V

3

H.136 "(x) =- cn x-I g' (xl "'- sec 2 x h'(x) "'- sec x • 19 x

contínua d) descontlnua

c)

H,92

a) - sen a ~

di

V2

-2-

hl cos a

b} sec 2 a

oi O il

ml

-sena

..j2

cl seca ·tga

91-~·

fi

../,-

B n}

kl ~ -3-

-.l.

oI a - b

2

s) ~

2

9

b)

a = -

c)

a

O

H.96 a) 210

b) 3--6

H.97 a) 81 e) e2

bl 4 e- 12

o}

f)

H.98

b}

cl

di 3

cl +00

di

o}

aI log3 2

H,99 a) +00

~2

bl

el a

I "·i

H.100a) 4 ~1

bl O

H.l03 a) e 3

bl o

i

.l-

d) I09T

H.122 Não existe

e6

h)

..!...

bl o

o

k E

z

..

2nx 2n - 1 + 2n x

f'(xl f'(x) f'(x) f'(x) f'(xl f'(x)

H.145 f'(O) = 4

= 4,

H.147v = -a' e~t. (cost + sentI -I a=2a'sent'e

e) e2

H.l09 a)

f)

til

e3 10[J l'

c)

H.149 a) f'(x) bl f'(x)

I

H.125 3if.l

H.12]

g)

cl

dI 2 Qn 3 5 Qn 2

1-

3 23

• Qn

= ab • cos bx dI F'(x) -= -(6x + 1) • sen(3x 2 + x + 5) e) F'(x) = eX • cos eX fi F'(x) = 1 + 12 • sec 24x sen g) F'(x) = a x • cos x • .2n a h) F'(x) '" -3 • cossec 2 (3x - 1) x2 i) F'(x) = (2x + 5) • a + sx+ I • .2n a

j) F'(x) '" -sen x • sec 2 (cos x) k) F'(x) = 6 • tg 2 2x • sec 2 2x sen 2) F'(xl '" 2 • e 2x • cos 2x

H.156f'(~1 = O

e- 2

=

8

=

-14, x-

=

-15 • x--6

=-

zero

_i

--2- • x +

H.159 Sim:

2

220-H

3e2 _e- 2 -~2--

aí '" cos x '" a2 ai '" cos(x +%l

e)

bJ

y

cl

-1

y "'- x d) y "'- -x + 2

fi

y

"'4 x

+ 1

..j2 x

y =-3-

2

f)

f'(x)

+3" g) f'(x) -

3 H,132

= a3

3) '" cos(x + 1T) " 34

2

(x _1)2 5x

1

+

3

x 2 - 4x - 7 (x _ 2)2

a~

'" cos(x

2x • sen X + x 2 • cos X _ x 2 • sen x eX

H.l11 a) f'(x) = _

+~)

= a n +1

~+ 2--.x

x

b) f'(x)

=

n x - 1 (n • Qn x + 1)

c) f'(x)

=

a • .2n x + axx+ b

I

-"9 m/s h) f'(xl "'- -

H.1l0e-

4

2x + 1 cl f'(x) = - (x2 + x + 1)2

H.130 "

2 d) Qn 10

cos 4x

cl F'(x)

e)f'(x)",--~

b) 3 Qn 2

2")

I

dI f'(x)

H.1OS a) 2

1

(-1,

7x • sen x + cos 7x b) F'(x) = x2

H,l5S y

H.124 "2

..!...

c) e2

11, 2")'

H.157f'(-1) '" 3 ' e-

H.1261\'

H.l07a) e

1

H.146 y "'- -3840 • x + (3840tr - 1024)

V2 H.123~ -2-

d) e- 3

c) e- S

=

2) f'(x) " a • cos x - b • sen x

H.1211

d)

f'(x)

+ 1 )2

b) 1

H.155 a) F'(x)

= 15x 4 + 4x 3 _+ 9x 2 + 8x + 1 = 9x8 + 7x 6 + 12x 5 + 4x 3 + 3x 2 '" 5(2x + 3)(x 2 + 3x + 2)4 = 104 • (2x + 3)S-cl "- (x 3 + 3x21 • eX " (1 + xl • eX - sen x gl f'(xl = 2a 2X • x 3 (2 + 2n aI h) f'(x) "- 3 • e3X 2 il f'(xl" (2x + 1) • eX +X+I jl f'(x) = - 5 • cos 4 x • sen x k) f'(x) = sen 6 x • cos 3 x (7· cos x - 3 • sen 2 x)

H.l44 a) b) c) d) e) f)

H.1194

d) 3

d)

o

H.l06al e 7 e) e4

f)

H.l53 a) y' -= (x2 c) iO, O),

x2

~) f(x) ~ 3x~ + 2x + 1 n~1

H,120 3

cl Qn 4

21

-""5

d) f'(x) "'- 4x 3 + 10x

H,1183

bl 1.:(1 31

2x

cl f'(x) '" 6x + 1

'

CAPITULO VI

fi +00

oi

H, 104 a)

di

I

o

4

± ~ + 2 k1T,

H,117 a "'-

O

~ V",-

47

d) +00

bl O fi O

oi

di •

bl f'(x) '"

bl 2

H.95 a) +00 e) +00

H.l0l aI

3

=

H.151y = (..!.-l)(x + 21

H.142a) f'(x) ~ SS x lO

I

(cos x - sen x)

2X h) f'(x) _ 2· e • (tg x _ sec 2 x) ~ t93x

CAP(TULO VII

H.115 a) descont(nua b) descontinua

H.116a)a=-1

bl

sec 2 x • (sen x + cos x) - Ig x g) f'(x) -= ---.(sen x + cos xl2

e2x - e2

b) 108 m/s 2 c) t =- 3s d) 1 = 2s

cl descontinua

H.93 aI O

=

fi

H.140 a) 32 m/s

H.114 a) contInua bl contínua cl descontínua d) nescontínua

I

a)

H,138Y

dI descontinua

2

pl O

ti

H.94

H.1 13 a) descontlnua b) continua c) contínua

f'(x) = - cossec 2 x f'(x) = sec x • tg x f'(x) = --eossec x • cotg x f(x) "'- 2 • tgx • sec 2 x f(x) " sec x • (t9 x ~ sec x) f'(x) = 2x· tgx + (x 2 + 1) • sec 2 x

H.150a) b) c) d) e)

x(senx+cosx) + cosx x2 • eX

d) f'(xl "" cos x • 2n x

+

se:

x

H,134 53 m/s2

221-H


cosx + x.Qnx 'senx x'cos 2 x

el f'(x)

h)

1

c) f(x) "'-~

,

cotg x

=

H,199 crescente para

d) f'(xl .. 2x +

f'(xl '" x • Qn x • Qn a

.1 f'(xl = -

V

,

H,176 Não, não existe 1 1 _ x2

V' _x'

- 2

H,162 al f'(x) .. 7 .4x317 fl f'(x) '" are tg x + b) f'(x) ..

c)

f'(x) '"

x

,+"';2

ti

---

g(xl

f'(x)

j)

=

2x 2 arcsenx 2 + Yl _ )(4

3x 2 e

x3

H.183c

1 - 2x

arc sen x + arc cos x

RI f'lxl

4ab

~ (ax + bl(ax

8x + 4 3~1 + x + x2

5x 2 + 3 m) f(x) .. 3 -Ç/ (xl + 1 )2

bIs

H.164

v

=

" 4""

H.184 c

~ 'arcsenx' arccosx 9 -4"

x

,

=

ri f(xl '" - /Vc:s x sl f'(x) '" 1 _1 x 2

,

t)

f'(x) ~ 2(1 + x) • Qn a

3 H.163a) f'(x) ." ~9;2

222-H

~

~

, -"2

1 ou

x

~

1

x E IR ..

decrescente para

V5

51T 12 +

k1l" ~ x ~

1111" 12 + k1T,

k

E

..z

<

H.207 crescente para 11" + 2k1T ~ x 21T + 2kll" decrescente para 2k1l" ~ x ~ '1T + 2kll" H.210

H.170a) f'(xl = (sen xlIx:!) • [2x. Qn sen x + x 2 J b) f'(xl .. x(x ) • 'x 2 [3 • ~ x + 1] X c) f'(x) '" x(e ) • eX • [Qn x + ~

cotg xl

l

H.190 f não é derivável em H.192 f:

l(

f(x) = 4x 3 + 10x f"(x) .. 12x"2 + 10 f"'(><I '" 24x f,v(xl '" 24 f(n)(x) .. O, V fi ~ 5 (_1)n • n! x- n - l ,'ti n EIN'

bJ t1n)(x)

=

c) f(n)(xl

= eX,Vn

:s;;;;; -

2

ou

g : 71f + 2k1l"

6

f'(xl = (e X )t9 3x • l3 • sec 2 3x • x + tg3x]

d)

pl f (x) '" 2(x _ 1 )3/2

2Y;

v2

H,189 f não é continua em I

a)

x

x

H.206 crescente para

H.188 f não é contl'nua em I

x-3

senV;

27

H.186 c '" 1 +

,

H.171

~

decrescente em IR ~

8

h: 5

2Y;. (2 _FI'

,

2"

H.205 crescente em IR+

H.167 g

2~

-

H.204 crescente para todo

H,165 13, 2..[21

15x 2 +8x+3

ql f'(xl = -

H,202 crescente para x ~ O decrescente para x ~ O

="2

H.185c = ±

(a - c) • [bx 2 + 2(a + dx + b] 2 y(ax2 + bx + c)(cx2 + bx + a)3

,

e, no entanto, 9 não é con·

decrescente para

-

x E IR

<>

.

4v3 H.181 c = - 3 -

klf'(xl"2~

b

01 f'(xl =

f(2)

=

~

d I H 201 crescente para x 2 tem deriva a nu a · decrescente para x 2

H.203 crescente par(l

f(x) .. 4 yax + bX3/2

n) f'(xl ..

f(-2)

I~2, 2,I

1

2ax + b h) f'(x) '" 3yax2 +bx+c

Q) f'(x)

, "'"3

2±V7 H.180c =-3--

il f'(xl = 2(1 +x 2 ) ~

3

= O,

.

no Intervalo

H.179c=3

f'(x) .. - ~. arccosx

h)

1 1 2 2.y; + 3~ +;3

kl f'(x) ""

, , ;r-:-;

tínua no intervalo.

-

,

-7;S

gl f'(x) "'2yax+b

j) f'(x)

=

3V x4 {1-x 2 )3(arcsenx)2

1

il

*-

para x gl f'(x) -

f'(x) '" 2 Y;

f) f'(x) '"

Y;

~

ou

2

Y1 _x 2 'arcsenx-3x

~...[;.7

m

3.,[2

O ~ x :;;;;;-2-

"'x "'0

-3.,[2 2

para

H.178 f(x) '" x , no intervalo 1-1, 4]. tem derivada nula H.200 decrescente para todo para x '" O e, no entanto, f(-1) '" 1 1(4) = 16;

3

dl f'(x) '" - -5-' x e)

decrescente

f'(2)

H,177 Não, não existe f'(O)

,

!!. • if";i 2

3.,[2

-3:;;;;; x:;;;;; - --2- ou

H.175 Sim.

2ax + b fl f'(x) - ax2 + bx + c gl f'(x)

CAPITULO VIII

3x'

b) f'(xl = - Y 1 _ x6

E. + 4

í:-1:S;;;;;x

2kll"

x

~

x

=1

E I

7

< x <.!..!..!!. + 2k1f, 6

<x <

~O

H.194f:x:S;;;;;-2 ou g:x ~ O h : não existe x I : não existe x

ou

k E,z

É.E: + 2k1f, k E..z 4

x

~1

5 2" e ponto

H.214 x .", -

O~x~l

H.215 x '"' 2

é ponto de mínimo

H.216 { x

=

3

=

-3

EIN-

d) f(n)(x) '" (~1)n, e-x, Vn EINe) f(n)(x)~" coslx + ~), V n E IW

H.195 crescente para x ~ 1 ou x decrescente para 1:S;;;;; x ~ 5

~

é ponto de mínimo e

5

l(

H.196 crescente para x;;:'-l decrescente para x ~ -1

H.217 x

=

2

é ponto de máximo

é ponto de inflexão

H.172 a) v(t) '" -aW sen(wt +..p) b) v(O) = -a<.o sen..p cl a(tl .", -aw 2cos(wt + ..pl dl a(l) = -aw2cos (w+..pl

x", 6 H.197 crescente para x ~ -2 ou -1 ~ x ~ 1 ou x;;:' 2 H.218 { x = 50 decrescente para -2"';;: x ~ -1 ou 1:S;;;;; x :s;;;;; 2 7

H.173 A

H.198 crescente para x ~ - 1 ou x ~ 1 decrescente para -1 ~ x :;;;;; 1 e x

'0

6

e

k

=

2

, .

de maxlmo

'*

O

H.219 x = -

5 2"

é ponto de máximo e é ponto de minimo

é ponto de mâximo

223-M


X

H.220

x

{

2k1r (k E Z)

-

-

é ponto de máximo e

3

(2k + 1 )1T

=

é ponto de máximo

711 + k1T (k E Z) 8

é ponto de mínuno

=

= e -I

H.222 x

H.247 r

' • 3 n + kn Ik E 21 8 { x

e

2 A v'2 ,.--3

=

R

y'2

e

h

=

{ cone. po,;t. p." x> O cone. negat, para x <O ponto de inflexão 10.01

H.274

{ cone. po,". pa" x > 1 ou -1 < x < O cone, negat, para x -1 ou O<x<1 ponto de inflexão: lO, O)

H.275

x<2 { cone. po,;1 p"" cone. negat. para x>2 ponto de Inflexão 12. 01

4 R

H.248 8 em H.249 a 4 km de B

<

H.223 x = O é ponto de mínimo

~)

'" 2..ç,t2

+8

v-::i

é valor mt"nimo

= ~ é valor máximo e

f(1)

H.228 { ft-1l '" -

~

v3 I

H.232 {

(V3, - 6 V3 I

1 H.233 ( 2"

' '3 I

3~

H.236

X = -1 é·ponto de míni.mo e H.2SS { x = 1 é ponto de máXImo

é ponto má~mo e é ponto mínimo

H.259 não tem extremante

~I

H.261 x = 1 x = -2

é ponto de máximo absoluto e é ponto de mínimo absoluto

H.262 x'" -1 x -= 2

é ponto de máximo absoluto e é ponto de mínimo absoluto

e ponto

(V;, ~e

) é ponto máximo

3

H.236a

=-"2

H.263 x '" -6

H.237 t

'"

3s

H.265 11. 21

x =1

H.23S x = 2 é ponto de mínimo absoluto x = 5 é ponto de máximo absoluto H.239 x = 6 é ponto de mínimo absoluto nSo existe máximo absoluto 31r

()

H.241 ai t

=k

(2'" + 2k1r

- .'lI

b) t

=~

(2k11 - R.I

e s =a

H.243 x = V

=1-

s =O

H.267h

4A

=3

H.288 h = 4R

r;:...;6õ

x

<,

{

cone negat para

1

< x <1

_ pontos de Inflexao. (1, 1)

H.278~ +

2k1r

< x < 941r + 2k1T,

H.279 x

3

<5

e

x

'*

ou

e

x

>~

x

H.285 4

70

("3'2'7)

k E,z

O

é ponto máximo

H.281

H.28? ao

f(x)

al

H.288

de mínimo absoluto e é ponto de máximo absoluto

e

4.,fi A ' " - 3- -

e

r '" R

.iff'

.'1f,V r OV.

H.270 18 em

e 24 em

H.269

>~

x

3

(e 2 ,

x

< x < Y6

(O, Dl, (V 6, ~),

cone pOSlt para

é ponto de mínimo

H.260

ponto mínimo

H.277

X = O é ponto de min~mo e H.256 { x' -2 é ponto de máxomo

=O

ou O

ou

1-Y6. - ~)

. . é ponto de mlnlmo

é ponto máximo

~lé

,

pontos de inflexão:

v'3i H.253 8 = are cos - - 8 -

H.258 x

< - V6

H.276

H.257 x = O é ponto de mt'nimo

H.231 f(1 I = O é valor m{nirno

H.234 (V-2,

x

1 -

H.254 x = 2

H.284

-VS < x < O

é valor mínimo

H.230 não tem extremos

5

cone. POSIt. para cone. negat. para

v3A

1

fiO) = O é valor mínimo e H.229 { f(-21 -= 4e-2 é valor mínimo

1-.../3. 6

, 1 H.251h·~

H.2524

= -5 é valor mínimo

H.226 f(21

H.227 f(

e

é ponto de máximo é ponto de mínimo

f(x)

6 km de C

é ponto de mínimo

H.224 x = -1 x -= 1

H.283

H.273

é ponto de ml'nimo

(k E Z)

3 H.221

4A

H.246h = 3

x

..J2

ifEiV 2

H.289

H.282

H.271 7 dias

c - b

H.244 5 R

H.272~

x

224-H

225....,H


o

TH.4 (PUC-71)

2 lim x - 4x + 4 X+2 x - 2

limite:

aI não existe b) não é nenhum número real c) vale 2

d) vale O e) vale 4

3

lim x - 8 X+2 x - 2

TH.5 (CESCEA-74) O valor do limite

TESTES

O

a)

bl 12

O

b) 1

a)

~

c) 2

~

cl

3

(I TA-69 I Sejam IR o conjunto dos números reais e C um subconjunto de IR. Definimos supremo de C como sendo o número L satisfazendo as seguintes condições: 1~)

2~)

L ~ x, L' L

<

x E C

=3 x

E C

I x

> L'

Seja C o conjunto dos números naturais menores do que 11. Assinale a afirmação verdadeira, relativa ao conjunto C:

b) L = 10 c) L = 11

naturais menores do que 10 naturais menores ou iguais a 10 racionais maiores do que 1 e menores ou iguais a 9 racionais maiores do que 1 e menores do que 10 nenhuma das afirmações anteriores é válida

aI não existe bl é 4 c) é zero

228-H

a)

5W

a)

..!... 3

2

c) 3a 3

_ 84

el 5a 3

x - 7

cl 7 s

cl~ <!2x + 6

lim x+1

V7

d)

el 1

é igual a:

d)2.. 3

5

vale:

S

<i7

~-1

bl_1_ 5

vale:

dI 4a 3

lim $rX+7 VX -

blO

bl2.. 5

el.23

3

lim~ x+o 1 + x - 1

TH.11 (PUC-74) a) _1_ 4

d)~

3

lim - - x+a x - a

TH.9 IPUC-70) O limite

TH.10 IPUC-71)

TH.2 (ITA-701 Seja B um subconjunto do conjunto dos números reais FI. Oizemos que um número b é um ponto de acumulação do conjunto B, se para qualquer número real positivo k, arbitrariamente dado, existir um elemento c de B tal que O < Ib - c 1< k. Nestas condições b = 10 é ponto de acumulação do conjunto dos

TH.3 IGV-71) O limite,

b) 2a 3

dI L = 12 e) não existe L.

a) L = 9

a) b) c) dI e)

a) a3

e) 6

dI 4

x4

TH.8 IPUC-71) O limite: TH.1

vale:

fim

b)

3

e) não sei.

2x 2 - 5x + 3 3 2 X+I x - 3x + 2

TH.7 IPUC-73)

LIMITES

d) 8

3 2 11m x - x - 2x x+2 x 2 - 3x + 2

TH.6 (PUC-701 O a)

c) 16

é:

e)~ 2

- 2

é igual a:

x - 1 dl_1_ 7

c) _1_

6

e)_l_

8

TH.12 (CESCEA-72) Assinale a afirmação falsa:

x2 - 4

Iim - - X+2 x - 2

a) lim 3x d) é 2 +00

e) é

x+o dllim x+-oo

2

+ x

x = 1

x 2 - 3x

b) lim

x+o

+5

2x 2 + 1

1

2'

~= x

x2 - 1

c) lim - - - = 2 x+l X - 1

el não sei.

229-H


ai -2

j

I"

TH •.13 (PUC-78)

~:(X) b) -1

4x 2 + ex + 3

x2 c) O

TH.22 (FEI-67)

é igual a:

5

-

d) 1

a) a ~

el 2

Se

_.!-) • tg ax ~

lim 11 x+O

3

bl a = -1

dla=-oo

3,

então:

x

c) a

= -3

e) nenhuma das respostas anteriores

TH.14 (CESCEA-73) Assinalar, dentre as afirmações seguintes, a correta: a) Iim

(1

+ .!...)n = 1

I" n 2 + 4n + 5 3 2 2 Im n++oo" + n +5

cl

TH.15 (PUC-72)

ai

.,f;;).

c) 2

b) 1

TH.16 (MACK-751 O

lim x+oo

c) 2

b) 1

ai O

e) 4

d) 3

(..J x 2

b)

+00

a)

..J x 2

+x +1 d) 3

e)

- x + 1)

é:

2

b) 1

- cos 2 x

a) O

00

x2

x+O

Conclui-se que

ai

é~ 2

sen2x

"

TH.19 (PUC-72)

TH.20 IPUC-70) Sobre o

sen TfX

x

c) TI

e) não existe

quando x tende a zero, é igual a:

ai Nenhuma das anteriores.

di 00

a) não existe

lim

= logaritmo neperiano, é igual a:

+00

d) 1

x a - 1 --x x+o

com

b) loge"

cl 1

a>O

e a ;6 1 d) e

e)

eX _ 1 lim - - - vale:

x+o

x

1

2

é igual a: e) a

c) 2

1. Iim 2 x++oo 2. lim a x++oo

x

X

= +00 = +00

se

3. lim (x2 - xl = O x++oo

então

x e)

d) 3

1 lim x ·sen-,

x+o

TH.21 (PUC-74)

d) é indeterminado

lim tg x b) 4

Qn

TH.27 (CESCEA-73) Dadas as afirmações

c) é infinito

b) O

x+O a) O

a) logae x2

TH.18 (FEI-66) O limite de a) 1

e)

c) -1

00

rIm

1.

x

d) e

e) TI

- cos x

lim x+O

b) é O

Qn ~ logaritmo neperiano, é igual

di O

eX + sen x - 1 Qn (1 + x) ,onde

cl O

bl

TH.26 (PUC-731 O = IIm - 2 -

onde

TH.25 (PUC-71 I Sendo e a base dos logaritmos neperianos, o limite

;rH.17 (FUVEST-77) Sabe-se que

lim X+O

c) 1

-00

TH.24 (CICE-681 O limite lim x+o

~oo

a) O

(x' Qn sen xl.

a:

~+oo

-

lim x+O+

1'+1

H

di lim

= O

(~

lim

TH.23 (CICE-68) O limite

r»'+ 00 n

n

1)++00

n2 -- = 1 +1

b) lim

ai b) c) di

1

pode-se afirmar:

x

b) é 1

c) é zero

tg x - x

é igual a:

todas as somente somente somente

afirmações são falsas as afirmações 1 e 2 são falsas as afirmações 1 e 3 são falsas as afirmações 2 e 3 são falsas

e) todas as afirmaçães são verdadeiras. dI é

00

e) n.r.a. TH.28 (CESCEM-741 Iim

[5

+ (1 +.!- )n]

0+00

vale

n

x+Q x - sen x a) O

230-H

b) 2

c) 3

d) 1

e)

4

a) 5e

b)

e5

cl 5 - e

d) 5

+e

231-H


Uma função

TH;29 (PUC-71) Se

lim

x+oo

(1 +-.!...)x = a, então, para k

x

f(x)

é derivável num intervalo

(a, b).

Então:

real e não nulo, o limite

P: f é contínua em cada ponto de (a, b) O: Se num ponto x se tem f'(x) > O a função é crescente nesse ponto R: Para dois pontos quaisquer xI e x2 de (a, b), tem-se

Iim x+oo

f'(xI + x2) = f'(xd + f'(x21

vale:

d) e + k

ai ke

TH.30 (CESCEM-73) O valor de a) O

b) 0,5

c) 2

4n

a) é crescente

(OI);

----xz:::4 ;

n = O, 1, 2, .... c) tende para um

das funções f, 9 e h, dadas por

_ sao

f

9

h

a)

2

2k7T

x>O

b)

2, -2

k2!.. 2

c)

O, -2, 2

d)

-2

é

k7T

não existem 7T

± 2"

não existem infinito

k7T

Ew e)

+ 11!

(a

() () -x 9 x ~ cot9 2x; h x = 4

e) não tem limite

TH.32 (CESCEM-74) A seqüência (an)n

(n

f ( x) =

xIx - 2)

e) inexistente

b) é decrescente

d) tende para zero

TH.35 (MACK-741 Os pontos de descontinuidade em R

e

k

1+2+ ... +n n2

lim n+oo

d) +00

TH.31 (CESCEM·71 A seqüência

e)

nenhum dos anteriores

+ l)n+1 n!

-;;n tende para a) O

b)

..!...

cll

e

d) e

DERIVADAS

e) +00

TH.33 (PUC-70) Sobre a função

y = f(x)

~

{1,.

+V

r--::

x - 3,

TH.36 (FEI-68) Indicando por Df a derivada de uma função f, tem-se: se x O;;;; 3 se

a)

x>3

pode-se afirmar: a) b) c) d) e)

>

TH.34 (FEI-68) Assinale a) b) c) d) e)

232-H

se se se se se

todas as proposições P, O, forem verdadeiras somente forem verdadeiras somente forem verdadeiras somente todas forem falsas

1

OU

b) D(uv)

d) D(uv) = v • Ou - u • Dv

definida e contínua V x E IR definida e contínua somente para x 3 definida V x E IR e descontínua somente para definida e contínua somente para x O;;;; 3 nenhuma das respostas anteriores

é é é é

D(.!..) = u

R forem verdadeiras P e O P e R R e O

Du' Dv

TH.37 (PUC-711 A derivada primeira da função

a) y'

1 - kx 2

1 b) y' = - k 2 x 2

d) y'

--;2

k

' =-1el y k2x

a) y' = x

Se

D(-) u

-;j2

e) nenhuma das respostas anteriores

x = 3

TH.38 (PUC-721

Du

1

c)

y --k'X 1 é:

cl

y' =

k x

x2 -1

- Ix ..,... -L -1) Y = x+1 • então:

b) y' = 1

c) y' = 2x

d) y'

2

e) nenhuma das anteriores.

233-H


TH,39IFEI-67) Sendo a) 2

b)

g(x) =

.!..

c)

3

~ ~

1-

: ' então a derivada d)

t

TH.40 (PUC-78) A derivada da função

2

a)

c) 4

b) 3

g'(~)

TH.48 (MACK-73) Sejam

é igual a:

e) nenhuma das respostas anteriores

x2 V = ~ no ponto

d) O

e)

x = 2,

1) g'(:!!..)

= - •

2) f'(:!!..) 4

=

4

é:

Y3x -

g(x) = sen x; flxl = cos 2x; h(x) =

~

2x 2

(g' é a função derivada da função g)

v2

-1

3) h'11) = -2

1

então TH.41IPUC-74) A derivada da função a) -2

b) -3

c) -1

1 + 2x V - - - - no ponto - 2x - 1

b) 1

c) 8

TH.43 (EPUSP:67) Sendo a) O

b) 4

c) 15

x = a)

b) 2

1

TH.45 (CICE-68) Seja ponto

x

a) bl c) d) el

é é é é

d) se todas as afirmativas são verdadeiras e) se nenhuma afirmativa é verdadeira

é igual a:

d) 25

a derivada

f'(4)

TH.49 (CICE-68) Sendo e a base dos logaritmos neperianos, a derivada da função Ixl V = eno ponto x = O:

vale:

a) é igual a 1

e) nenhuma das respostas anteriores

flx) = tg x,

d) é igual a

calculada no ponto

=

Y2 2

c)

d)

V(x) a função v(xl

V3 2 =

e) O

~. x =F O, V(O) .x

±e

a. Pode-se afirmar que no

O:

bl eX

cl 1

descontínua qualquer que seja a contínua qualquer que seja a contínua se for a = O derivável se for a = O é contínua se for a = 1

3

-4

b)

3211

x

1

f(xl

=

O~x

d) O

2X e • Qn x,

então:

=

< 11,

2 • e2

e)

e) nenhuma das respostas anteriores

+ x2

b)

1

(1

+ x) y'X

c) f'(1)

V = arc

O

tg...r;

é:

c) 2x

e) nenhuma das respostas anteriores

nenhuma das respostas anteriores

TH.54 (PUC-71) A derivada primeira da função V = arc t9

234-H

=

311 4

dI f'(1) = e 2

d) 2 y'X (1 + x)

11 TH.47 (PUC-70) Sendo f(xl = sen 2 2x ,então sua derivada primeira calculada para x = 8 a)

a derivada primeira de y em re·

b) f'(1)

a) 1

é derivável em qualquer ponto é derivável mas não é contínua

l.

a) f'(1) = 3'e 2

1

é descontínua nos pontos da forma k11 (k inteiro) não é derivável nos pontos da forma k11

e) 2

11 -2

c)

é:

é igual a:

TH.53 (E.E.L1N5-67) A derivada da função

V = Isen x I

elogex

d) O

-4

TH.52 (E.E.L1NS-68) Se

V

3x - 11 V = Qn cos ( -4--

lação a x no ponto

a)

±1

e) não existe

TH.51 (CICE-68) Sendo

=

c) é igual a

b) é igual a -1

TH.50 (PUC-71) A derivada da função a) x

TH.46 (EPUSP-661 A função a) b) c) d) e)

f'(3)

vale:

4'

V(x) V(x) V(x) v(x) V(x)

a) se apenas as afirmativas 1) e 2) são verdadeiras bl se apenas as afirmativas 2) e 3) são verdadeiras c) se apenas as afirmativas 11 e 3) são verdadeiras

el nenhuma das respostas anteriores

~,

TH.44 (PUC-70) A derivada da função 11

a derivada

d) 16

flxl = 5 •

é:

e) -4

dI -5

TH.42IFEI-66) Sendo flx) = (5 - 2x)8, a) -8

x = 1

O

b) 1

c) 2

d) 3

e) 4

vale:

a) cos x

+ sen x

b)

..!.. 3

e)

2"

1 - cos x

sen x

é:

c) sen x - cos x

1

dI cos 2x

235-H


TH.55 (PUC-71) Se y a) x + 1

=

TH.56 (EPUSP-68) Seja y A derivada de y(x)

2

c)

b) x - 1 =

e) 3

d) -2

y(x) uma função derivável tal que no ponto x = 1:

b) é igual a 1

d) é igual a ;

e) nenhuma das re,;postas anteriores

TH.57 (CESCEA-73) Seja x = 1, é:

f(x)

=

b) 10

3x 2 + 4x - 1, x E IR.

9

c)

y(1)

>O

e x 2 + y2

=

2.

Xo = 2 Xo = 2 é descontínua no ponto Xo = 2 não é definida no ponto Xo = 2 assume o valor -2 no ponto Xo = 2 tem o limite

-2

no ponto

é derivável no ponto

TH.64 (EPUSP-68) Considere-se a função f (x) = x 2 - 1, para x 1.

c) é igual a -1

a) não existe

a) O

a) b) c) d) e)

sen(arc sen x) + cos(arc cos x), a derivada primeira y' será igual a:

definida por

>

a) b) c) d) e)

Então, a derivada de f, no ponto

f f f a

=

1,

e

não é definida no ponto

x

=

1.

é derivável em qualquer ponto. derivada de

f

tem limite quando

x

tende a

1.

nenhuma das anteriores.

x3

y =3

-

1

no ponto

ai f'lx) = 11

x = O vale:

I. "Ix

y

f(x)

=

Ix - 71.

pode-se afirmar:

E IR

b) f'(x) = O, "Ix E IR

.!.3

d) 2

1

c)

e) O

c) f'(x) = 1,

di f'lx) = -1,

TH.59 (CESCEM-72) O valor numérico do polinômio derivado de P(x) = 3x 4 + 12x - 7 para x = - 1 vale: b) -7

a) -16

+ 1, para x

d) 6

TH.58 (GV-71) A derivada da função dada por b)

2x

é contínua em qualquer ponto.

TH.65 (PUC-70) Sobre a derivada da função

a) -1

f(x)

e) 24

d) 3

O

c)

TH.60 (MACK-74) A derivada da função

se x se x

>O

<O

e) nenhuma das respostas anteriores TH.66 (MACK-73) Seja f a função definida por f(x) = 1 x I. O gráfico da função f' If' é a função derivada da função f) restrita ao conjunto

------11_

dada por

- 00

1

f-I- - - - - - - 1

00

é

x f(x) =

3x 2 6x

12;

y

é:

y

a)

a) 3x 2 ;

dI 15x4 ;

c) -4x 3 - 4;

b) não existe;

c)

bl

e) 6x 4 .

_ 1

TH,61 (CESCEA-721 Assinale a afirmação falsa: a) a derivada da função flxl = x é f'(x) b) f(x) = k, k constante ~ f'(x) = O d) f(x)

x

~ f'I1 I

TH.62 (CESCEM-74) Se a) 6x 2 + 3x + k

I

-1 c)

f(xl=..!...~f'(1)=lim x

~

- 1 = lim _x_ _

x-H

(.!.-11

I I

x

y

= f(x)

X

= { -x

x

v c) 6x 1 2 + k

flx)

d)

vale;

dI 3x

+k

el x 6

y e)

+ k -1 x

se se

1';::

x

x .;::

>- 2

Pode-se afirmar que a função

236-H

-1

x

-1

-1

TH.63 (PUC-71l Seja:

-1

I

e) não sei.

podemos afirmar que

b) 3x 2 + k

1 -1

-/

x

x-*l x - 1

f'(x) = 6x,

~'

-

2

f(x):

237-H


DERIVADAS - APLlCAÇOES

o valor

TH.67 (CICE-68)

1

a)

de a para o qual a função y = a • x • e

1

(.,[2' ..;2)

ponto

..r;

TH,72 (CICE-70) Seja f(x) uma qualquer função estritamente crescente no intervalo (a, b) e possuindo .derivada segunda f"(xl contínua em (a, bl. Pode·se afirmar que:

_x 2

é igual a:

a derivada f'(x) de f(x) é positiva em (a. bl f"(x) é positiva em (a, bl Se f'(x) se anula em algum ponto Xo de (a, bl, f"(x) é negativa em (a, b)

então

f"(xo)

O

e) todas as afirmações são falsas

..;2

b)

tem um máximo no

a) b) c) d)

e) indeterminado

TH.73 (GV-731 Dentre todos os números x e y tais que 2x + Y = 60 existe um par a e b para o qual o produto xy é O maior possl'vel. Então b - a vale: TH.68 (FUVEST-78) Sendo b

+

f(x) = x 4

bx 2

a)

e c reais, a função f, definida por

+ c,

tem um ou dois pontos de mínimo (ver figura). Terá dois pontos de mínimo se e so-

O

bl 10

c)

d) 15

50

e)

5

TH.74 (COMBITEC-COMBIMED-75) Cada extremidade de uma haste PQ de comprimento 8 é forçada a mover-se em uma guia, como indicado na figura.

mente se: a)

b2 - 4c ;;;. O

bl b 2 - 4c

>O

c) b <O

d) c

<O

e)

bc

<O

T

.,

TH.69 (PUC-71I Uma função real de variável real y, cuja derivada primeira e y = todo

x

'*

O,

I_J~~~-

para

possui a propriedade:

a) y tem valor máximo para b) y tem valor mínimo para

x = 1 x = 1

c) é sempre crescente d) é sempre decrescente e) é crescente se x O e decrescente se

>

TH.70 (PUC-71l Na função y a) (1, -10)

1 X2

=

x 3 - 3x 2

bl (2, 10)

x

I

I

<O

f-X(t)

+ 4x -12 as coordenadas do ponto de inflexão são:

c) H, 20)

d) (-1, -10)

4

e) (2, -10) Se ao ponto Q se imprime um movimento definido por x(t) = 4 sen 3t, a velocidade de P em qualquer instante t é:

TH.71 (EPUSP-68) No intervalo

-1';;; x ,;;; 1

a função

2x/(x 2 + 2) a)

a) b) c) d)

tem tem tem tem

ponto ponto ponto ponto

de de de de

mínimo mínimo máximo inflexão

e ponto de máximo. mas não de máximo. mas não de mínimo. horizontal.

e) nenhuma das anteriores.

238-H

-2 sen 6t

V4 - sen 2 3t

c) 12 cos3t e) 4

V9 -

b) 4 cos 3t

d)

-6 sen 6t

V4

- sen 2 3t

4 cos2 3t

239-H


TH.75 (PUC-72) A altura do cilindro circular reto de volume V máximo. que pode ser inscrito em uma esfera de raio R é:

R

R

a)

V2

.,f3

b)

e)

2R

..j3

TH.85IPUC-78) A equação da reta tangente à elipse da equação

x2

y2

25

9

-+-=

no ponto

12

9 = 20 Ix - 3)

a) y -

TH.76IMACK-74 ) Sendo f' IXo)

f(x) - f(xo) I ·Im x+xQ x - Xo

x

= 2x -

d) 2;

e)

b) y = -x + 2

I

a) 2x + y = O

bl x

d)

e)

O

d) Y

-2.

12

+ ""5

=

1

-3 (x

- 3)

e) y -

c) y

+

y =

no ponto d) y

= 3x - 2

(1, 1)

=

é:

x

x=I

é:

c) x - y = O

y = O

2x -

= 6x - 4

cl y

bl y = 3x

y

=

x 3 • no ponto

x

= 3x - 3

b) (2.4);

(-2,4);

c)

d) (-4,2);

(4,2);

2x

b) y = -2x - I

c) y

= -2x -3

=

e) nenhum dos

TH.81 (FUVEST) A equação da reta que é tangente à curva de equação y (-1,-1), é: a) y =

-

3

10 (x

- 3)

9

""5 =

- 20 (x - 3) 2

=

b +4 x 2 + bx +-4-' o seu polinômio de-

a) é sempre igual a 2, qualquer que seja o valor de b. b) é igual a 4 para b = O. cl é igual a b para b = 3. d) não pode ser determinada, pois a condição de tangência é impossível, na hipôtese. formulada. e) é igual a 5 para algum valor de b.

=

O. EntJo o

f(x)

é

x +I x - I

,

então f é crescente nos inter-

valos:

e (-1, 1)

(-1. O)

e

(O, 1)

bl (- 00, -1)

d) (-1,1)

e

(1, +001

e) nenhuma das respostas anteriores

a)

TH.80 (MACK-74) Uma tangente a curva f(x) = x2 é paralela à reta 8x - 2y + 5 ponto de tangência é: a)

é:

e) y = 6x + 5

di y = 3x - 2

12

""5 =

y = O

TH.79 (MACK-73) A equação da reta tangente a curva y

c) y -

do ponto de tangência.

TH.87 (FEI-67) Se a derivada da função

ai

12

- 3)

rivado tem como gráfico uma reta tangente ao gráfico do trinômio. E ntão, a ordenada

x2

TH.78 (PUC-73) A equação da tangente à curva y = x 2 + 1 no ponto da abscissa

x - 2y =

> O,

3

12

-"5 = 10 (x

TH.86 (CESCEM-73) Dado o trinômio do 2 0 grau y

TH.77ICESCEA-73) A equação da reta tangente à curva

a) y

x

x

x-H 1 - x

c) 1;

b) -1;

f'lx) ~ dd [Iogex] =~, uv

e

bl y

é:

logex L = lim - - - é igual a:

podemos concluir que a) O;

=

5

12 P = 13'5)

TH.88 (EPUSP-65) Se a derivada de um polinômio

anteriore~'.

c)

(-00, -1)

e

(1, +00)

Plx) apresentar o seguinte gráfico

y

x Ix I, no pontr

d) y = -2x

e) y =

2x + 1

TH.82 (GV-71) O ponto (x, y) do I? quadrante no qual a tangente ao gráfico da função dada por y x 3 - 6x é paralela ao eixo dos x é tal que x 2 vale: b) 25

a) O

d) 2

cl 4

e) nenhuma das alternativas anteriores

TH.83 (MACK-74) Seja a curva de equação y = tg x. A tangente a esta curva no ponto de abscissa x = rr/4 é perpendicular à reta:

ai x - 2y + 3 d)

x +

= O

2y + 3

=O

b) 2x - y + 3 = O e) x

+Y

c) 2x + 2y - 3

(5,3)

d) (-3, -5)

240-H

b) Hi.3)

O

= O

TH.84 (E.E.L1NS-68) O ponto de eontaeto da tangente à curva y = reta 5x + 3y - 2 O é: a)

=

c)

(-3. 51

Vx2

-

16 e paralela à

a) b) c) d)

P(x) P(x) P(x) P(x)

será crescente de 1 a 2 e decrescente de 2 a 3 terá três zeros reais e distintos apresentará um máximo para x = 2 se anulará para x = I

e) nenhuma das respostas anteriores

TH.89 (CONSART-751 Sabendo que a função derivada f' é estritamente crescente e que f'(3) = -1, o gráfico que pode representar f é

e) nenhuma das respostas anteriores

241-H


(C)

(B)

o.

a) tem máximo no ponto

x =

b) tem mínimo no ponto

x = O.

c} não tem máximo nem mínimo. d) tem máximo e mínimo

e) nenhuma das afirmações anteriores é verdadeira.

TH.96 (PUC-78) A função

y

x2 + 1

~ x 2 _ 4 (x 01= ± 21 tem um ponto de máximo para x igual

a:

b) 2

a)

TH.97 (FFCLUSP-69) x 3 - (4

(E)

(DI

+

mlx 2

d) O

c) -1

el -2

Para que a equação algébrica

+ (4 + 4m)x - 4 m ~ O

admita o valor 2 como raiz dupla, o valor de a)

y ~ x3 :

a) tem valor máximo para bl tem valor mínimo para

c) tem um extremo em TH.91 (PUC-71) A função ai O

x = O x = O

>O

d) <3

el n.r.a.

d) não tem rnaxlmo nem m(nimo e) não tem tangente no ponto x = O

TH.98 (CESCEA-74) Na figura abaixo, a reta r é tangente ao gráfico da função dada por 2 y = x , no ponto (2, 4). Determinar a área hachurada da figura sabendo que ela vale 2/3 da tangente do ángulo a.

y ~ x 3 - 3x

tem um ponto de mínimo relativo para x igual a: 1 c) -1 d) 3 el

:r-

TH.92 (CICE-701 O maior valor de x 2 -

b) -3

Ix I +

no intervalo

[-3, 3]

di 6

cl O

b) (-4, O), (-2, -4)

(-2, -4), (O, O)

el (-4, O). (-2, 41, (O, O)

é: e) 7

TH.93 (CICE-6S) OIs) extremo(sl relativo(s) do gráfico da função y ~ a)

c)

x = O

b) 1

a) 2

b) 2

NOÇÕES DE INTEGRAL

O

TH.90 (PUC-71) Afunção

01=2

m deve ser:

Ix 2 + 4x I

são:

c)(-2, 4), (-2, -4)

d) (-2, -41

x

TH.94 (PUC-74) O ponto de máximo da função y = 3x s - 25x 3 a)

Xo

=

O

+ b)

60x - 1

Xo

no intervalo

= -1

c)

Xo

~

[-

2

~ ,3]

é:

d) xo

ai 8/3

=

e) Xo

b) 10/3

c) 413

d) 14/3

TH.99 (EPUSP-67) A derivada da função

TH.95 (USP-67) A função

242-H

1 y = 1 + x2

e) não sei.

= -2 1

f(x)

é~. Se

2T. +

1

fIO) ~ 1,

então

f(1)

é

igual a: a) O

c)

4

d) nenhuma das respostas anteriores

243-H


vale y' ~ 15x 2 - 6x + 2. Sa· TH.100 (CICE-6 8) Seja y(x) uma função cuja primeira derivada a: igual é y(x) bendo que y(1) ~ 5, d) 5x 3 - 3x 2 + x + 2 a) 3x 2 + 2x el 5x 3 - 3x 2 + 2x b) 3x 3 - 2x + 4 c) 5x 3 - 3x 2 + 2x + 1

RE SP OS TA S

TH.1 b TH.2 d TH.3 b TH.4 d TH.5 b l:H.6 e TH.7 d TH.8 d TH.9 a TH.10e TH.11 c TH.12d TH.13e TH.14c TH.15a TH.16b TH.17 a TH.18c TH.1ge TH.20c TH.21 b TH.22 c TH.23d TH.24 a TH.25d

244-H

TH.26b TH.27d TH.28d TH.29b TH.30b TH.31 d TH.32b TH.33 c TH.34b TH.35b TH.36 c TH.37 a TH.38b TH.39d TH.40d TH.41 e TH.42d TH.43b TH.44b TH.45e TH.46b TH.47b TH.48 e TH.49 e TH.50c

TH.51 a TH.52d TH.53d TH.54 e TH.55 c TH.56 c TH.57b TH.58e TH.59 c TH.60c TH.61 c TH.62b TH.63 c TH.64d TH.65d TH.66 a TH.67 a TH.68 c TH.69d TH.70a TH.71e TH.72c TH.73d TH.74d TH.75 e

TH.76b TH.77 a TH.78e TH.79d TH.80b TH.81 e TH.82d TH.S3d TH.S4b TH.85 e TH.86 a TH.87c TH.88 e TH.8ge TH.90d TH.91b TH.92e TH.93 e TH.94d TH.95a TH.96d TH.97a TH.98 a TH.99 c TH.100c


Turn static files into dynamic content formats.

Create a flipbook
Issuu converts static files into: digital portfolios, online yearbooks, online catalogs, digital photo albums and more. Sign up and create your flipbook.